mcs theory

94
Q.1) Explain briefly features of an IDEAL management control system? Management control is a process of assuming that resources are obtained and used effectively and efficiently in the accomplishment of the organization’s objectives. It is a fundamental necessity for the success of a business and hence from time to time the current performance of the various operations is compared to a predetermined standard or ideal performance and in case of variance remedial measures are adopted to confirm operations to set plan or policy. Some of the features of MANAGEMENT CONTROL SYSTEM are as follows: Total System: MANAGEMENT CONTROL SYSTEM is an overall process of the enterprise which aims to fit together the separate plans for various segments as to assure that each harmonizes with the others and that the aggregate effect of all of them on the whole enterprise is satisfactory. Monetary Standard: MANAGEMENT CONTROL SYSTEM is built around a financial structure and all the resources and outputs are expressed in terms of money. The results of each responsibility centre in respect to production and resources are expressed in terms of a common denominator of money. Definite pattern: It follows a definite pattern and time table. The whole operational activity is regular and rhythmic. It is a continuous process even if the plans are changed in the light of experience or technology. Coordinated System: It is a fully coordinated and integrated system. Emphasis: Management control requires emphasis both on the search for planning as well as control. Both should go hand in hand to achieve the best results. Function of every manager: Manager at every level as to focus towards future operational and accounting data, taking into consideration past performance, present trends and anticipated economic and technological changes. The nature, scope and level of control will be governed by the level of manager exercising it. Existence of goals and plans: MANAGEMENT CONTROL SYSTEM is not possible without predetermined goals and plans. These two provide a link between such future anticipations and actual performance. Forward looking: MANAGEMENT CONTROL SYSTEM is on the basis of evaluation of past performance that the future plans or guidelines can be laid down. Management Control involves managing the overall activity of the enterprise for the future. It prevents deviations in operational goals. Continuous process: It is a continuous process over the human and material resources. It demands vigilance at every step. Deciding, planning and regulating the activities of people associated in the common task of attaining the objectives of the organization is a the primary aim of MANAGEMENT CONTROL SYSTEM. People oriented: It is the managers, engineers and operators which implement the ideas and objectives of the management. The coordination of the main division of an organization helps in smoother operations and less friction which results in the achievement of the predetermined objectives. Scope of control MANAGEMENT CONTROL SYSTEM is an important process in which accounting information is used to accomplish the organizations objectives. Therefore the scope of control is very wide which covers a very wide range of management activities. Policies control: Success if a business depends on formulation of sound policies and their proper implementation. Control over organization: It involves designing and organizing the various departments for the smooth running of the business. It attempts to remove the causes of such friction and rationalizes the organizational structure as and when the need arises. Control over personnel: Anything that the business accomplishes is the result of the action of those people who work in the organization. It is the people, and not the figures, that get things done. Control over costs: The cost accountant is responsible to control cost sets, cost standards, labour material and over heads. He makes comparisons of actual cost data with standard cost. Cost control is a delicate task and is supplemented by budgetary control systems.

Upload: junaid-khan

Post on 31-Mar-2015

147 views

Category:

Documents


3 download

TRANSCRIPT

Page 1: MCS theory

Q.1) Explain briefly features of an IDEAL management control system?

Management control is a process of assuming that resources are obtained and used effectively and efficiently in the accomplishment of the organization’s objectives. It is a fundamental necessity for the success of a business and hence from time to time the current performance of the various operations is compared to a predetermined standard or ideal performance and in case of variance remedial measures are adopted to confirm operations to set plan or policy.Some of the features of MANAGEMENT CONTROL SYSTEM are as follows:

Total System: MANAGEMENT CONTROL SYSTEM is an overall process of the enterprise which aims to fit together the separate plans for various segments as to assure that each harmonizes with the others and that the aggregate effect of all of them on the whole enterprise is satisfactory.

Monetary Standard: MANAGEMENT CONTROL SYSTEM is built around a financial structure and all the resources and outputs are expressed in terms of money. The results of each responsibility centre in respect to production and resources are expressed in terms of a common denominator of money.

Definite pattern: It follows a definite pattern and time table. The whole operational activity is regular and rhythmic. It is a continuous process even if the plans are changed in the light of experience or technology.

Coordinated System: It is a fully coordinated and integrated system.

Emphasis: Management control requires emphasis both on the search for planning as well as control. Both should go hand in hand to achieve the best results.

Function of every manager: Manager at every level as to focus towards future operational and accounting data, taking into consideration past performance, present trends and anticipated economic and technological changes. The nature, scope and level of control will be governed by the level of manager exercising it.

Existence of goals and plans: MANAGEMENT CONTROL SYSTEM is not possible without predetermined goals and plans. These two provide a link between such future anticipations and actual performance.

Forward looking: MANAGEMENT CONTROL SYSTEM is on the basis of evaluation of past performance that the future plans or guidelines can be laid down. Management Control involves managing the overall activity of the enterprise for the future. It prevents deviations in operational goals.

Continuous process: It is a continuous process over the human and material resources. It demands vigilance at every step. Deciding, planning and regulating the activities of people associated in the common task of attaining the objectives of the organization is a the primary aim of MANAGEMENT CONTROL SYSTEM.

People oriented: It is the managers, engineers and operators which implement the ideas and objectives of the management. The coordination of the main division of an organization helps in smoother operations and less friction which results in the achievement of the predetermined objectives.

Scope of control

MANAGEMENT CONTROL SYSTEM is an important process in which accounting information is used to accomplish the organizations objectives. Therefore the scope of control is very wide which covers a very wide range of management activities.

Policies control: Success if a business depends on formulation of sound policies and their proper implementation.

Control over organization: It involves designing and organizing the various departments for the smooth running of the business. It attempts to remove the causes of such friction and rationalizes the organizational structure as and when the need arises.

Control over personnel: Anything that the business accomplishes is the result of the action of those people who work in the organization. It is the people, and not the figures, that get things done.

Control over costs: The cost accountant is responsible to control cost sets, cost standards, labour material and over heads. He makes comparisons of actual cost data with standard cost. Cost control is a delicate task and is supplemented by budgetary control systems.

Control over techniques: It involves the use of best methods and techniques so as to eliminate all wastages in time, energy and material. The task is accomplished by periodic analysis and checking of activities of each department with a view to avoid an eliminate all non-essential motions, functions and methods.

Control over capital Expenditure: Capital budget is prepared for the whole concern. Every project is evaluated in terms if the advantage it accrues to the firm. For this purpose capital budgeting, project analysis, study of cost of capital etc are carried out.

Overall control: A master plan is prepared for overall control and all the departments of the concern are involved in this procedure.

Q.2 What is the concept of free cash flow as applied to organization. Explain process of computation?

Page 2: MCS theory

We define net cash flow as net income plus non cash adjustment which typically means net income plus depreciation though that cash flows cannot be maintained over time unless depreciated fixed assets are replaced. So management is not completely free to use its cash flows however it chooses. Therefore we define the term free cash flows.

Free cash flow is the cash flow actually available for distribution to investor after the company has made all the investment in fixed assets and working capital necessary to sustain ongoing operation. When we studied income statement in accounting the emphasis was probably on the firm’s net income, which is accounting profit. However the value of company’s operation is determined by the stream of cash flows that the operations will generate now and in the future. To be more specific, the value of operation depends on all the future expected free cash flows, defined as after- tax operating profit minus the amount of new investment in working capital and fixed assets necessary to sustain the business. Therefore the way for managers to make their companies more valuable is to increase their free cash flow.Uses of FCF:1. Pay interest to debt holders, keeping in mind that the net cost to the company is the after tax interest expense.2. Repay debt holders, that is, pay off some of debt.3. Pay dividends to shareholders.4. Repurchase stock from shareholders.5. Buy marketable securities or other non operating assets.

In practice, most companies combine these five uses in such a way that the net total is equal to FCF. For example, a company might pay interest and dividends, issue new debts, also sell some of its marketable securities. Some of these activities are cash outflows (paying interest and dividends) and some are cash inflows (issuing debt and selling marketable securities), but the net cash flow from these five activities is equal to free cash flows.

Computation of free cash flows:Eg:Suppose the company had a 2001 NOPAT of $170.3million and depreciation is only the non cash charge which is $100million then its operating cash flow in 2001 would be NOPAT plus any non cash adjustment on the statement of cash flows.

Operating cash flow =NOPAT +depreciation (non cash adjustment) = $17.03 + $100 = $270.3

Company has $1,455million operating assets, at the end of 2000, but $1,800 at the end of 2001.it made a net investment in operating assets of

Net investment in operating assets = $18, 00 - $1,455 = $345million

If net fixed assets rose from $870million to $1000million however company reported $100million of depreciation. So its gross investment in fixed assets would be

Gross investment = net investment + depreciation = $130 + $100 = $230million

Company free cash flows in 2001 was FCF = operating cash flow – gross investment in operating assets = $270.3 - $445 = - $174.7million

An algebraically equivalent equation is

FCF = NOPAT - Net investment in operating assets = $170.3- $345 = - $174.7million

Even though company had a positive NOPAT, its very high investment in operating assets resulted in a negative free cash flow. Because free cash flow is what is available for distribution to investor, not only was there nothing for investors, but investor actually had to provide additional money to keep the business ongoing. A negative current FCF not necessarily bad provided it is due to the high growth or to support the growth. There is nothing wrong with profitable growth; even it causes negative free cash flow in the short term

Q.3) What is Balance Scorecard? What is the process of implementation and difficulties in

implementation?

The Balanced Scorecard (BSC) is a performance management tool which began as a concept for measuring whether the smaller-scale operational activities of a company are aligned with its larger-scale objectives in terms of vision and strategy.

By focusing not only on financial outcomes but also on the operational, marketing and developmental inputs to these, the Balanced Scorecard helps provide a more comprehensive view of a business, which in turn helps organizations act in their best long-term interests.

The underlying rationale is that organizations cannot directly influence financial outcomes, as these are "lag" measures, and that

2

Page 3: MCS theory

the use of financial measures alone to inform the strategic control of the firm is unwise. Organizations should instead also measure those areas where direct management intervention is possible. In so doing, the early versions of the Balanced Scorecard helped organizations achieve a degree of "balance" in selection of performance measures. In practice, early Scorecards achieved this balance by encouraging managers to select measures from three additional categories or perspectives: "Customer," "Internal Business Processes" and "Learning and Growth."

The balance scorecard suggests that we view the organization from four perspectives, and to develop metrics, collect data and analyze it relative to each of these perspectives:

The learning and growth perspective : “To achieve our vision, how will we sustain our ability to change and improve?”

The business process perspective : “To satisfy our shareholders and customers what business processes must we excel at?”

The customer perspective : “To achieve our vision, how should we appear to our customer?”

The financial perspective : “To succeed financially, how should we appear to our shareholders?”

Implementing a Balanced Scorecard

We can summarize the implantation of a balanced scorecard in four general steps;1. Define strategy.2. Define measure of strategy.3. Integrate measures into the management system.4. Review measures and result frequently.

Each of these steps is iterative, requiring the participation of senior executive and employees throughout the organization

Define StrategyThe balance scorecard builds a link between strategy and operational action. As a result it is necessary to begin the process of defining a balanced scorecard by defining the organization goals are explicit and what that targets have been developed.

Define Measures of StrategyThe next step is to develop measures in support of the articulate strategy. It is imperative that the organization focuses on a few critical measures at this point; otherwise management will be overloaded with measures. Also, it is important that the individual measures be linked with each other in a cause effect manner

Integrated Measures into the management systemThe balanced scorecard must be integrated with the organization formal and informal structure, its culture, and its human resources practice. While the balanced Scorecard gives some means for balancing measures, the measures can still become unbalanced by others system in the organization such as compensation policies that compensate the manager strictly based on financial performance.

Review Measures and result FrequentlyOnce the balance scorecard is up and running it must be consistently reviewed by senior management. The organization should be looking for the following

How do the outcome measures say the organization is doing? How do the driver measures say the organization is doing? How has the organization’s strategy changed since the last review? How has the scorecard measures changed?

The most important aspects of these reviews are as follows;

They tell management whether the strategy is being implemented correctly and how successfully the strategy is working.

They show that management is serious about the importance of these measures. They maintain alignment of measure to ever changing strategies.

Difficulties in implementing Balanced Scorecard

The following problems unless suitably dealt with, could limit the usefulness of the balanced scorecard approach: Poor correlation between nonfinancial measures and result. Fixation on financial result. No mechanism for improvement. No mechanism for improvement. Measures overload.

Poor Correlation between Nonfinancial measures and resultSimply put there is no guarantee that future profitably will allow targets achievement in any nonfinancial area. This is probably the biggest problem with the balanced scorecard because there is an inherent assumption that future profitability does follow from achieving the scorecard measures, identifying the cause effect relationships among the different measures is easier said than done.

This will be a problem with any system that is trying to develop proxy measures for future performance. While this does not mean that the balanced Scorecard should be abandoned it is imp that comp adopting such a system understand that the links between nonfinancial measures and financial performance are still poorly understood.

3

Page 4: MCS theory

Fixation on Financial ResultsAs previously discussed not only are most senior managers well trained and very adept with financial measures but they also most keenly feel pressure regarding the financial performance of their comp. Shareholder are vocal and the board of directors often applies pressure on the stakeholders behalf .this pressure often overwhelms the long term uncertain payback of the nonfinancial measures.

Non mechanism for ImprovementOne of the most overlooked pitfalls of the balanced scorecard is that a company cannot achieve Stretch goals if the Company has no mechanism for improvement .Unfortunately achieving many of these goals require complete shifts in the way that business is done yet the company often does not have mechanism to make those shifts . The mechanism available takes additional resource and requires a changed in the company culture. These changes do not happen overnight nor do they respond automatically to a new stretch targets. Inertia often works against the company employees are accustomed to a self limited cycle of setting targets, missing those targets and readjusting the targets to reflect what was actually achieved. Without a method for making improvement, improvements are unlikely to consistently happen no matter how good the stretch goal sound.

Measurement overloadHow many critical measures can one manager track at one time without losing? Unfortunately there is no right answer to this question except it is more than 1 and less than 50. It too few then the manager is ignoring measures that are critical to creating success. If it too many then the manager may risk losing focus and trying to do too many things at once.

Q.5ABC ltd. (MCS-2008) Numerical

Particulars Division X (Rs.) Division Y (Rs.)ROI 28% 26%Sales 100 Lacs 500 lacsInvestment 25 lacs 100 LacsEBIT 7 Lacs 26 lacs

Analyze and comment upon performances of both the divisionsSolution:

Division XROI = (Profit / investment)* 100Profit = (28/100)*25lacs = 7lacs

Profit margin = (Profit/sales)*100= (7/100)*100

= 7lacs

Turnover of investments = (Sales/investment)*100 = (100/25)*100 = 4 times

Division YROI = (Profit / investment)* 100Profit = (26/100)*100lacs = 26lacs

Profit margin = (Profit/sales)*100 = (26/500)*100 = 5.2lacs

Turnover of investments = (Sales/investment)*100= (500/100)*100

= 5 times

Profit margin of X is better than profit margin of division Y. Turnover of investment of division Y is better than Division X.

Hence cost management of Division X is better than Division Y.

Set 2

Q1. MCS designers apparently disagree whether single measure to evaluate the profit performance and capital

investment performance is preferable or SEPARATE measures for each are preferable – COMMENT ?

ANS. There should be different measures used for evaluating profit performance and capital investment performance as

needed.The goal of performance measurement systems is to implement strategy. In setting up such systems, senior

management selects measures that best represent the company's strategy. These measures can be seen as current and

future critical success factors; if they are improved, the company has implemented its strategy. The strategy's success

4

Page 5: MCS theory

depends on its soundness. A performance measurement system is simply a mechanism that improves the likelihood the

organisation will implement its strategy successfully. 

Measuring Profitability  

There are two types of profitability measurements used in evaluating a profit center, just as there are in evaluating an

organization as a whole. First, there is a measure of management performance, which focuses on how well the manager is

doing. This measure is used for planning, coordinating, and controlling the profit center's day-to-day activities and as a device

for providing the proper motivation for its manager. Second, there is the measure of economic performance, which focuses on

how well the profit center is doing as an economic entity. The messages conveyed by these two measures may be quite

different from each other. For example, the management performance report for a branch store may show that the store's

manager is doing an excellent job under the circumstances, while the economic performance report may indicate that

because of economic and competitive conditions in its area the store is a losing proposition and should be closed.  

  The necessary information for both purposes usually cannot be obtained from a single set of data. Because the management

report is used frequently, while the economic report is prepared only on those occasions when economic decisions must be

made, considerations relating to management performance measurement have first priority in systems design-that is, the

system should be designed to measure management performance routinely, with economic information being derived from

these performance reports as well as from other sources. 

Capital Investment Measurement : Most proposals require significant new capital. Techniques for analyzing

capital investment proposals attempt to find either (a) The net present value of the project, that is, the excess of the

present value of the estimated cash inflows over the amount of investment required, or

(b) The internal rate of return implicit in the relationship between inflows and outflows. An important point is that these

techniques are used in only about half the situations in which, conceptually, they are applicable.  

There are at least four reasons for not using present value techniques in analyzing all proposals.  

1) The proposal may be so obviously attractive that a calculation of its net present value is unnecessary. A newly

developed machine that reduces costs so substantially that it will pay for itself in a year is an example.

2) The estimates involved in the proposal are so uncertain that making present value calculations is believed to be not

worth the effort-one can't draw a reliable conclusion from unreliable data. This situation is common when the results

are heavily dependent on estimates of sales volume of new products for which no good market data exist. In these

situations, the "payback period" criterion is used frequently.

3) The rationale for the proposal is something other than increased profitability. The present value approach assumes

that the "objective function" is to increase profits, but many proposed investments win approval on the grounds that

they improve employee morale, the company's image, or safety.

4) There is no feasible alternative to adoption. Environmental laws may require investment in a new program,as an

example. The management control system should provide an orderly way of deciding on proposals that cannot be analyzed by

quantitative techniques. Systems that attempt to rank non-quantifiable projects in order of profitability won't work. Many

projects do not fit into a mechanical ranking scheme.

Q.No. 2. What are the different methods to measure profits of a profit center in organizations? Which different

messages each type of measure is likely to convey to managers?

Ans: When financial performance in a responsibility center is measured in terms of profit, which is the difference between

the revenues and expenses, the responsibility center is called a profit center.Profit as a measure of performance is especially

useful since it enables senior management to use one comprehensive measure instead of several measures that often point to

different directions.

5

Page 6: MCS theory

There are two types of profitability measurements in a profit center, just as there are for the organization as a whole. There is,

first, a measure of management performance, in which the focus is on how well the manager is doing. This measure is used

for planning, coordinating and controlling the day-to-day activities of the profit center. Second, there is a measure of

economic performance, in which the focus is on how well the profit center is doing as an economic entity. The message

given by these two measures may be quite different.

Types of Profitability measures:

In order to evaluate the economic performance of a profit center, one must use net income after allocating all costs. However,

in evaluating the performance of manager, any of five different measures of profitability can be used.

1) Contribution Margin: The logic behind using contribution margin as a measure is that fixed expenses are not

controllable by the manager, and therefore he should focus on maximizing the spread between revenue and

expenses. But the problem with this is that some fixed costs are controllable and all fixed costs are partially

controllable. A focus on the contribution margin tends to direct attention away from this responsibility.

2) Direct Profit: This measure shows the amount that the profit center contributes to the general overhead and profit

of the corporation. It incorporates all expenses incurred in or directly traced to the profit center, regardless of

whether these items are entirely controllable by the profit center manager. A weakness of this measure is that it does

not recognize the motivational benefit of charging headquarters costs.

3) Controllable Profit: Headquarters expenses are divided into two categories: controllable and non-controllable. The

controllable expenses are controlled by business unit manager. Consequently, if these costs are included in the

management system, the profit will be after the deduction of all expenses that are influenced by profit center

manager.

4) Income before Taxes: In this measure, all corporate overhead is allocated to profit centers. The basis of allocation

reflects the relative amount of expense that is incurred for each profit center. If corporate overheads are allocated to

profit centers, budgeted costs, not actual costs, should be allocated. Then the performance report will show an

identical amount in the “budget” and “actual” columns for such overheads.

5) Net Income: Here, companies measure performance of domestic profit centers at the bottom line, the amount of net

income after income tax. There are two arguments 1) Income after tax is constant percentage of the pretax income,

so there is no advantage in incorporating income taxes 2) many decisions that have impact on income taxes are

made at headquarters, and it is believed that profit center manager should not be judged by the consequences of

these decisions.

Q3: Explain special characteristics of professional organizations which impact Management Control. What are

interactive controls?

Special Characteristic of a Professional Organization:

1. Goals

A goal of a manufacturing company is to earn a satisfactory profit specially a satisfaction profit, specially a satisfactory return

on assets its principle assets is the skill of its professional staff which doesn’t appear on its balance sheet .return on assets

employed therefore is essential meaningless in such organization .their financial goal is to provide adequate compensation to

the professional.

2. Professionals

Professional organization is labour intensive and the labour is of a special type. Research and development organization use in

setting selling price and for other management purposes .standard cost system ,separation of fixed and variable cost and

analyses of variance were built on the foundation are example of organization whose product are professional service.

Professional tends to give in adequate weight to the financial implication of their decision they want to do the best job they can

regardless of its cost.

Because profession are the organization most important resource some authors have advocated that the value of these

profession should be counted as assets the system that does this is called human resource accounting .in the 1970’s many

6

Page 7: MCS theory

books and articles were written on this subject but few comp actually such a system and we do not know of any that one

current .the problem of measuring the value of human assets is intractable.

3. Output and input measurement

The output of a profession organisation cannot be measured in physical terms, use in setting selling price and for other

management purposes .standard cost system, seperstion of fixed and variable cost and analyses of variance were built on the

foundation. We can measures the number of patient a physician treats n a day and even classify these visit by type of

complaint but this is by no means equivalent to measuring the amt or quality earned is one measures of output in some

professional organization but these monetary amts at most relate to the quantity of service rendered not to their quality.

Some profession notably scientist engineer, and professional are reluctant to keep track of how they spend their time and this

complicate the track of measuring performance .this reluctant seems to have its root in tradition usually it can be overcome if

senior management is willing to put appropriate emphasis on the necessity for accurate time reporting .nevertheless difficult

problem arise in deciding how time should be charged to clients .if the normal work week is 40 hrs should a job be charged for

1/40th of a week compensation for each other spent on it? If so how should work done on evening and weekend be counted how

to account for time spent reading literature ,going to meeting ,and otherwise keeping up to date?

4. Small Size

With a few exception such as some law firm and accounting firms ,professional organisations are relatively small and operate

at a single location .senior management in such organisations can personally observe what is going on and personally motivate

employee .thus there is less need for a sophisticated management control system ,with profit centres and formal performance

reports nevertheless even a small organisations need a budget a regular comparison of performance against budget ,and a way

relating compensation to performance.

5. Marketing

In a manufacturing company there is a dividing line between marketing activities and production activities only senior

management is concerned with both .such a clean separation does not exist in most Professional organisation, however their

time and this complicate the track of measuring performance .this reluctant seems to have its root in tradition usually it can be

overcome if senior management is willing to put appropriate emphasis on the necessity for accurate time reporting.

Nevertheless difficult problem arise in deciding how time should be charged to clients .if the normal work. These marketing

activities are conducted by professional usually by professional, usually by professional who spend much of their time in

production work that is working for clients.

In such situation it is difficult to assign appropriate credit to the person responsible for selling a new customer; in a consulting

firm for example a new engagement may result from a conversation between a member of the firm or from the reputation of

one of the firm professional as an outgrowth of speeches or articles. Moreover the profession al who is responsible for

obtaining the engagement may not personally involved in carrying it out .until fairly recently these marketing contribution

were rewarded subjectively –that is they were taken into account in promotion and compensation decisions .some

organisation now give explicit credit, perhaps as a percentage of the project revenue, if the person revenue, if the person who

hold sold the project can be identified.

What is Interactive Control?

Interactive control alerts management of strategic uncertainties either trouble or opportunities that become the basis for

manager to adapt to a rapidly changing environments by thinking about new strategies.

1. A subset of the management control information that has a bearing on the strategic uncertainties facing the buss

becomes the focal point.

2. Senior executive take such information seriously.

3. Managers at all levels of the org focus attention on the information produced by the system.

Q5: Shandilya Ltd. (MCS-2008) Numerical

7

Page 8: MCS theory

Shandilya Ltd. has adopted Economic Value Added (EVA) technique for the appraisal of performance of its three divisions A,B

and C. Company charges 6% for current assets and 8 % for Fixed Assets, while computing EVA relevant data are given

below :-

Particulars Div A Div B Div C Total

Budgete

d

Actual Budgete

d

Actual Budgeted Actual Budgete

d

Actual

Profit 360 320 220 240 200 200 780 760

Current Assets 400 360 800 760 1200 1400 2400 2520

Fixed Assets 1600 1600 1600 1800 2000 2200 5200 5600

Solution:

Particulars Div A Div B Div C Total

Budgete

d

Actual Budgete

d

Actual Budgeted Actual Budgete

d

Actual

ROA 18% 16% 9% 9% 6% 6% 10% 9%

EVA 208 170.4 44 50.4 -32 -60 220 160.8

b) Comment upon both methods, based on results.

There are three apparent benefits of an ROA measure. First, it is a comprehensive measure in that anything that effects the

financial statements is reflected in this ratio. Secondly, ROA is easy to calculate, easy to understand, and meaningful in

absolute sense. Finally, it is a common denominator that may be applied to any organizational units responsible for

profitability, no matter what its size or what business it practices. The performance of different units may be compared

directly to each other. Also, ROI data is available for competitors that can be used as a basis for comparison. Nevertheless, the

EVA approach has some inherent advantages over ROA.

There are three compelling reasons to use EVA over ROI. First, with EVA all business units have the same profit objective for

comparable investments. The ROI approach, on the other hand, provides different incentives for investment across business

units. For example, a business unit that is currently achieving 30% ROA would be most reluctant to expand unless it is able to

earn a ROI of 30% or more on additional assets. Second, decision that increase a centre’s ROI may decrease its overall profits.

Third advantage of EVA is that different interest rates may be used for different types of assets. For example, a relatively low

rate May be used for inventories while a higher rate may be used for different types of fixed assets.

SET .3

Q.1) Describe differences in budgeting perspective of engineered and discretionary expense centre

1.Expense centers:

Expenses center are responsibility centers for which input or expenses are measured in monetary terms, but for

which outputs are not measured in monetary terms. There are two general types: engineered expense center and

discretionary expense center. They correspond to two types of costs.. Engineered costs are elements of cost for

which the right or proper amount of costs that should be incurred can be estimated with a reasonable degree of

reliability. Costs incurred in factory for direct labour direct material component supplies and utilities are examples.

8

Page 9: MCS theory

2.Engineered expense centers:

Engineered expense center have the following characteristics:

1. Their inputs can be measured in monetary terms.

2. Their output can be measured in physical terms.

3. The optimal dollar amount of input required to produce one unit of output can be established

Engineered expense center usually are found in manufacturing operations. Warehousing, distribution, trucking and

similar units in the marketing organization also may be engineered expense center and so many certain

responsibility center within administrative and support department. Examples are accounts receivable account

payable and payroll section in the controller department personnel record and cafeteria in the human resource

department shareholder record in the corporate secretary department and the company motor pool. Such units

perform repetitive task for which standard cost can be developed

In an engineered expense center the output multiplied by the standard cost or each unit produced represents what

the finished product should have cost. When this cost is compared to actual costs, the difference between the two

represents the efficiency of the organization unit being measured.

We emphasize that engineered expense centers have other important tasks not measured by cast alone. The

effectiveness of these aspects of performance should be controlled. For example expenses center supervisor are

responsible for the quality of good and for the volume of production in addition to their responsibility for cost

efficiency. Therefore the type and amount of production is prescribed and specific quality standards are set so that

manufacturing costs are not minimized at the expense of quality. Moreover manager of engineered expense center

may be responsible for activities such a training that are not related to current production judgment about their

performance should include an appraisal of how well they carry out these responsibilities.

There are few if any responsibility center in which all cost items are engineered. Even in highly automated

production department the amount of indirect labour and of various services used can vary with management

discretion.

Thus, the term engineered costs center refers to responsibility center in which engineered cost predominate but it

does not imply that valid engineering estimates can be made for each and every cost item.

3.Discretionary expense center:

The output of discretionary expenses center cannot be measured in monetary terms. They include administration

and support units research and development organization and most marketing activities.

The term discretionary does not mean that management judgments are capricious or haphazard. Management has

decided on certain policies that should govern the operation of the company. One company may have a small

headquarter staff another company of similar size and in the same industry may have a staff that is 10 time as

large the management of both companies may be concerned that they made the correct decision on staff size but

there is no objective way judging which decision was actually better manager are hired and paid to make such

decision after such a drastic change the level of discretionary expenses generally has a similar pattern from one

year to the next.

The difference between budgeted and actual expense is not a measure of efficiency in a discretionary expense

centre it is simply the difference between the budgeted input and the actual input. It in no way measures the value

of the output, if actual expense do not exceed the budget amount, the manager has ‘lived within the budget ‘

however ,because by definition the budget does not purport to measure the optimum amount of spending we

cannot say that living within the budget is efficient performance .

4.Differences in budgeting perspective of engineered and discretionary expense centre

Budget preparation

The decision that management make about a discretionary expense budget are different from the decisions that it

makes about the budget for an engineered expense center. For the latter, management decides whether the

proposed operating budget represent the cost of performing task efficiently for the coming period. Management is

9

Page 10: MCS theory

not so much concerned with the magnitude of the task because this is largely determined by the actions of other

responsibility centers, such as the marketing departments’ ability to generate sales. In formulating the budget for

a discretionary expense center, however management principal task is to decide on the magnitude of the job that

should be done.

Incremental budgeting:

Here the current level expenses in a discretionary expense center is taken as a starting points this amount is

adjusted for inflation for anticipated changes in the workload of continuing tasks for special tasks and if the data

are readily available for the cost of comparable work in similar units.

There are two drawbacks to incremental budgeting. First because managers of these centers typically want to

provide more service they tend to request additional resources in the budgeting process and if they make a

sufficiently strong case these request will be granted. This tendency is expressed in Parkinson’s second law:

overhead costs tend to increase period. There is ample evidence that not all this upward creep in cost is necessary.

This problem is especially compounded by the fact that the current level of expenditure in the discretionary

expenses center is taken for granted and is not re-examined during the budget preparation process. Second when

a company faces a crises or when a new management takes over overhead costs are sometimes drastically

reduced without any adverse consequences.

Despite this limitation most budgeting in discretionary expense centers is incremental. Time does not permit the

more thorough analysis described in the next section.

Zero based review:

An alternative approach is to make a thorough analysis of each discretionary expense center on a schedule that

will cover all of them over a period of five year or so. That analysis provides a new base. There is a likelihood that

expenses will creep up gradually over the next five years and this is tolerated at the end of five years, another new

base is established. Such an analysis is often called a zero base review.

In contrast with incremental budgeting which takes the current level of spending as the starting point this more

intensive review attempts to build up de now the resources that actually are needed by the activity. Basic question

are raised;(1) should use customer?(2) what should the quality level be ?are we doing too much(3)should the

function be performed in this way (4) how much should it cost?

Cost variability:

In discretionary expense center costs tend to vary with volume from one year to the next but they tend not to vary

with short run fluctuation in volume within a given year. By contrast costs in engineering expense center are

expected to vary with short run changes in volume. In part this reflect the fact that volume changes do have an

impact throughout the company even though their actual impact cannot be measures the ; in part this reflect the

fact that volume changes do have an impact throughout the company even though their actual impact cannot be

measured in part this result from a management personnel and personnel related costs are by far the largest

expense item in most discretionary expense center the annual budget for these center tend to be a constant

percentage of budgeted sales volume.

Q.2) Explain some factors which may influence top management style and the implication of the top

management style on management control.

The management control function in an organization is influenced by the style of senior management. The style of

the chief executive officer affects the management control process in the entire organization. Similarly, the style of

the business unit manager affects the unit's management control process, and the style of functional department

managers affects the management control process in their functional areas.  

Differences in Management Styles

Managers manage differently. Some rely heavily on reports and certain formal documents; others prefer

conversations and informal contacts. Some are analytical; others use trial and error. Some are risk takers; others

are risk averse. Some are process oriented; others are results oriented. Some are long-term oriented; others are

short-term oriented. Some emphasize monetary rewards; others emphasize a broader set of rewards.  

Management style is influenced by the manager's background and personality. Background includes things like

age, formal education, and experience in a given function, such as manufacturing, technology, marketing, or

10

Page 11: MCS theory

finance. Personality characteristics include such variables as the manager's willingness to take risks and his or her

tolerance for ambiguity. 

Implications for Management Control

The various dimensions of management style significantly influence the operation of the control systems. Even if

the same reports with the same set of data go with the same frequency to the CEO, two CEOs with different styles

would use these reports very differently to manage the business units.  

Style affects the management control process – how the CEO prefers to use the information, conducts performance

review meetings, and so on – which in turn affects how the control system actually operates, even if the formal

structure does not change under a new CEO. In fact, when CEOs change, subordinates typically infer what the new

CEO really wants based on how he or she interacts during the management control process.  

Personal versus Impersonal Controls

Presence of personal versus impersonal controls in organizations is an aspect of managerial style. Managers differ

on how much importance they attach to formal budgets and reports as well as informal conversations and other

personal contacts. Some managers are "numbers oriented"; they want a large flow of quantitative information, and

they spend much time analyzing this information and deriving tentative conclusions from it. Other managers are

"people oriented"; they look at a few numbers, but they usually arrive at their conclusions by talking with people,

judging the relevance and importance of what they learn partly on their appraisal of the other person. They visit

various locations and spend time talking with both supervisors and staff to get a sense of how well things are

going.

Managers' attitudes toward formal reports affect the amount of detail they want, the frequency of these reports,

and even their preference for graphs rather than tables of numbers, and whether they want numerical reports

supplemented with written comments. Designers of management control systems need to identify these

preferences and accommodate them. 

Tight versus Loose Controls

A manager's style affects the degree of tight versus loose control in any situation. The manager of a routine

production responsibility center can be controlled relatively tightly or loosely, and the actual control reflects the

style of the manager's superior. Thus, the degree of tightness or looseness often is not revealed by the content of

the forms or aspects of the formal control documents, rules, or procedures. It is a factor of how these formal

devices are used. The degree of looseness tends to increase at successively higher levels in the organization

hierarchy: higher-level managers typically tend to pay less attention to details and more to overall results.  

The style of the CEO has a profound impact on management control. If a new senior manager with a different style

takes over, the system tends to change correspondingly. It might happen that the manager's style is not a good fit

with the organization's management control requirements. If the manager recognizes this incongruity and adapts

his or her style accordingly, the problem disappears. If, however, the manager is unwilling or unable to change, the

organization will experience performance problems. The solution in this case might be to change the manager.

Q.3) Explain advantages and disadvantages of two step transfer pricing and profit sharing methods

Transfer pricing: If two or more profit center is jointly responsible for product development manufacturing

and marketing each should share in the revenue that is generated when the product is finally sold. The

transfer price is not primarily an accounting tool; rather, it is a behavioral tool that motivates manager to

make the right decisions. In particular the transfer price should be designed so that it accomplishes the

following objective: It should provide each segment with the relevant information required to determine the

optimum tradeoff between company cost and revenues It should induce goal congruent decisions that is the

system should be so designed that decision improve business unit to earn more profit It should help measure

the economic performance of the individual profit center

Two step pricing: First, a charge is made for each unit sold that is equal to the standard variable

cost of production. Second a periodic charge is made for the buying unit. One or both of these

components should include a profit margin. The two step pricing method correct this problem by

transferring variable cost on a per unit basis, and transferring fixed cost and profit on a lump sum

basis under this method the transfer price for product A would be 5$ for each unit that unit Y

purchases plus $20000 per month for fixed cost. Plus $10000 per month for profit: if transfer of

product A in a certain month are at the expected amount 5000 units then under the two step

11

Page 12: MCS theory

method unit y will pay the variable cost of $25000 plus $30000 for the fixed cost and profit a total

of $55000 .this is the same amount as the amount it would pay unit x if the transfer price is less

than 5000 units say 4000unoits.unit y would pay $50000 under the two step methods compared

with the $44000 it would pay if the transfer price were $11 per unit.

The difference is their transfer prices were for not using a portion of unit X capacity that it has

reserved. Note that under two step method the company variable cost for product A is identifiable

to unit Y variable cost for the product, and unit Y will make the correct short term marketing

decisions. Unit Y also has information on upstream fixed costs and profit related to product A and

it can use these data for long term decision.The fixed cost calculation in the two step pricing

method is based on the capacity that is reserved for the production of product A that is sold to unit

Y the investment represented by this capacity is allocated to product A. The return on investment

that unit X earns on competitive product is calculated and multiplied by the investment assigned

to the product. In the example we calculated the profit allowance as a fixed monthly amount. It

would be appropriate under some circumstance to divide the investment into variable and fixed

component. Then, a profit allowance based on a return on investment on variable assets would be

added to the standard variable cost for each unit sold.

Profit sharing: If the two step pricing system just described is not feasible, a profit sharing system might be

used to ensure congruence of business unit interest with company interest. This system operates somewhat as

follows.

1. The product is transferred to the marketing unit at standard variable cost.

2. After the product is sold, the business units share the contribution earned which is selling price minus the

variable manufacturing and marketing costs.

This method of pricing may be appropriate if the demand for the manufactured product is not steady enough to

warrant the permanent assignment of facilities as in the two step method. In general, this method accomplished

the purpose of making the marketing unit’s interest congruent with the companies. There are several practical

problems in implementing such profit sharing system. First, there can be arguments over the way contribution is

divided between the two profit centers. Which is costly, time consuming and work against basic reason for

decentralization namely autonomy of the business units mangers. Second, arbitrarily divided up the profit between

units does not give valid information on the profitability of each segment of the organization.

Third since the contribution is not allocated until after the sale has been made the manufacturing units

contribution depends upon the marketing unit’s ability to sell and on the actual selling price. Manufacturing units

may perceive this situation to be unfair

Two set of price: in this method, the manufacturing unit’s revenue is credited at the outside sales price,

and the buying unit is charged the total standard costs. The difference is changed to a headquarter

account and eliminated when the business unit statement are consolidated, this transfer pricing method is

sometimes used when there are frequent conflict between the buying and selling units that cannot be

resolved by one of the other method both the buying and selling

There are several disadvantages to the system of having two set of transaction prices, however

the sum of the business unit profit is greater than overall company profits, senior management must be

aware of this situation in approved budget for the business units and in subsequent evaluation of

performance against these budget. Also, this system create an illusion feeling that business units are

making money while in fact the overall company might be losing after taking account of the debits to

headquarter. Further this system might motivate business unit to concentrate more on internal transfers at

the expense of outside sales

The fact that the conflict between the business units would be lessened under this system could

be viewed as a weakness. Sometime, it is better for the headquarter to be aware of the conflict arising

out of transfer prices because such conflict may signal problem in either the organizational structure or In

other management systems. Under the two sets of prices method these conflicts are smoothed over

thereby not alerting senior management to these problems.

Q.4) Discuss special challenges faced in controlling R & D activities and possible management

initiatives

12

Page 13: MCS theory

Type of financial control: The financial control exercised in a discretionary expense center is quite

different from that in engineered center the latter attempts to minimize operating cost by setting a

standard and reporting actual costs against this standards. The main purpose of a discretionary expense

budget on the other hand is to allow the manager to control Cost for particular in the planning. Costs are

controlled primarily by deciding what task should be undertaken and what level of effort is appropriate for

each. Thus in a discretionary expense center financial control is primary exercised at the planning stage

before the amount are incurred.

Measurement of performance: The primary job of the manager of a discretionary expense center is to

accomplish the desired output spending an amount that is on budget is satisfactory. This is in contrast with

the report in an engineered expense center which helps higher management to evaluate the manger

efficiency. If these two types of responsibility center are carefully distinguished management may treat the

performance report for the discretionary expense center as if it were an indication of efficiency Control

over spending can be exercised by requiring that the manger approved be obtain before the budget is over

sometimes a certain percentage of overrun is permitted without additional approval if the budget really set

forth the best estimate of actual cost there is 50 percent probability that it will overrun and this is the

reason that some latitude is often permitted.

Control problems: The control of R & D centers, which are also discretionary expense center is difficult

for the following at least a semi tangible output reasons.

1. Results are difficult to measure quantitatively. As contrasted with administrative activities, R&D usually has at

least a semi tangible output in patent, new products, or new processes. Nevertheless, the relationship of these

outputs to inputs is difficult to measure and appraise. A complete product of an R&D group may require several

year of effort; consequently input as stated in an annual budget may be unrelated to outputs. Even if an output

can be identified a reliable estimate of its value often cannot be made. Even if the value of the output can be

calculated, it is usually not possible for management to evaluate the efficiency of the R&D effort because of its

technical nature. A brilliant effort may come up against an insuperable obstacle, whereas a mediocre effort

may, by luck result in a bonanza.

2. The goal congruence problem in R&D center is similar to that in administrative centers. The research managers

typically want to build the best research organization that money can buy, even though this is more expensive

than the company can afford. A further problem is that research people often may not have sufficient

knowledge of the business to determine the optimum direction of the research efforts.

3. Research and development can seldom be controlled effectively on an annual basis. A research project may

take year s to reach fruition, and the organization must be built up slowly over a long time period. The

principal cost is for the work force obtaining highly skilled scientific talented is often difficult, and short term

fluctuation in the work force are in efficient. It is not reasonable, therefore to reduce R&D costs in years when

profits are low and increase them in year when profits are high. R&D should be looked at as a long term

investment not as an activity that varies with short run corporate profitability.

The R&D continuum:Activities conducted by R&D organization lie along a continuum. At one extreme

is basic research; the other extreme is product testing. Basic research has two characteristics: first, it is

unplanned management at most can specify the general area that is to be explored second there is

often a very long time lag before basic research result in successful new product introductions. Financial

control system has little value in managing basic research activities. In some companies, basic research

in included as a lump sum in the research program and budget. In others, no specific allowance is made

for basic research as such; there is an understanding that scientists and engineers can devote part of

their time to explorations in whatever direction they find most interesting, subject only to informal

agreement with their supervisor.For product testing projects, on the other hand, the time and financial

requirement can be estimated, not as accurately as production activities.

Q.5) Explain problems faced in pricing corporate services provided to business units organized as

Profit Centers

Services are intangible in nature. This characteristic of services makes it difficult for pricing. Charging business

units for services furnished by corporate staff units becomes challenging work due to intangibility of services.

While pricing corporate services, we exclude the cost of central service staff units over which business units have

13

Page 14: MCS theory

no control (e.g., central accounting, public relations, and administration). If these costs are charged at all, they are

allocated, and the allocations do not include a profit component. The allocations are not transfer prices.

We need to consider two types of transfers:

O For central services that the receiving unit must accept but can at least partially control the

amount used.

O For central services that the business unit can decide whether or not to use.

Business units may be required to use company staffs for services such as information technology and research

and development. In these situations, the business unit manager cannot control the efficiency with which these

activities are performed but can control the amount of the service received. There are three schools of thought

about such services.

One school holds that a business unit should pay the standard variable cost of the discretionary services. If it pays

less than this, it will be motivated to use more of the service than is economically justified. On the other hand, if

business unit managers are required to pay more than the variable cost, they might not elect to use certain

services that senior management believes worthwhile from the company's viewpoint. This possibility is most likely

when senior management introduces a new service, such as a new project analysis program. The low price is

analogous to the introductory price that companies sometimes use for new products.

A second school of thought advocates a price equal to the standard variable cost plus a fair share of the standard

fixed costs-that is, the full cost. Proponents argue that if the business units do not believe the services are worth

at least this amount, something is wrong with either the quality or the efficiency of the service unit. A third school

advocates a price that is equivalent to the market price, or to standard full cost plus a profit margin. The market

price would be used if available (e.g., costs charged by a computer service bureau); if not, the price would be full

cost plus a return on investment. The rationale for this position is that the capital employed by service units should

earn a return just as the capital employed by manufacturing units does. Also, the business units would incur the

investment if they provided their own service.

Optional Use of Services

In some cases, management may decide that business units can choose whether to use central service units.

Business units may procure the service from outside, develop their own capability, or choose not to use the service

at all. This type of arrangement is most often found for such activities as information technology, internal

consulting groups, and maintenance work. These service centers are independent; they must stand on their own

feet. If the internal services are not competitive with outside providers, the scope of their activity will be

contracted or their services may be outsourced completely.

For example, Commodore Business Machines outsourced one of its central service activities-customer service-to

Federal Express. James Reeder, Commodore's vice president of customer satisfaction, said, "At that time we didn't

have the greatest reputation for customer service and satisfaction. But this was FedEx's specialty, handling more

than 300,000 calls for service each day. Commodore arranged for FedEx to handle the entire telephone customer

service operation from FedEx's hub in Memphis.

After losing $29 million online the previous year, Borders Group turned to rival Amazon.com to manage its online

sales. Borders get to maintain an Internet sales channel and gains the operational effectiveness provided by

Amazon.com while being able to focus on the growth of its bricks and mortar business.

In this situation, business unit managers control both the amount and the efficiency of the central services. Under

these conditions, these central groups are profit centers. Their transfer prices should be based on the same

considerations as those governing other transfer prices.

(Numerical) MCS – 2004

Division B of Shayana company contracted to buy from Div. A, 20,000 units of a components which goes into the

final product made by Div. B. The transfer price for this internal transaction was set at Rs. 120 per unit by mutual

agreement. This comprises of (per unit) Direct and Variable labour cost of Rs. 20; Material Cost of Rs.60; Fixed

overheads of Rs.20 (lumpsum Rs.4 lacs) and Rs.20 lacs that Div. A would require for this additional activity. During

the year, actual off take of Div. B from Div. A was 19,600 units. Div. A was able to reduce material consumption by

5% but its budgeted investment overshot by 10%.

a) As Financial controller of Div. A, compare Actual Vs Budgetred Performance

b) Its implications for Management Control?

14

Page 15: MCS theory

For 20,000 Units For 19,600 Units

Solution:

Particulars Budgeted

(Rs. Per

Unit)

Budgeted

(Total in Rs.)

Actual

(Rs. Per Unit)

Actual

(Total in Rs.)

Direct and

Variable Labour

Cost

20 4,00,000 20 3,92,000

Material Cost 60 12,00,000 57 11,17,200

Fixed Overheads 20 4,00,000 4,00,000

Total Cost 100 20,00,000 19,09,200

Transfer Price 120 24,00,000 119.86 23,49,200

Profit 20 4,00,000 4,40,000

Investment 20 20,00,000 22,00,000

ROI =

Profit/Investment

20% 20%

Despite of increase in investment by 10%, there is negligible difference in transfer price. Also the sales have

decreased by 400 units. Therefore we can say that additional investment has not achieved any positive results.

SET-4

Q.1) A)Explain the concept of ROI. What are its advantages?

Return on investment (ROI) is the ratio of profit before tax to the gross investment.

ROI is calculated with the help of the following formula:

ROI = (Pre-Tax Profit/Sales) X (Sales/Net Assets) or (Pre-Tax Profits/Net Assets)

The numerator is profit before tax as reported in the P&L account. The profit should include only the profits arising out of the normal activities of the division. Unusual items of receipts and expenses should be excluded from the profit figure. One should also ignore windfalls and income from investments not related to the operations of the division. Tax is excluded from the numerator because the marginal of the SBU is not responsible for or in control of the tax paid.

Capital employed can be ascertained from the balance sheet by including fixed and current assets. Assets not currently put to divisional use should be excluded from the investment base. One also needs to exclude their relative earnings if any. The company should also exclude intangible assets like goodwill, deferred revenue expenses, preliminary expenses, etc.

ROI can be improved by:

o Increasing the profit margin on sales.

o Increasing the capital turnover

o Increasing both profit margin and capital turnover.

o Reducing cost as that adds to the total earnings of the firm.

o Increasing the profits by expanding present operations or developing new product line, increasing market share, etc.

o Diversifying, introducing productivity imporevement measures, expansion, replacement of old equipments

Advantages of ROI

o ROI relates return to the level of investment and not sales as the rate of return is more realistic.

o ROI can be decomposed into other variables as shown. These variables have tremendous analytical value.

o ROI is an effective tool for inter-firm comparison.

15

Page 16: MCS theory

Question 1 (b):

Many experts regard EVA as a concept superior to ROI and yet in certain cases, EVA does not do justice to the evaluation of investment center. Explain this phenomenon with as illustration.

EVA does not solve all the problems of measuring profitability in an investment center. In particular, it does not solve the

problem of accounting for fixed assets discussed above unless annuity depreciation is also used, and this is rarely done in

practice. If gross book value is used, a business unit can increase its EVA by taking actions contrary to the interests of the

company, as shown in Exhibit 7-3. If net book value is used, EVA will increase simply due to the passage of time. Furthermore,

EVA will be temporarily depressed by new investments because of the high net book value in the early years. EVA does solve

the problem created by differing profit potentials. All business units, regardless of profitability, will be motivated to increase

investments if the rate of return from a potential investment exceeds the required rate prescribed by the measurement

system.

Moreover, some assets may be undervalued when they are capitalized, and others when they are expensed. Although the

purchase cost of fixed assets is ordinarily capitalized, a substantial amount of investment in start-up costs, new product

development, dealer organization, and so forth may be written off as expenses, and, therefore, not appear in the investment

base. This situation applies especially in marketing units. In these units the investment amount may be limited to inventories,

receivables, and office furniture and equipment. When a group of units with varying degrees of marketing responsibility are

ranked, the unit with the relatively larger marketing operations will tend to have the highest EVA.

In view of all these problems, some companies have decided to exclude fixed assets from the investment base. These

companies make an interest charge for controllable assets only, and they control fixed assets by separate devices. Con-

trollable assets are, essentially, receivables and inventory.

Business unit management can make day-to-day decisions that affect the level of these assets. If these decisions are wrong,

serious consequences can occur-quickly. For example, if inventories are too high, unnecessary capital is tied up, and the risk

of obsolescence is increased; whereas, if inventories are too low, production interruptions or lost customer business can result

from the stockouts. To focus attention on these important controllable items, some companies, such as Quaker Oats, 17

include a capital charge for the items as an element of cost in the business unit income statement. This acts both to motivate

business unit management properly and also to measure the real cost of resources committed to these items.

Investments in fixed assets are controlled by the capital budgeting process before the fact and by post completion audits to

determine whether the anticipated cash flows, in fact, materialized. This is far from being completely satisfactory because

actual savings or revenues from a fixed asset acquisition may not be identifiable. For example, if a new machine produces a

variety of products, the cost accounting system usually will not identify the savings attributable to each product.

The argument for evaluating profits and capital investments separately is that this often is consistent with what senior

management wants the business unit manager to accomplish; namely, to obtain the maximum long-run cash flow from the

capital investments the business unit manager controls and to add capital investments only when they will provide a net

return in excess of the company's cost of funding that investment. Investment decisions, then, are controlled at the point

where these decisions are made. Consequently, the capital investment analysis procedure is of primary importance in

investment control. Once the investment has been made, it is largely a sunk cost and should not influence future decisions.

Nevertheless, management wants to know when capital investment decisions have been made incorrectly, not only because

some action may be appropriate with respect to the person responsible for the mistakes but also because safeguards to

prevent a recurrence may be appropriate.

Q.2 What are the different methods to evaluate the performance of an investment centre? Discuss the merits and demerits of each? Which method would you recommend?

The following techniques are useful in evaluating the performance of an investment centre:

1. Return on investment (ROI):

The rate of return on investment is determined by dividing net profit or income by the capital employed or investment made to achieve that profit.

ROI = Profit / Invested capital * 100

16

Page 17: MCS theory

ROI consists of two components viz.

Profit margin

Investment turnover

ROI = Net profit / Investment

= (Net profit / Sales) * (Sales / Investment in assets)

It will be seen from the above formula that ROI can be improved by increasing one or both of its components viz. the profit margin and the investment turnover in any of the following ways:

Increasing the profit margin

Increasing the investment turnover

Increasing both profit margin and investment turnover

Capital employed is taken to be the total of shareholders funds, loans etc

The profit figure used is in calculating ROI is usually taken from the profit and loss account, profit arising out of the normal activities of the company should only be taken.

Capital employed for the company as a whole can be arrived at as follows:

Share capital of the company xxx

Reserves and surplus xxx

Loans (secured/unsecured) xxx

------

xxx

Less: a. Investment outside the business xxx

b. Preliminary expenses xxx

c. Debit balance of P & L A/c xxx xxx

-------

xxxx

Merits:

Return on investment analysis provides a strong incentive for optimum utilization of the assets of the company. This encourages managers to obtain assets that will provide a satisfactory return on investment and to dispose off assets that are not providing an acceptable return. In selecting amongst alternative long-term investment proposals, ROI provides a suitable measure for assessment of profitability of each proposal.

Demerits:

ROI analysis is not very suitable for short-term projects and performances. In the initial stages a new investment may yield a small ROI which may mislead the management. Most likely the rate would improve in course of time when the initial difficulties are overcome.

The book value of assets decline due to depreciation, the investment base will continuously decrease in value, causing the rate of return to increase.

2. Residual income:

Residual income can be defined as the operating profit (or income) of the company less the imputed interest on the assets used by the company. In other words, interest on the capital invested in the company is treated as a cost and any surplus is the residual income. Residual income is profit minus notional interest charge on capital employed.

17

Page 18: MCS theory

Residual income is affected by the size of the organization and therefore will not provide a basis for evaluation of organizational performance. This is probably the main reason why the management continues to make use of ROI which is relative measure.

Not all projects start off with positive or sufficiently large positive profits in the early years of a project to produce a positive increment to residual income.

It has been argued that a more suitable measure of performance for investment centres, which could encourage managers to be more willing to undertake marginally profitable projects, is residual income.

We recommend RI as a method of evaluating performance of an investment centre. Because when RI is adopted for evaluation purposes, emphasis is placed on marginal profit amount above the cost of capital rather than on the rate itself.

Q.3 What are the objectives of Transfer Pricing?

Transfer price if designed appropriately has the following objectives:

It should provide each segment with the relevant information required to determine the optimum trade-off between company costs and revenues.It should induce goal congruent decisions-i.e. the system should be so designed that decisions that improve business unit profits will also improve company profits. It should help measure the economic performance of the individual profit centers. The system should be simple to understand and easy to administer.

What is ideal transfer price in the situations of Limited Market Shortage of Capacity in the industry

The ideal transfer price in the situations of Limited Market

By limited market it means that the markets for buying and selling profit centers may be limited.

Even in case of limited market the transfer price that is ideal or satisfies the requirement of a profit center system is the competitive price. In case if a company is not buying or selling its product in an outside market there are some ways to find the competitive price. They are as follows:

If published market prices are available, they can be used to establish transfer prices. However, these should be prices actually paid in the market-place and the conditions that exist in the outside market should be consistent with those existing within the company.

For example, market prices that are applicable to relatively small purchases are not valid in this case.

Market prices are set by bids. This generally can be done only if the low bidder has a reasonable chance of obtaining the business. One company accomplishes this by buying about one-half of a particular group of products outside the company and one-half inside the company. The company then puts all of the products out to bid, but selects one-half to stay inside. The company obtains valid bids, because low bidders can expect to get some of the business. By contrast, if a company requests bids solely to obtain a competitive price and does not award the contracts to the low bidder, it will soon find that either no one bids or that the bids are of questionable value.

If the production profit center sells similar products in outside markets, it is often possible to replicate a competitive price on the basis of the outside price. If the buying profit center purchases similar products from the outside market, it may be possible to replicate competitive prices for its proprietary products. This can be done by calculating the cost of the difference in design and other conditions of sale between the competitive products and the proprietary products.

Shortage of Capacity in the industry

In this case, the output of the buying profit center is constrained and again company profits may not be optimum. Some companies allow either buying profit center to appeal a sourcing decision to a central person or committee. In this scenario a buying profit center could appeal a selling profit center’s decision to sell outside.

The person/group would then make a sourcing decision on the basis of the company’s best interests. In every case the transfer price would be the competitive price. In other words, the profit center is appealing only the sourcing decision.

18

Page 19: MCS theory

Even if there are constraints on sourcing, the market price is the best transfer price. If the market price can be approximated, it is ideal transfer price.

When do you use Cost Based Transfer Pricing?

We use cost-based transfer pricing if there is no way of approximating valid competitive price. Transfer prices may be set up on the basis of cost plus a profit, even though such transfer prices may be complex to calculate and the results less satisfactory than a market-based price.

Two aspects need to be considered for cost-based transfer pricing:

The cost basis: The usual basis is the standard cost. Actual costs should not be used because production inefficiencies will then be passed on to the buying profit center. If the standard costs are used, there is a need to provide an incentive to set tight standards and to improve standards.

The profit markup: In calculating the profit markup, there also are two decisions:

What is the profit markup to be based?

The simplest and most widely used base is percentage of costs. If this base is used, however, no account is taken of capital required. A conceptually better base is a percentage of investment. But there may be a major practical problem in calculating the investment applicable to a given product. If the historical cost of the fixed assets is used, new facilities designed to reduce prices could actually increase costs because old assets are undervalued

What is the level of profit allowed?

The second problem with the profit allowance is the amount of the profit. The conceptual solution is to base the profit allowance on the investment required to meet the volume needed by the buying profit centers. The investment would be calculated at a “standard” level, with fixed assets and inventories at current replacement costs. This solution is complicated and, therefore, rarely used in practice.

Q.4 (a) “Transfer Pricing is not an accounting tool” comment with an illustration

If a group has subsidiaries that operate in different countries with different tax rates, manipulating the transfer prices between the subsidiaries can scale down the overall tax bill of the group. For example the tax rate in Country A is 20% and is 50% in Country B. In the larger interest of the group, it would be advisable to show lower profits in Country B and higher profits in Country A. For this, the group can adjust the transfer price in such a way that the profits in Country A increase and that in Country B get reduced. For this the group should fix a very high transfer price if the Division in Country A provides goods to the Division in Country B. This will maximize the profits in Country A and minimize the profits in Country B. The reverse will be true if the Division in Country A acquires goods from the Division in Country B.

There is also a temptation to set up marketing subsidiaries in countries with low tax rates and transfer products to them at a relatively low transfer price.

Transfer price is viewed as a major international tax issue. While companies indulge in all types of activities to lower their tax liability, the tax authorities monitor transfer prices closely in an attempt to collect the full amount of tax due. For this they enter into agreements whereby tax is paid on specific transactions in one country only. But if companies set unrealistic transfer price to minimize their tax liabilities and the same is spotted by the tax authority, then the company is forced to pay tax in both countries leading to double taxation.

There have been instances where companies have fixed unrealistic transfer prices. The first case relates to Hoffman La Roche that imported two drugs Librium and Valium into UK at prices of 437 pounds and 979 pounds per kilo respectively. While the tax authorities in UK accepted the price, the Monopolies Commission did not accept the company's argument, since the same drugs were available from an Italian firm for 9 pounds and 28 pounds per kilo.

The company's lawyers argued the case before the Commission on two grounds viz.

1. The price was not set on cost but on what the market would bear and

2. The company had incurred an R&D cost that was included in the price.

These arguments did not go well with the Commission and the company was fined 1.85 million pounds for the manipulative practices adopted while fixing the transfer price.

The second case is of Nissan. The company had falsely inflated freight charges by 40-60% to reduce the profits. The manipulation helped the company to hide tax to the tune of 237 million dollars. The next year Nissan was made to pay 106 million dollars in unpaid tax in the USA because the authorities felt that part of their US marketing profits were being

19

Page 20: MCS theory

transferred to Japan, as transfer prices on import of cars and trucks were too high. Interestingly the Japanese tax authorities took a different view and returned the double tax.

With a view to avoid such cases from recurring, Organisation for Economic Cooperation and Development issued some guidelines in 1995. These guidelines aim at encouraging world trade. They evolved what came to be known as the arm's length price. The principle states that the transfer price would be arrived at on the basis as if the two . companies are independent and unrelated. The price is determined through:

Comparable Price Method where the price is fixed on the basis of prices of similar products or an approximation to one.Gross Margin Method where a gross margin is established and applied to the seller's manufacturing cost.

In spite of all these efforts, it has to be admitted that setting a fair transfer price is not easy. So the onus of proving the price has been put on the taxpayer who is required to produce supporting documents. If the taxpayer fails to do this he is required to pay heavy penalty. For example, in USA, failure to provide documentary evidence results in a 40% penalty on the arm's length price. In UK the penalty is to the tune of 100% of any tax adjustment. Other countries are also in the process of evolving tight norms for the same.Countries across the globe also allow the taxpayer to enter into an Advance Pricing Agreement whereby dispute can be avoided and so also the costly penalty of double taxation and penalty.

Q.4.( b) Market Price is ideal transfer price even in limited markets. Comments

By limited market it means that the markets for buying and selling profit centers may be limited.

Even in case of limited market the transfer price that is ideal or satisfies the requirement of a profit center system is the competitive price. In case if a company is not buying or selling its product in an outside market there are some ways to find the competitive price. They are as follows:

1. If published market prices are available, they can be used to establish transfer prices. However, these should be prices actually paid in the market-place and the conditions that exist in the outside market should be consistent with those existing within the company. For example, market prices that are applicable to relatively small purchases are not valid in this case.

2.Market prices are set by bids. This generally can be done only if the low bidder has a reasonable chance of obtaining the business. One company accomplishes this by buying about one-half of a particular group of products outside the company and one-half inside the company.

The company then puts all of the products out to bid, but selects one-half to stay inside. The company obtains valid bids, because low bidders can expect to get some of the business. By contrast, if a company requests bids solely to obtain a competitive price and does not award the contracts to the low bidder, it will soon find that either no one bids or that the bids are of questionable value.

3.If the production profit center sells similar products in outside markets, it is often possible to replicate a competitive price on the basis of the outside price.

4.If the buying profit center purchases similar products from the outside market, it may be possible to replicate competitive prices for its proprietary products. This can be done by calculating the cost of the difference in design and other conditions of sale between the competitive products and the proprietary products.So we see from the above arguments that market price is ideal transfer price even in limited markets

20

Page 21: MCS theory

SET .5

Q.1) Describe and illustrate significance of human behavior patterns in management control system.

Ans. Management control systems influence human behavior. Good management control systems influence behavior in a goal

congruent manner; that is, they ensure that individual actions taken to achieve personal goals also help to achieve the

organization's goals. The concept of goal congruence, describing how it is affected both by informal actions and by formal

systems.

Senior management wants the organization to attain the organization's goals. But the individual members of the organization

have their own personal goals, and they are not necessarily consistent with those of the organization. The central purpose of a

management control system, then, is to ensure a high level of what is called "goal congruence." In a goal congruent process,

the actions people are led to take in accordance with their perceived self interest are also in the best interest of the

organization.

The significance of human behavior patterns in management control system can be explained with the help of Informal

Factors that influence Goal Congruence. In the informal forces both internal and external factors play a key role.

External Factors

External factors are norms of desirable behavior that exist in the society of which the organization is a part. These norms

include a set of attitudes, often collectively referred to as the work ethic, which is manifested in employees' loyalty to the

organization, their diligence, their spirit, and their pride in doing a good job (rather than just putting in time). Some of these

attitudes are local that is, specific to the city or region in which the organization does its work. In encouraging companies to

locate in their city or state, chambers of commerce and other promotional organizations often claim that their locality has a

loyal, diligent workforce. Other attitudes and norms are industry-specific. Still others are national; some countries, such as

Japan and Singapore, have a reputation for excellent work ethics.

Internal Factors

Culture

The most important internal factor is the organization's own culture-the common beliefs, shared values, norms of behavior and

assumptions that are implicitly and explicitly manifested throughout the organization. Cultural norms are extremely important

since they explain why two organizations with identical formal management control systems, may vary in terms of actual

control. A company's culture usually exists unchanged for many years. Certain practices become rituals, carried on almost

automatically because "this is the way things are done here." Others are taboo ("we just don't do that here"), although no one

may remember why. Organizational culture is also influenced strongly by the personality and policies of the CEO, and by those

of lower-level managers with respect to the areas they control. If the organization is unionized, the rules and norms accepted

by the union also have a major influence on the organization's culture. Attempts to change practices almost always meet with

resistance, and the larger and more mature the organization, the greater the resistance is.

21

Page 22: MCS theory

Management Style

The internal factor that probably has the strongest impact on management control is management style. Usually,

subordinates' attitudes reflect what they perceive their superiors' attitudes to be, and their superiors' attitudes ultimately

stem from the CEO.

Managers come in all shapes and sizes. Some are charismatic and outgoing; others are less ebullient. Some spend much time

looking and talking to people (management by walking around); others rely more heavily on written reports.The Informal

Organization

The lines on an organization chart depict the formal relationships-that is, the official authority and responsibilities-of each

manager. The chart may show, for example, that the production manager of Division A reports to the general manager of

Division A. But in the course of fulfilling his or her responsibilities, the production manager of Division A actually

communicates with many other people in the organization, as well as with other managers, support units, the headquarters

staff, and people who are simply friends and acquaintances. In extreme situations, the production manager, with all these

other communication sources available, may not pay adequate attention to messages received from the general manager;

this is especially likely to occur when the production manager is evaluated on production efficiency rather than on overall

performance. The realities of the management control process cannot be understood without recognizing the importance of

the relationships that constitute the informal organization.

Perception and Communication

In working toward the goals of the organization, operating managers must know what these goals are and what actions they

are supposed to take in order to achieve them. They receive this information through various channels, both formal (e.g.,

budgets and other official documents) and informal (e.g., conversations). Despite this range of channels, it is not always clear

what senior management wants done. An organization is a complicated entity, and the actions that should be taken by anyone

part to further the common goals cannot be stated with absolute clarity even in the best of circumstances.

Moreover, the messages received from different sources may conflict with one another, or be subject to differing

interpretations. For example, the budget mechanism may convey the impression that managers are supposed to aim for the

highest profits possible in a given year, whereas senior management does not actually want them to skimp on maintenance or

employee training since such actions, although increasing current profits, might reduce future profitability. The informal

factors discussed above have a major influence on the effectiveness of an organization’s management control. The other

major influence is the formal systems. These systems can be classified into two types: (1) the management control system

itself and (2) rules, which are described in this section.

The Formal Control System

Rules

We use the word rules as shorthand for all types of formal instructions and controls, including: standing instructions, job

descriptions, standard operating procedures, manuals, and ethical guidelines. Rules range from the most trivial (e.g., paper

clips will be issued only on the basis of a signed requisition) to the most important):e.g., capital expenditures of over $5

million must be approved by the board' of directors).

Some rules are guides; that is, organization members are permitted, and indeed expected, to depart from them, either under

specified circumstances or when their own best judgment indicates that a departure would be in the best interests of the

organization.

Some rules are positive requirements that certain actions be taken (e.g., fire drills at prescribed intervals). Others are

prohibitions against unethical, illegal, or other undesirable actions. Finally, there are rules that should never be broken under

any circumstances: a rule prohibiting the payment of bribes, for example, or a rule that airline pilots must never take off

without permission from the air traffic controller.

Some specific types of rules are listed below:

Physical Controls

Security guards, locked storerooms, vaults, computer passwords, television surveillance, and other physical controls may

be part of the control structure.

Manuals

Much judgment is involved in deciding which rules should be written into a manual, which should be considered to be

guidelines rather than fiats, how much discretion should be allowed, and a host of other considerations. Manuals in

bureaucratic organizations are more detailed than are those in other organizations; large organizations have more

manuals and rules than small ones; centralized organizations have more than decentralized ones; and organizations with

22

Page 23: MCS theory

geographically dispersed units performing similar functions (such as fast-food restaurant chains) have more than do

single-site organizations

System Safeguards

Various safeguards are built into the information processing system to ensure that the information flowing through the

system is accurate, and to prevent (or at least minimize) fraud of every sort. These include: cross-checking totals with

details, requiring signatures and other evidence that a transaction has been authorized, separating duties, counting cash

and other portable assets frequently, and a number of other procedures described in texts on auditing.

Task Control Systems

Task control is the process of assuring that specific tasks are carried out efficiently and effectively. Many of these tasks

are controlled by rules. If a task is automated, the automated system itself provides the control.

Q.2) Write short notes on

a. Concept of profit centre in non-profit organization

b. Management control in matrix structures

c. Implications of differentiated strategies on controls.

Ans. a) Concept of profit centre in NPO

By law NPO are allowed to make profit but are restrained from distributing it to owners and management This way they are

non profit making organizations (from the owner's point of view). Such organizations include religious, charitable and

educational trusts. Prime goal of management control systems in such organization is enhancing the service spread first and if

possible then cost control rather and than operating efficiency. On the financial front, they enjoy many concessions from the

government such as taxes, subsidies, grants etc so also they attract special control from these assisting institutes.

Characteristics:

1. Absent of profit performance measure leads to problems in assessing the efficiency of the organization. If the

organization shows large net income it may be because that NPO may not be providing the services to the extent possible/

expected. If the organization shows net losses it may show the NPO facing risk of bankruptcy. Hence non availability of clear-

cut performance yardstick makes the problem of control worst.

2. NPO's have contributed capital Plant: NPOs do not have shareholder as its stakeholder. The capital contribution to the

business comes by way of contributions to assets such as building and equipments. Second kind of contribution could be in

the form of monetary assistance, which entitles the organization to reap the interest on it keeping the principal amount

intact.

3. Operating Assets represents the resources used for running day to day activities. And the contributed assets

are not allowed to mix up with the operating assets.

4. Fund accounting: NPO need to keep two types of financial statements one set for contributed capital and another for

operating capital. The nature of the contributed capital is beyond control of the management and therefore management

concentrates on controlling the operating assets/investments.

5. Governance: Usually NP

of the product/services - what could be appropriate price? Usually it is set at total/full cost. The more stress expected on

allocation of scare resources. Though not stricter control, but a sense of control can be built among the managers by way of

using budgets for various O are managed by trusts, who exercise less control on operational matters. Hence performance

control is less demanding from owners' point of view and difficult from the point of view of management.

activities and expenses. Non profit basis makes performance evaluation quite impossible. But one can make the things easier

by concentrating on adherence to costs budgets, and enhancing the service base.

b) Management control in matrix structures

Matrix organizational structure assigns multiple responsibilities to the functional heads. Evaluation of performance of such

organizational entities is very difficult. Though they offer economies of using scares functional staff, it poses problems of

casting the individual responsibility. This form of organization is very complex, from the point of view of management control

23

Page 24: MCS theory

system.

At the end we must not forget that the management control system is for the organization and not the organization exists for

management control system. One has to mold and remold the management control system to suit the given organization

structure

A citation by Anthony is worth noting in this regard.

Usually in an advertisement agency, account supervisors are shifted from one

account to another on periodic basis, this practice allows the agency to look at the account from the perspectives of different

executives. However taking in to consideration the time lag of result realization in such services is quite large.

And this may pose problem of performance assessment of a particular executive. This does not mean a control system

designer should insist on abandoning the rotation system of the executives.

c) Implications of differentiated strategies on controls

Different corporate strategies imply the following differences in the context in which control systems need to be designed: As

firms become more diversified, corporate-level managers may not have significant knowledge of, or experience in, the

activities of the company's various business units. If so, corporate-level managers for highly diversified firms cannot expect to

control the different businesses on the basis of intimate knowledge of their activities, and performance evaluation tends to be

carried out at arm's length. Single-industry and related diversified firms possess corporatewide core competencies (on which

the strategies of most of the business units are based. Communication channels and transfer of competencies across business

units, therefore, are critical in such firms. In contrast, there are low levels of interdependence among the business units of

unrelated diversified firms. This implies that as firms become more diversified, it may be desirable to change the bal ance in

control systems from an emphasis on fostering cooperation to an emphasis on encouraging entrepreneurial spirit.

Strategic planning: given the low level of interdependencies, conglomerates tend to use vertical strategic planning

systems-that is, business units prepare strategic plans & submit to senior management to review & approve. The

horizontal dimension might be incorporated into the strategic planning process in a number of different ways. First, a

group executive might be given the responsibility to develop a strategic plan for the group as a whole that explicitly

identifies synergies across individual business units within the group. Second, strategic plans of individual business units

could have an interdependence section, in which the general manager of the business unit identifies the focal linkages

with other business units and how those linkages will be exploited. Third, the corporate office could require joint strategic

plans for interdependent business units. Finally, strategic plans of individual business units could be circulated to

managers of similar business units to critique and review. These methods are not mutually exclusive. In fact, several of

them could be pursued fruit. fully at the same time.

Budgeting: The chief executives of single-industry firms may be able to control the operations of subordinates through

informal and personally oriented mechanisms, such as frequent personal interactions. This lessens the need to rely as

heavily on the budgeting system as the tool of control. On the other hand, in a conglomerate it is nearly impossible for the

chief executive to rely on informal interpersonal interactions as a control tool; much of the communication and control has

to be achieved through the formal budgeting stem. This implies the following budgeting system characteristics in a

conglomerate. Business unit managers have somewhat greater influence in developing their budgets since they, not the

corporate office, possess most of the information about their respective product/market environments. Greater emphasis

is often placed on meeting the budget since the chief executive has no other informal controls available.

Transfer Pricing: Transfers of goods and services between business units are more frequent in single-industry and

related diversified firms than in conglomerates. The usual transfer pricing policy in a conglomerate is to give sourcing

flexibility to business units and use arm's-length market prices. However, in a single-industry or a related diversified firm,

synergies may be important, and business units may not be given the freedom to make sourcing decisions.

Incentive Compensation: The incentive compensation policy tends to differ across corporate strategies in the following

ways-

Use of formulas: Conglomerates, in general, are more likely to use formulas to determine business unit managers'

bonuses; that is, they may base a larger portion of the bonus on quantitative, financial measures, such as X percent

bonus on actual economic value added (EVA) in excess of budgeted EVA. These formula-based bonus plans are employed

because senior management typically is not familiar with what goes on in a variety of disparate businesses. Senior

managers of single-industry and related diversified firms tend to base a larger fraction of the business unit managers’

bonus on subjective factors. In many related diversified firms, greater degrees of interrelationships imply that one unit's

performance can be affected by the decisions and actions of other units. Therefore, for companies with highly

24

Page 25: MCS theory

interdependent business units, formula-based plans that are tied strictly to financial performance criteria could be

counterproductive.

Profitability measures: In the case of unrelated diversified firms, the incentive bonus of the 'business unit managers

tend to be determined primarily by the profitabi1ity of that unit, rather than the profitability of the firm~ Its purpose is to

motivate managers to act as though the business unit were their own company.

In contrast, single-industry and related diversified firms tend to base the incentive bonus of a business unit manager on

both the performance of that unit and the performance of a larger organizational unit (such as the product group to which

the business unit belongs or perhaps even .the overall corporation). When business units are interdependent, the more

the incentive bonus of general managers emphasizes the separate performance of each unit, the greater the possibility of

interunit conflict. On the other hand, basing the bonus of general managers more on the overall corporate performance is

likely to encourage greater interunit cooperation, thereby increasing managers' motivation to exploit interdependencies

rather than their individual results.

Business Unit Strategy: Diversified corporations segment themselves into business units and typically assign different

strategies to the individual business units. Many chief executive officers of multi business organizations do not adopt a

standardized, uniform approach to controlling their business units; instead, they tailor the approach to each business

unit's strategy.The strategy of a business unit depends on two interrelated aspects: (1) Its mission ("What are its overall

objectives?") and (2) its competitive advantage. ("How should the business unit compete in its industry to accomplish its

mission?"). Typically business units choose from four missions: build, hold, harvest, and divest. The business unit has two

generic ways to compete and develop a sustainable competitive advantage: low cost and differentiation.

Mission The mission for existing business units could be either build, hold, or harvest. These missions constitute a

continuum, with "pure build" at one end and "pure harvest" at the other end. To implement the strategy effectively, there

should be congruence between the mission chosen and the types of controls used. The mission of the business unit

influences the uncertainties that general managers face and the short-term versus long-term trade-offs they make.

Management control systems can be systematically varied to help motivate the man ager to cope effectively with

uncertainty and make appropriate short-term versus long term trade-offs. Thus, different missions often require

systematically different management control systems.

Mission and Uncertainty "Build" units tend to face greater environmental uncertainty than "harvest" units for several

reasons: Build strategies typically are undertaken in the growth stage of the product life cycle, whereas harvest strategies

typically are undertaken in the mature decline stage of the product life cycle. Such factors as manufacturing process;

product technology; market demand; relations with suppliers, buyers, and distribution channels; number of competitors;

and competitive structure change more rapidly and are more unpredictable in the growth stage than in the

mature/decline stage. An objective of a build business unit is to increase market share. Because the total market share of

all firms in an industry is 100 percent, the .battle for market share is a zero-sum game; thus, a build strategy puts a

business unit in greater conflict with its competitors than does a harvest strategy. Competitors' actions are likely to be

unpredictable, and this contributes to the uncertainty that build business units face. On both the input side and the output

side, build managers tend to experience greater dependencies on external individuals and organizations than do harvest

managers. For instance, a build mission signifies additional capital investment (greater dependence on capital markets),

expansion of capacity (greater dependence on the technological environment), increase in market share (greater

dependence suppliers and labor markets), and so on. The greater the external dependencies a business unit faces, the

greater the uncertainty it confronts.Build business units are often in new and evolving industries; thus, build managers

are likely to have less experience in their industries. This also contributes to the greater uncertainty that managers of

build units face in dealing with external constituencies.

Mission and Time Span The choice of build versus harvest strategies has implications for short-term versus long-term

profit trade-offs. The share-building strategy includes (a) price 'cutting, (b) major R&D expenditures (to introduce new

products), and (c) major market development expenditures. These actions are aimed at establishing market leadership,

but they depress short-term profits. Thus, many decisions that a build unit manager makes, today may not result in

profits until some future period. A harvest strategy, on the other hand, concentrates on maximizing short-term profits.

Strategic Planning When the environment is uncertain, the strategic planning process is especially important

management needs to think about how to cope with the uncertainties, and this usually requll1 longer-range view of

planning than is possible in the annual budget. If the environment is stable, there may be no strategic planning process at

25

Page 26: MCS theory

all or only a broad-brush strategic plan. Thus, the strategic planning process is more critical and more important for build,

as compared with harvest, business units.

Nevertheless, some strategic planning of the harvest business units may be necessary because the company's overall

strategic plan must encompass all of its businesses to effectively balance cash flows.

In screening capital investments and allocating resources, the system may be more quantitative and financial for harvest

units. A harvest business unit operates in a mature industry and does not offer tremendous new investment possibilities.

Hence, the required earnings rate for such a business unit may be relatively high to motivate the manager to search for

project with truly exceptional returns. Because harvest units tend to experience stable environments with predictable

products, technologies, competitors, and customers), discounted cash flow PCF) analysis often can be used more confidently.

The required information used to evaluate investments from harvest units is primarily financial. A build unit, however, is

positioned on the growth stage of the product life cycle. Since the corporate office wants to take advantage of the

opportunities in a growing market, senior management may set a relatively low discount rate, thereby motivating build

managers to forward more investment ideas to corporate office. Given the product/market uncertainties, financial analysis of

some projects from build units may be unreliable. For such projects, nonfinancial data are more important.

Budgeting The calculational aspects of variance analysis comparing actual results with the budget identify variances as

either favorable or unfavorable. However, a favorable variance does not necessarily imply favorable performance, nor

does an unfavorable variance imply unfavorable performance. The link between a favorable or unfavorable variance, on

the one hand, and favorable or unfavorable performance, on the other hand, depends on the strategic context of the

business unit under evaluation.

Incentive Compensation Syste In designing an incentive compensation package for business unit managers, the

following questions need to be resolved:

1. 1. What should the size of incentive bonus payments be relative to the general manager's base salary? Should the

incentive bonus payments have upper limits?

2. What measures of performance (e.g., profit, EVA, sales volume, market share, product

development) should be used when deciding the general manager's incentive bonus awards? If multiple performance

measures are employed, how should they be weighted?

3. How much reliance should be placed on subjective judgments in deciding on the bonus amount?

4. How frequently (semiannual, annual, biennial, etc.) should incentive awards be made?

With respect to the first question, many firms use the principle that the riskier the strategy, the greater the proportion of the

general manager's compensation in bonus compared to salary (the "risk/return" principle). They maintain that because

managers in charge of more uncertain task situations should be willing.to take greater risks, they should have a higher per-

centage of their remuneration in the form of an incentive bonus. Thus, "build" managers are more likely than "harvest"

managers to rely on bonuses.

As to the second question, when rewards are tied to certain performance criteria, behaviour ls influenced by the desire to

optimize performance with respect to those criteria. Some performance criteria (cost control, operating profits, and cash flow

from operations) focus more on short-term results, whereas other performance criteria (market share, new product develop-

ment, market development, and people development) focus on long-term profitability. Thus,

linking incentive bonus to short-term criteria tends to promote a short-term focus on the part of the general manager and,

similarly, linking incentive bonus to long-term criteria is likely to promote long-term focus. Considering the relative differences

in time horizons of build and harvest managers, it may not be appropriate to use a single, uniform financial criterion, such as

operating profits, to evaluate the performance of every business unit. A better idea would be louse multiple performance

criteria, with differential weights for each criterion depending on the business unit's mission.

The third question asks how much subjective judgment should affect bonus amounts. At one extreme, a manager's bonus

might be a strict formula-based plan, with the bonus tied to performance on quantifiable criteria (e.g., X percent bonus on

actual profits in excess of budgeted profits). At the other extreme, a manager's incentive bonus amounts might be based

solely on the superior's subjective judgment or discretion. Alternatively, incentive bonus amounts might also be based on a

combination of formula-based and subjective approaches. Performance on most long-term criteria (market development, new-

product development, and people development) is harder to measure objectively than is performance along most short-run

criteria (operating profits, cash flow from operations, and return on investment).As already noted, build managers- in contrast

with harvest managers, should concentrate more on the long run, so they typically are evaluated more subjectively than are

26

Page 27: MCS theory

harvest managers.

As to the final question, the frequency of bonus awards does influence the time horizon of managers. More frequent bonus

awards encourage managers to concentrate on short-term performance since they have the effect of motivating managers to

focus on those facets of the business they can affect in the short run.

Competitive Advantage A business unit can choose to compete. Either as a differentiated player or as a

low-cost player, Choosing a differentiation 'approach, rather than a low-cost approach, increases uncertainty of a business

unit's task environment for three reasons.

1. Product innovation is more critical for differentiation business units than for low cost business units. This is partly

because a low-cost business unit, with primary emphasis on cost reduction, typically prefers to keep its product

offerings stable over time; a differentiation business unit, with its primary focus on uniqueness & exclusivity, is likely

to engage in greater product innovation.

2. A low cost business unit typically tend to have narrow product lines to minimize the inventory carry costs as well as

to benefit from scale economies. Differentiation business units on the other hand tend to have a broader set of

products to create uniqueness.

3. Low cost business units typically produce no-frill commodity products& these products succeed primarily because

they have lower prices than competing products. However product differentiation business units succeed if customers

perceive that the products have advantages over competing products. Since the customer perception is difficult to

learn about, & since customer loyalty is subject to change resulting from actions of competitors or other reasons, the

demand for differentiated products is typically more difficult to predict than the demand for commodities.

Q.3) Which management control practices, if followed, in performance measurement of investment centres are

likely to induce goal congruence, in respect of following assets

d. (i) Idle (ii) Intangible (iii) Leased

e. (i) Cash (ii) Receivables (iii) Inventories

Ans. In some business units, the focus is on profit as measured by the difference between revenues and expenses. In other

business units, profit is compared with the assets employed in earning it. We refer to the latter group of responsibility centers

as investment centers.

Measuring Assets Employed

In deciding what investment base to use to evaluate investment center managers, headquarters asks two questions: First,

what practices will induce business unit managers to use their assets most efficiently and to acquire the proper amount and

kind of new assets? Presumably, when their profits are related to assets employed, business unit managers will try to improve

their performance as measured in this way. \Senior management wants the actions that they take toward this end to be in the

best interest of the whole corporation. Second, what practices best measure the performance of the unit as an economic

entity?

Cash

Most companies control cash centrally because central control permits use of a smaller cash balance than would be the case if

each business unit held the cash balances it needed to weather the unevenness of its cash inflows and outflows. Business unit

cash balances may well be only the "float" between daily receipts and daily disbursements. Consequently, the actual cash

balances at the business unit level tend to be much smaller than would be required if the business unit were an independent

company. Many companies therefore use a formula to calculate the cash to be included in the investment base. For example,

General Motors was reported to use 4.5 percent of annual sales; Du Pont was reported to use two months' costs of sales minus

depreciation.

One reason to include cash at a higher amount than the balance carried by a business unit is that the higher amount is

necessary to allow comparisons to outside companies. If only the actual cash were shown: by internal units would appear

27

Page 28: MCS theory

abnormally high and might mislead senior management.

Some companies omit cash from the investment base. These companies reason that the amount of cash approximates the

current liabilities; if this is so, the sum of accounts receivable and inventories will approximate the working capital.

Receivables: Business unit managers can influence the level of receivables, not only indirectly by their ability to

generate sales, and directly, by establishing credit terms by approving individual credit accounts and credit limits,

and by the collecting overdue amount. In the interest of simplicity, receivable included at the actual end-.of-period

balances, although the average of intraperiod balances is conceptually a better measure of the am should be related

to profits.

Whether to include accounts receivable at selling prices or at cost of goods sold is debatable. One could argue that the

business unit's real investment in accounts receivable is only the cost of goods sold and that a satisfactory return on this

investment is probably enough. On the other hand, it is possible to argue that the business unit could reinvest the money

collected from accounts receivable, and, therefore, accounts receivable should be included at selling prices. The usual

practice is to take the simpler alternative-that is, receivables at the book amount, which is the selling price less an

allowance for bad debts.

If the business unit does not control credits and collections, receivables may be calculated on a formula basis. This formula

should be consistent with the normal payment period-for example, 30 days' sales where payment is made 30 days after

the shipment of goods.

Inventories: Inventories ordinarily are treated in a manner similar to receivables –that is they are often recorded at end-

of-period amounts even though intraperiod averages would be preferable conceptually. If the company uses LIFO (last in

first out) for financial accounting purposes, a different valuation method usually is used for business unit profit reporting

because LIFO inventory balances tend to be unrealistically low in periods of inflation. In these circumstances, inventories

should be valued at standard or average costs, and these same costs should be used to measure cost of sales on the

business unit income statement

If work-in-process inventory is financed by advance payments or by progress payments from the customer, as is typically the

case with goods that require a long manufacturing period, these payments either are subtracted from the gross inventory

amounts or reported as liabilities.

For e.g. with manufacturing periods a year or greater, Boeing received progress payments for its airplanes and

recorded them as liabilities.

Some companies subtract accounts payable from inventory on the grounds that accounts payable represent financing of part

of the inventory by vendors, at zero cost to the business unit. The corporate capital required for inventories is only the

difference between the gross inventory amount and accounts payable. If the business unit can influence the payment period

allowed by vendors, then including accounts payable in the calculation encourages the manager to seek the most favorable

terms. In times of high interest rates or credit stringency, managers might be encouraged to consider forgoing the cash

discount to have, in effect, additional financing provided by vendors. On the other hand, delaying payments unduly to reduce

net current assets may not be in the company's best interest since this may hurt its credit rating.

Leased Assets

Suppose the business unit whose financial statements are shown in Exhibit 1 (see page 21) sold its fixed assets for their book

value of $300,000, returned the proceeds of the sale to corporate headquarters, and then leased back the assets at a rental

rate of $60,000 per year. As Exhibit 2 (see page 21) shows, the business unit's income before taxes would decrease because

the new rental expense would be higher than the depreciation charge that was eliminated. Nevertheless, economic valued

added would increase because the higher cost would be more than offset by the decrease in the capital charge. Because of

this, business unit managers are induced to lease, rather than own, assets whenever the in terest charge that is built into the

rental cost is less than the capital charge that is applied to the business unit's investment base. (Here, as elsewhere, this

generalization oversimplifies because, in the real world, the impact of income taxes must also be taken into account.)

Many leases are financing arrangements-that is, they provide an alternative way of getting to use assets that otherwise would

be acquired by funds obtained from debt and equity financing. Financial leases (i.e., long-term leases equivalent to the

present value of the stream of lease charges) are similar to debt and are so reported on the balance sheet. Financing

decisions usually are made by corporate headquarters. For these reasons, restrictions usually are placed on the business unit

manager's freedom to lease assets.

28

Page 29: MCS theory

Idle Assets

If a business unit has idle asset that can be used by other units, the business unit may be permitted to exclude them from the

investment base if it classifies them as available. The purpose of this permission is to encourage business unit managers to

release underutilized assets to units that may have better use for them. However, if the fixed assets cannot be used by other

units, permitting the business unit manager to remove them from the investment base could result in dysfunctional actions

For example; it could encourage the business unit manager to idle partially utilized assets that are not earning a return equal

to the business unit's profit objective. If there is no alternative use for the equipment, any contribution from this equipment

will improve company profits.

Intangible Assets

Some companies tend to be R&D intensive (e.g., pharmaceutical firms such as Novartis spend huge amounts on developing

new products); others tend to be marketing intensive (e.g., consumer products firms such as Unilever spend huge amounts on

advertising). There are advantages to capitalizing intangible assets such as R&D and marketing and then amortizing them

over a selected life. This method should change how the business unit manager views these expenditures. By accounting for

these assets as long-term investments, the business unit manager will gain less short-term benefit from reducing out lays on

such item, For instance, if R&D expenditures are expensed immediately, each dollar of R&D cut would be a dollar more in

pretax profits. On the other hand, if R&D costs are capitalized, each dollar cut will reduce the assets employed by a dollar; the

capital charge is thus reduced only by one dollar times the cost of capital, which has a much smaller positive impact on eco -

nomic valued added.

SET.6

Q.1) What do you understand by Goal Congruence? What are the informal factors that influence goal

congruence?

Ans: This term is used when the same goals are shared by top managers and their subordinates. This is one of the many

criteria used to judge the performance of an accounting system. The system can achieve its goal more effectively and perform

better when organizational goals can be well aligned with the personal and group goals of subordinates and superiors. The

goals of the company should be the same as the goals of the individual business segments. Corporate goals can be

communicated by budgets, organization charts, and job descriptions.

Goal Congruence- Meaning Individuals work in different hierarchies and handle different responsibilities &

may have different goals. But they must come together as far as Company’s Goal is concerned (there action must speak Co’s

language.)

Goal Congruence

Example 1– The HR manager has devised a HR training program to enhance the skills of its sales personnel, with an objective

to enhance their productivity But if company is in strategic need of attaining a certain sales volume in a given quarter, it can

not do so on account of non availability of personnel.

Example 2– The marketing department has planned an impressive advertising campaign, which promises good returns, But

say due to cash crunch Company’s current financial position may not let to lose the strings

Example 3 – Production Manager may get a good applause for reducing cycle time; But at what cost? Building up the high

inventory i.e. higher investment in current assets. While doing so he just overlooked the financial interest of the company. •

After completing the given activity in more efficient manner the concerned manager scores the point/s on his score card. •

Whether his actions are leading to scoring of points on the organization’s score card too? if it is so then only one can say the

organization is marching towards a common goal.

Every individual working in an organization has got his own motive to do the work. Individuals act in their own interest, based

on their own motivations. And it is always not necessarily consistent with the Co’s goal. In a goal congruence process, the

actions the people are led to take in accordance with their perceived self interest are also in the best interest of the

organization i.e. Goal congruence ensures that the action of manager taken in their best interest is also in the best interest of

the organization.

Informal factors that influence goal congruence:

Informal Factors –

External factors – set of attitudes of the society, work ethics of the society

Internal factors – (Factors within the organization)

Culture-Common beliefs, shared values, norms of behavior & assumptions

29

Page 30: MCS theory

Implicitly accepted and explicitly built into.

Mgt. Style – Informal/Formal

The Communication Channels

Perception and Communication – e.g. Budget (meaning) strict profit.

Organizations with Business Divisions (Profit Centre) format have observed that Divisional

Controllers experience divided loyalty in carrying out their functions, causing a possible dysfunction. How could

such a situation be resolved? Define role of controller which suits your suggestion.

To the extent the decision are decentralized top management may lose some control. Relying on control reports is not as

effective as personal knowledge of an operation. With profit center, top management must change its approach to control.

Instead of personal direction senior management must rely to a considerable extent on management control reports.

Competent units that were once cooperating as functional units may now compete with one another dis advantageously. An

increase in one manager’s profit may decrease those of another. This decrease in cooperation may manifest itself in a

manager unwillingness to refer sales lead to another business unit, even though that unit is better qualified to follow up on the

lead in production decision that have undesirable cost consequence on other units or in the hoarding of personnel or

equipment that from the overall company standpoint would be better off used in another units.

There may be too much emphasis on short run profitability at the expense of long run profitability. In the desire to report high

current profits, the profit center manager may skip on R&D, training, maintenance. This tendency is especially prevalent when

the turnover of profit center managers is relatively high. In these circumstances, manager may have good reason to believe

that their action may not affect profitability until after they have moved to other job.

There is no complete satisfactory system for ensuring that each profit center by optimizing its own profit , will optimize

company profits.

If headquarter management is more capable or has better information then the average profit center manager the quality of

some of the decision may be reduced.

Divisionalization may cause additional cost because it may require additional management staff personnel and recordkeeping

and may lead to redundant at each profit center.

Business units as profit centers:

Business units are usually set up at profit centers. Business unit managers tend to control product development,

manufacturing, and marketing resources. They are in a position to influence revenue and cost and as such can be held

accountable for the bottom line. However as pointed out in the next section a business unit manager authority may be

constrained such constrained should be incorporated in designing and operating profit center.

Constraint on business unit authority

To realize fully the advantage of the profit center concept the business unit manger would have to be as autonomous as the

president of the independent company. As a practical matter however such autonomy is not feasible. If a company were

divided into completely independent units the organization would be giving up the advantage of size and synergism. Also

senior management authority that a board of director gives to the chief executive. Consequently business unit structure

represents trade off between business unit autonomy and corporate constraint. The effectiveness of a business units

organization is largely dependent on how well these trade off are made.

The performance of a profit center is appraised by comparing actual results for one or more orf these measures with

budgeting amounts. In addition, data on competitors and the industry provide a good cross check on the appropriate of the

budget. Data for individual companies are available from the securities and exchange commission for about key business

ratios; standard & poor computer services, Inc; Robert Morris associates annual statement studies; and annual survey

published in fortune, business week, and Forbes. Trade associations publish data for the companies in their industries.

Revenues: choosing the appropriate revenue recognition method is important. Should revenue be recognized at the time as

order is received, at the time an order is shipped, or at the time cash is received?

In addition to that decision, issues related to common revenues may need to be considered. There are some situations in

which two or more profit centers participate in the sales effort that results in a sale; ideally, each should be given appropriate

credit for its part in this transaction. Many companies have not given much attention to the solution of these common revenue

problems. They take the position that the identification of price responsibility for revenue generation is too complicated to be

practical and that sale personnel must recognize they are working not only for their own profit center but also for the overall

good of the company. They for example, may credit the business unit that takes an order for a product handled by the

30

Page 31: MCS theory

another unit with the equivalent of a brokerage commission or a finder fee. In the case of a bank the branch performing a

service may be given explicit credit for that service even though the customer account is maintained in another branch.

Role of controller

It should publish procedure and forms for the preparation of the budget.

It should provide assistance to budgetees in the preparation of their budget.

It should administer the process of making budget revision during the year.

It should coordinate the work of budget departments in lower echelons

It should analyze reported performance against budget, interprets the result, and prepares summary report for senior

management.

Part of a multinational group, Sundaram Shoe Company(SSC), established its own facilities in India over 75

years ago and enjoyed an excellent record-high market share for its diverse range of shoes, growth and

profits. SC markets its products through company owned shops and its own personnel. Organization

structure is functional. Since 2001, profitability, market share are slipping. Pressure from cheap Chinese

shoes and also premium shoes like Nike has made the company think< of organizational restructuring and

introducing Comensurate Control System to regain its position. Although SSC outsources, 30% of products,

it is seen as a production oriented company. SSC wants to adopt measures to reduce costs, strengthen

marketing and be in a position to produce and meet unexpected and unusual customer demands. How

should the company reorganize to achieve Goal Congruence. Define Performance Metric?

In a goal congruent process, the actions people are led to take in accordance with their perceived self-interest are also in the

best interest of the organization. A firm’s strategy has a major influence on its structure. The type of structure in turn

influences the design of the organization’s management control systems. Sundaram Shoe Company’s (SSC) organization

structure is functional which involves the notion of a manager who brings specialized knowledge to bear on decisions related

to a specific function, vis-à-vis a general purpose manager who lacks the specialized knowledge. A skilled marketing and

production manager would be able to make better decisions in their respective fields. He would also be able to supervise

workers in the same function better than the generalist would. Thus an important advantage of the functional structure is

efficiency. A major disadvantage of this structure is that there is no unambiguous way of determining the effectiveness of the

separate functional managers because each function contributes jointly to the organization’s final output. Therefore, there is

no way of determining how much of the profit was earned respectively by the several production departments.

Sundaram Shoe Company which was a market leader for a period of over 75 years has been losing market share, which has

impacted its profitability. Also it needs to be seen that the company outsources about 30% of its products. The company aims

to strengthen marketing, reduce costs and wants to be in a position to customize products as per the demands of the

customer. Thus, Sundaram needs to re-organize its organization structure which is functional to a Business Unit form of

organization. The benefits of the re-organization would be that the business unit or the division would be responsible for all the

functions involved in producing and marketing a specified product line. The business managers act almost as if their units are

separate companies. They are responsible for planning and co-coordinating the work of the separate functions. Their

performance is measured by the profitability of the business unit. This is a valid criterion because profit reflects the activities

of both marketing and production.

Though business unit managers exercise broad authority over their units, headquarters reserves certain key prerogatives.

Headquarters are responsible for obtaining funds for the company as a whole and allocating it to the business unit, as well as

approving budgets and judging the performance of business unit managers, setting their compensation.

A major advantage of the Business unit structure of organization is that because it is close to the market for its products than

the headquarters, its manager may make sounder production and marketing decisions than headquarters might and the unit

as a whole reacts to new threats or opportunities quickly. This re-organization would help in achieving goal congruence in the

organization.

Performance Metrics are high-level measures what you are doing; that is, they assess your overall performance in the areas

you are measuring. They are external in nature and are most closely tied to outputs, customer requirements, and business

needs for the process.

The performance measurement system should cover the following areas at a minimum:

31

Page 32: MCS theory

CUSTOMERS

1. Performance against customer requirements

2. Customer Satisfaction

PERFORMANCE OF INTERNAL WORK PROCESSES

1. Cycle times

2. Product and service quality

3. Cost performance (could be productivity measures, inventory, etc.)

SUPPLIERS

1. Performance of suppliers against your requirements

FINANCIAL

1. Profitability (could be at the company, product line, or individual level)

2. Market share growth and other standard financial measures

EMPLOYEE

1. Associate satisfaction

SET .7

Q: 1) (A) Describe the factors which impact service organization

Ans: Factors which impact service organization:

Absence of Inventory Buffer:

Goods can be held as inventory, which is a buffer that dampens the impact on production activity of fluctuations in sales

volume. Services cannot be stored. The airplane seat, hotel room, hospital operating room, or the hours of lawyers, physicians,

scientists, and other professionals that are not used today are gone forever. Thus, although a manufacturing company can

earn revenue in the future from products that are on hand today, a service company cannot do so. It must try to minimize its

unused capacity.

32

Page 33: MCS theory

Moreover, the costs of many service organizations are essentially fixed in the short run. In the short run, a hotel cannot reduce

its costs substantially by closing off some of its rooms. Accounting firms, law firms, and other professional organizations are

reluctant to layoff professional personnel in times of low sales volume because of the effect on morale and the costs of

rehiring and training.

Difficulty in Controlling Quality:

A manufacturing company can inspect its products before they are shipped to the consumer, and their quality can be

measured visually or with instruments (tolerances, purity, weight, color, and so on). A service company cannot judge product

quality until the moment the service is rendered, and then the judgments are often subjective. Restaurant management can

examine the food in the kitchen, but customer satisfaction depends to a considerable extent on the way it is served. The

quality of education is so difficult to measure that few educational organizations have a formal quality control system.

Labor Intensive:

Manufacturing companies add equipment and automate production lines, thereby replacing labor and reducing costs. Most

service companies are labor intensive and cannot do this. Hospitals do add expensive equipment, but mostly to provide better

treatment, and this increases costs. A law firm expands by adding partners and new support personnel.

Multi-Unit Organizations:

Some service organizations operate many units in various locations; each unit relatively small. These organizations are fast-

food restaurant chains, auto rental companies, gasoline service stations, and many others. Some of the units are owned;

others operate under a franchise. The similarity of the separate units provides a common basis for analyzing budgets and

evaluating performance not available to the manufacturing company. The information for each unit can be compared with

system wide or regional averages, and high performers and low performers can be identified. However because units differ in

the mix of services they provide, in the resources that they use, and in other ways, care must be taken in making such

comparisons.

Q:1) (B) Explain special characteristics of professional organization which would have a bearing on their control

system.

Ans: Special Characteristics of Professional Organization:

Goals:

A dominant goal of a manufacturing company is to earn a satisfactory profit, specifically a satisfactory return on assets

employed. A professional organization has relatively few tangible assets; its principal asset is the skill of its professional staff,

which doesn't appear on its balance sheet. Return on assets employed, therefore, is essentially meaningless in such

organizations. Their financial goal is to provide adequate compensation to the professionals.

In many organizations, a related goal is to increase their size. In part, this reflects the natural tendency to associate success

with large size. In part, it reflects economies of scale in using the efforts of a central personnel staff and units responsible for

keeping the organization up to- date. Large public accounting firms need to have enough local offices to enable them to audit

clients who have facilities located throughout the world.

Professionals:

Professional organizations are labor intensive, and the labor is of a special type. Many professionals prefer to work

independently, rather than as part of a team. Professionals who are also managers tend to work only part time on

management activities; senior partners in an accounting firm participate actively in audit engagements; senior partners in law

firms have clients.

Education for most professions does not include education in management, but quite naturally stresses the skills of the

profession, rather than management; for this and other reasons, professionals tend to look down on managers. Professionals

tend to give inadequate weight to the financial implications of their decisions; they want to do the best job they can, re- I

regardless of its cost. This attitude affects the attitude of support staffs and nonprofessionals in the organization; it leads to

inadequate cost control.

Output and Input Measurement:

The output of a professional organization cannot be measured in physical terms, such as units, tons, or gallons. We can

measure the number of hours a lawyer spends on a case, but this is a measure of input, not output. Output is the

effectiveness of the lawyer's work, and this is not measured by the number of pages in a brief or the number of hours in the

courtroom. We can measure the number of patients a physician treats in a day, and even classify these visits by type of

complaint; but this is by no means equivalent to measuring the amount or quality of service the physician has provided. At

most, what is measured is the physician's efficiency in treating patients, which is of some use in identifying slackers and hard

workers. Revenues earned is one measure of output in some professional organizations, but these monetary amounts, at

33

Page 34: MCS theory

most, relate to the quantity of services rendered, not to their quality (although poor quality is reflected in reduced revenues in

the long run).

Furthermore, the work done by many professionals is non repetitive. No two consulting jobs or research and development

projects are quite the same. This makes it difficult to plan the time required for a task, to set reasonable standards for task

performance, and to judge how satisfactory the performance was. Some tasks are essentially repetitive: the drafting of simple

wills, deeds, sales contracts, and similar documents; the taking of a physical inventory by an auditor; and certain medical and

surgical procedures. The development of standards for such tasks may be worthwhile, although in using these standards,

unusual circumstances that affect a specific job must be taken into account.

Small Size:

With a few exceptions, such as some law firms and accounting firms, professional organizations are relatively small and

operate at a single location. Senior management in such organizations can personally observe what is going on and personally

motivate employees. Thus, there is less need for a sophisticated management control system, with profit centers and formal

performance reports. Nevertheless, even a small organization needs a budget, a regular comparison of performance against

budget, and a way of relating compensation to performance.

Marketing:

In a manufacturing company there is a clear dividing line between marketing activities and production activities; only senior

management is concerned with both. Such a clean separation does not exist in most professional organizations. In some, such

as law, medicine, and accounting, the profession's ethical code limits the amount and character of overt marketing efforts by

professionals (although these restrictions have been relaxed in recent years). Marketing is an essential activity in almost all

organizations, however. If it can't be conducted openly, it takes the form of personal contacts, speeches, articles,

conversations on the golf course, and so on. These marketing activities are conducted by professionals, usually by

professionals who spend much of their time in production work-that is, working for clients.

In this situation, it is difficult to assign appropriate credit to the person responsible for "selling" a new customer. In a

consulting firm, for example, a new engagement may result from a conversation between a member of the firm and an

acquaintance in a company, or from the reputation of one of the firm's professionals as an outgrowth of speeches or articles.

Moreover, the professional who is responsible for obtaining the engagement may not be personally involved in carrying it out.

Until fairly recently, these marketing contributions were rewarded subjectively- that is, they were taken into account in

promotion and compensation decisions. Some organizations now give explicit credit, perhaps as a percentage of the project's

revenue, if the person who "sold" the project can be identified.

Q:2) Every SBU is a profit center but every profit center is not a SBU? What are the conditions that should be fulfill for an organization unit to be converted into a profit center? What are the different ways to measure the performance of profit center? Discuss their relevant merits and demerits.

Ans: Conditions for an organization to be converted into a profit centre: Many management decisions involve

proposals to increase expenses with the expectation of an even greater increase in sales revenue. Such decisions are said to

involve expense/revenue trade-offs. Additional advertising expense is an example. Before it is safe to delegate such a trade-

off decision to a lower-level manager, two conditions should exist.

The manager should have access to the relevant information needed for making such a decision.

There should be some way to measure the effectiveness of the trade-offs the manager has made.

A major step in creating profit centers is to determine the lowest point in an organization where these two conditions prevail.

All responsibility centers fit into a continuum ranging from those that clearly should be profit centers to those that clearly

should not. Management must decide whether the advantages of giving profit responsibility offset the disadvantages, which

are discussed below. As with all management control system design choices, there is no clear line of demarcation.

Ways to Measure Performance:

There are two types of profitability measurements used in evaluating a profit center, just as there are in evaluating an

organization as a whole. First, there is the measure of management performance, which focuses on how well the manager is

doing. This measure is used for planning, coordinating, and controlling the profit center's day-to-day activities and as a device

34

Page 35: MCS theory

for providing the proper motivation for its manager. Second, there is the measure of economic performance, which focuses on

how well the profit center is doing as an economic entity. The messages conveyed by these two measures may be quite

different from each other. For example, the management performance report for a branch store may show that the store's

manager is doing an excellent job under the circumstances, while the economic performance report may indicate that

because of economic and competitive conditions in its area the store is a losing proposition and should be closed. .

The necessary information for both purposes usually cannot be obtained from a single set of data. Because the management

report is used frequently, while the economic report is prepared only on those occasions when economic decisions must be

made, considerations relating to management performance measurement have first priority in systems design-that is, the

system should be designed to measure management performance routinely, with economic information being derived from

these performance reports as well as from other sources.

Types of Profitability Measures

A profit center's economic performance is always measured by net income (i.e., the income remaining after all costs, including

a fair share of the corporate overhead, have been allocated to the profit center). The performance of the profit center

manager, however, may be evaluated by five different measures of profitability: (1) contribution margin, (2) direct profit, (3)

controllable profit, (4) income before income taxes, or (5) net income

(1) Contribution Margin:

Contribution margin reflects the spread between revenue and variable expenses. The principal argument in favor of using it to

measure the performance of profit center managers is that since fixed expenses are beyond their control, managers should

focus their attention on maximizing contribution. The problem with this argument is that its premises are inaccurate; in fact,

almost all fixed expenses are at least partially controllable by the manager, and some are entirely controllable. Many expense

items are discretionary; that is, they can be changed at the discretion of the profit center manager. Presumably, senior

management wants the profit center to keep these discretionary expenses in line with amounts agreed on in the budget

formulation process. A focus on the contribution margin tends to direct attention away from this responsibility. Further, even if

an expense, such as administrative salaries, cannot be changed in the short run, the profit center manager is still responsible

for controlling employees' efficiency and productivity.

(2) Direct Profit:

This measure reflects a profit center's contribution to the general overhead and profit of the corporation. It incorporates all

expenses either incurred by or directly traceable to the profit center, regardless of whether or not these items are within the

profit center manager's control. Expenses incurred at headquarters, however, are not included in this calculation. A weakness

of the direct profit measure is that it does not recognize the motivational benefit of charging headquarters costs.

(3) Controllable Profit:

Headquarters expenses can be divided into two categories: controllable and non controllable. The former category includes

expenses that are controllable, at least to a degree, by the business unit manager-information technology services, for

example. If these costs are included in the measurement system, profit will be what remains after the deduction of all

expenses that may be influenced by the profit center manager. A major disadvantage of this measure is that because it

excludes non controllable headquarters expenses it cannot be directly compared with either published data or trade

association data reporting the profits of other companies in the industry.

(4) Income before Taxes:

In this measure, all corporate overhead is allocated to profit centers based on the relative amount of expense each profit

center incurs. There are two arguments against such allocations. First, since the costs incurred by corporate staff departments

such as finance, accounting, and human resource management are not controllable by profit center managers, these

managers should not be held accountable for them. Second, it may be difficult to allocate corporate staff services in a manner

that would properly reflect the amount of costs incurred by each profit center.

There are, however, three arguments in favor of incorporating a portion of corporate overhead into the profit centers'

performance reports. First, corporate service units have a tendency to increase their power base and to enhance their own

excellence without regard to their effect on the company as a whole. Allocating corporate overhead costs to profit centers

increases the likelihood that profit center manager§ will question these costs, thus serving to keep head office spending in

check. (Some companies have actually been known to sell their corporate jets because of complaints from profit center

managers about the cost of these expensive items.) Second, the performance of each profit center will become more realistic

and more readily comparable to the performance of competitors who pay for similar services. Finally, when managers know

35

Page 36: MCS theory

that their respective centers will not show a profit unless all-costs, including the allocated share of corporate overhead, are

recovered, they are motivated to make optimum long-term marketing decisions as to pricing, product mix, and so forth, that

will ultimately benefit (and even ensure the viability of) the company as a whole.

If profit centers are to be charged for a portion of corporate overhead, this item should be calculated on the basis of budgeted,

rather than actual, costs, in which case the "budget" and "actual" columns in the profit center's performance report will show

identical amounts for this particular item. This ensures that profit center managers will not complain about either the

arbitrariness of the allocation or their lack of control over these costs, since their performance reports will show no variance in

the overhead allocation. Instead, such variances would appear in the reports of the responsibility center that actually incurred

these costs. .

(5) Net Income:

Here, companies measure the performance of domestic profit centers according to the bottom line, the amount of net income

after income tax. There are two principal arguments against using this measure: (1) after tax income is often a constant

percentage of the pretax income, in which case there would be no advantage in incorporating income taxes, and (2) since

many of the decisions that affect income taxes are made at headquarters, it is not appropriate to judge profit center managers

on the consequences of these decisions. There are situations, however, in which the effective income tax rate does vary

among profit centers. For example, foreign subsidiaries or business units with foreign operations may have different effective

income tax rates. In other cases, profit centers may influence income taxes through their installment credit policies, their

decisions on acquiring or disposing of equipment, and their use of other generally accepted accounting procedures to

distinguish gross income from taxable income. In these situations, it may be desirable to allocate income tax expenses

to profit centers not only to measure their economic profitability but also to motivate managers to minimize tax liability.

Merits:

The quality of decisions may improve because they are being made by managers closest to the point of decision.

The speed of operating decisions may be increased since they do not have to be referred to corporate headquarters. .

Headquarters management, relieved of day-to-day decision making, can concentrate on broader issues.

Managers, subject to fewer corporate restraints, are freer to use their imagination and initiative.Because profit

centers are similar to independent companies, they provide an excellent training ground for general management.

Their managers gain experience in managing all functional areas, and upper management gains the opportunity to

evaluate their potential for higher-level jobs.

Profit consciousness is enhanced since managers who are responsible' for profits will constantly seek ways to

increase them. (A manager responsible for marketing activities, for example, will tend to authorize promotion

expenditures that increase sales, whereas a manager responsible for profits will be motivated to make promotion

expenditures that increase profits.).

Profit centers provide top management with ready-made information on the profitability of the company's individual

components. . Because their output is so readily measured, profit centers are particularly responsive to pressures to

improve their competitive performance.

Demerits:

Decentralized decision making will force top management to rely more on management control reports than

on personal knowledge of an operation, entailing some loss of control.

If headquarters management is more capable or better informed than the average profit center manager, the

quality of decisions made at the unit level may be reduced.

Friction may increase because of arguments over the appropriate transfer price, the assignment of common

costs, and the credit for revenues that were formerly generated jointly by two or more business units working

together.

Organization units that once cooperated as functional units may now be in competition with one another. An

increase in profits for one manager may mean a decrease for another. In such situations, a manager may fail

to refer sales leads to another business unit better qualified to pursue them; may hoard personnel or

equipment that, from the overall company standpoint, would be better off used in another unit; or may make

production decisions that have undesirable cost consequences for other units.

Divisionalization may impose additional costs because of the additional management, staff personnel, and

record keeping required, and may lead to task redundancies at each profit center.

36

Page 37: MCS theory

Q:3) what are different types of Strategic Missions at SBU level? How do these missions affect Strategic Planning process and Budgeting at SBU Level?

Different Types of Strategic Missions:

Business Unit Mission:

In a diversified firm one of the important tasks of senior management is resource deployment, that is, make decisions

regarding the use of the cash generated from some business units to finance growth in other business units. Several planning

models have been developed to help corporate level managers of diversified firms to effectively allocate resources. These

models suggest that a firm has business units in several categories, identified by their mission; the appropriate strategies for

each category differ. Together, the several units make up a portfolio, the components of which differ as to their risk/reward

characteristics just as the components of an investment portfolio differ. Both the corporate 'office and the business unit

general manager are involved in identifying the missions of individual business units. Of the many planning models, two of the

most widely used are Boston Consulting Group's two-by-two growth-share matrix and General Electric Company/McKinsey &

Company's three-by-three industry attractiveness-business strength matrix. While these models differ in the methodologies

they use to develop the most appropriate missions for the various business units, they have the same set of missions from

which to choose: build, hold, harvest, and divest.

Build: This mission implies an objective of increased market share, even at the expense of short-term earnings and

cash flow (e.g., Merck's bio-technology, Black and Decker's handheld electric tools).

Hold: This strategic mission is geared to the protection of the business unit's market share and competitive

position (e.g.: IBM's mainframe computers).

Harvest: This mission has the objective of maximizing short-term earnings and cash flow, even at the

expense of market share (e.g., American Brands' tobacco products, General Electric's and Sylvania's light bulbs)

Divest: This mission indicates a decision to withdraw from the business either through a process of slow

liquidation or outright sale. While the planning models can aid in the formulation of missions, they are not cook books. A

business unit's position on a planning grid should not be the sole basis for deciding its mission.

Business Unit Competitive Advantage: Every business unit should develop a competitive advantage in

order to accomplish its mission. Three interrelated questions have to be considered in developing the business unit's

competitive= advantage. First, what is the structure of the industry in which the business unit operates? Second, how should

the business unit exploit the industry's structure? Third, what will be the basis of the business unit's competitive advantage?

Industry Analysis: Research has highlighted the important role industry conditions play in the performance

of individual firms. Studies have shown that average industry profitability is, by far, the most significant predictor of firm

performance. According to Porter, the structure of an industry should be analyzed in terms of the collective strength of five

competitive forces.

1. The intensity of rivalry among existing competitors. Factors affecting direct rivalry are industry growth, product

differentiability, number and diversity of competitors, level of fixed costs, intermittent overcapacity, and exit barriers.

2. The bargaining power of customers. Factors affecting buyer power are number of buyers, buyer's switching costs, buyer's

ability to integrate backward, impact of the business unit's product on buyer's total costs, impact of the business unit's

product on buyer's product quality/ performance, and significance of the business unit's volume to buyers.

3. The bargaining power of suppliers. Factors affecting supplier power are number of suppliers, supplier's ability to integrate

forward, presence of substitute inputs, and importance of the business unit's volume to suppliers.

4. Threat from substitutes. Factors affecting substitute threat are relative price/performance of substitutes, buyer's switching

costs, and buyer's propensity to substitute.

5. The threat of new entry. Factors affecting entry barriers are capital requirements, access to distribution channels,

economies of scale, product differentiation, technological complexity of product or process, expected retaliation from existing

firms, and government policy.

We make three observations with regard to the industry analysis:

1. The more powerful the five forces are, the less profitable an industry is likely to be. In industries where average profitability

is high (such as soft drinks and pharmaceuticals), the five forces are weak (e.g., in the soft drink industry, entry barriers are

high). In industries where the average profitability is low (such as steel and coal), the five forces are strong (e.g., in the steel

industry, threat from substitutes is high).

37

Page 38: MCS theory

2. Depending on the relative strength of the five forces, the key strategic issues facing the business unit will differ from one

industry to another.

3. Understanding the nature of each force helps the firm to formulate effective strategies. Supplier selection (a strategic issue)

is aided by the analysis of the relative power of several supplier groups; the business unit should link with the supplier group

for which it has the best competitive advantage. Similarly, analyzing the relative bargaining power of several buyer groups will

facilitate selection of target customer segments.

Generic Competitive Advantage:

The five-force analysis is the starting point for developing a competitive advantage since it helps to identify the opportunities

and threats in the external environment. With this understanding, Porter claims that the business unit has two generic ways of

responding to the opportunities in the external environment and developing a sustainable competitive advantage: low cost

and differentiation.

Low Cost: Cost leadership can be achieved through such approaches as economies of scale in production;

experience curve effects, tight cost control, and cost minimization (in such areas as research and development, service, sales

force, or advertising). Some firms following this strategy include Charles Schwab in discount brokerage, Wal-Mart in discount

retailing, Texas Instruments in consumer electronics, Emerson Electric in electric motors, Hyundai in automobiles, Dell in

computers, Black and Decker in machine tools, Nucor in steel, Lincoln Electric in arc welding equipment, and BIC in pens.

Differentiation:

The primary focus of this strategy is to differentiate the product offering of the business unit, creating something that is

perceived by customers as being unique. Approaches to product differentiation include brand loyalty (Coca-Cola and Pepsi

Cola in soft drinks), superior customer service (Nordstrom in retailing), dealer network (Caterpillar Tractors in construction

equipment), product design and product features (Hewlett-Packard in electronics), and technology (Cisco in communications

infrastructure). Other examples of firms following a differentiation strategy include BMW in automobiles; Stouffer's in frozen

foods, Neiman-Marcus in retailing, Mont Blanc in pens, and Rolex in wristwatches.

SET 8

1. What is a responsibility centre? List and explain different types of Responsibility Centers with sketches.

Responsibility centers:

A responsibility center is an organization unit that is headed by a manager who is responsible for its activities. In a sense, a company is a collection of responsibility centers. Each of which is represented by box on the on the organization are responsibility centers for section work shifts or other small organization units. At a higher level are departments or business units that consist of several of these smaller units plus staff and management people these larger units are also responsibility center. And from the stand point of senior management and the board of directors, the whole company is responsibility center although the term is usually used to refer to unit within the company.

Nature of responsibility centers

A responsibility center exist one or more purpose are its objectives. The company as a whole has goals, and senior management has decided on a set of strategies to accomplish these goals. The objectives of responsibility centers are to help implement these strategies. Because the organization is the sum of its responsibility centers, if the strategies are sound and if each responsibility center, if the strategies are sound and if each responsibility center meets its objectives the whole organization should achieve its goals. A responsibility center uses inputs, and a variety of services. Its work with these resources and it usually require working capital, equipment, and other asset to do this work. As a result of this work the responsibility center produces output which is classified either as goods if they are tangible or as services if they are intangible. Every responsibility center has output that is it does something. In a production plant, the outputs are goods. In staff units, such as human resources, transportation, engineering, accounting, and administration, the output s are services.

38

Page 39: MCS theory

For many responsibility centers, especially staff units, outputs are difficult to measure; nevertheless, they exist. The products produced by a responsibility center or to the outside marketplace. In the first case, the product are inputs to the other responsibility center in the latter case, they are output s of the whole organization.

Types of Responsibility Centers

Cost Center

Cost centers are divisions that add to the cost of the organization, but only indirectly add to the profit of the company. Typical examples include Research and Development, Marketing and Customer service. Companies may choose to classify business units as cost centers, profit centers, or investment centers. There are some significant advantages to classifying simple, straightforward divisions as cost centers, since cost is easy to measure. However, cost centers create incentives for managers to underfund their units in order to benefit themselves, and this underfunding may result in adverse consequences for the company as a whole (reduced sales because of bad customer service experiences, for example). Because the cost centre has a negative impact on profit (at least on the surface) it is a likely target for rollbacks and layoffs when budgets are cut. Operational decisions in a contact centre, for example, are typically driven by cost considerations. Financial investments in new equipment, technology and staff are often difficult to justify to management because indirect profitability is hard to translate to bottom-line figures. Business metrics are sometimes employed to quantify the benefits of a cost centre and relate costs and benefits to those of the organization as a whole. In a contact centre, for example, metrics such as average handle time, service level and cost per call are used in conjunction with other calculations to justify current or improved funding.

Profit Center

A responsibility centre is called a profit centre when the manager is held responsible for both costs (inputs) and revenues (outputs) and thus for profit. Despite the name, a profit centre can exist in nonprofits organizations (though it might not be referred to as such) when a responsibility centre receives revenues for its services. A profit centre is a big segment of activity for which both revenues and costs are accumulated: A centre, whose performance is measured in terms of both - the expense it incurs and revenue it earns, is termed as a profit centre. The output of a responsibility centre may either be meant for internal consumption or for outside customers. In the latter case, the revenue is realized when the sales are made. That is, when the output is meant for outsiders, then the revenue will be measured from the price charged from customers. If the output is meant for other responsibility centre, then management takes a decision whether to treat the centre as profit centre or not. In fact, any responsibility centre can be turned into a profit centre by determining a selling price for its outputs. For instance, in case of a process industry, the output of one process may be transferred to another process at a profit by taking into account the market price. Such transfers will give some profit to that responsibility centre. Although such transfers do not increase the Company’s assets, they help in management control process.

Investment Centre

An investment centre goes a step further than a profit centre does. Its success is measured not only by its income but also by

relating that income to its invested capital, as in a ratio of income to the value of the capital employed. In practice, the term investment centre is not widely used. Instead, the term profit centre is used indiscriminately to describe centers that are always assigned responsibility for revenues and expenses, but may or may not be assigned responsibility for the capital investment. It is defined as a responsibility centre in which inputs are measured in terms of cost / expenses and outputs are measured in terms of revenues and in which assets employed are also measured. A responsibility centre is called an investment centre, when its manager is responsible for costs and revenues as well as for the investment in assets used by his centre. He is responsible for maintaining a satisfactory return on investment i.e. asset employed in his responsibility centre. The

39

Page 40: MCS theory

investment centre manager has control over revenues, expenses and the amounts invested in the centre’s assets. The manager of an investment centre is required to earn a satisfactory return. Thus, return on investment (ROI) is used as the performance evaluation criterion in an investment centre. He also formulates the credit policy, which has a direct influence on debt collection, and the inventory policy, which determines the investment in inventory. The Vice President (Investments) of a mutual funds company may be in charge of an Investment Centre. In the Investment Centre, the manager in charge is held responsible for the proper utilization of assets. He is expected to earn a satisfactory return on the assets employed in his responsibility centre. Measurement of assets employed poses many problems. It becomes difficult to determine the amount of assets employed in a particular responsibility centre. Some of the assets are in the physical possession of the responsibility centre while for some assets it may depend upon other responsibility centers or the Head Office of the company. This is particularly true of cash or heavy plant and equipment. Whether such assets should be included in the figure of assets employed of the responsibility centre and if included, at how much value, is a difficult question. On account of these difficulties, investment centers are generally used only for relatively large units, which have independent divisions, both manufacturing and marketing, for their individual products.

2. Explain the process of evaluation of Responsibility Center from one stage to another with the help of illustration-cum-experiences of the corporate.

Process of evaluation of Responsibility Center.

1. The organization is divided into various responsibility centers. Each responsibility centre is put under the charge of a responsibility manager.

2. The targets or budgets of each responsibility centre are set in consultation with the manager of responsibility centre, so that he may be able to give full information about his department. The manager of responsibility centre should know as what is expected of him - each centre should have a clear set of goals. The responsibility and authority of each centre should be well defined.

3. Managers are charged with the items and responsibility, over which they can exercise a significant degree of direct control.

4. Goals defined for each area of responsibility should be attainable with efficient and effective performance.5. The actual performance is communicated to the managers concerned. If it falls short of the standards, the variances

are conveyed to the top management. The names of persons responsible for the variances are also conveyed so that responsibility may be fixed.

The purpose of all these steps is to assign responsibility to different individuals so that their performance is improved and costs are controlled. The personal factor in Responsibility Accounting is most important. The management may prepare the best plan or the budget and put up before its staff, but its success depends upon the initiative and the will of the workers to execute it

Example of Responsibility CenterThe Sarva Shiksha Abhiyan emphasizes quality improvement in elementary education for

which it deems necessary that resource groups and responsibility centers from national to sub-district levels are identified. These groups would oversee the policy, planning, implementation and monitoring of all quality related interventions. Their major role would be to advise and assist at various levels in curriculum development, pedagogical improvement, teacher education/training and activities related to classroom transaction. In order to facilitate a decentralized mode of education, these groups would need to be constituted at various operational levels, namely - national, state, district and sub district. The following could be involved in the groups:

National level - NCERT, NIEPA, Universities, NGOs, experts and eminent educationists.

40

Page 41: MCS theory

State level - SCERT, SIEMAT, Universities, IASEs/CTEs, NGOs, experts and eminent educationists.

District level - DIETs, representatives from DPEP District Resource Group, higher educational institutions, innovative teachers from the districts, NGOs.

Sub-district - BRC/BEO, representatives from CRCs, innovative teachers.

3. Briefly define Discretionary Expense Center, Engineered Expense Center, Profit Centre and Investment

Centre? How is budget prepared in Discretionary Expenses Centre?

Engineered expense centers:

Engineered expense center have the following characteristics:

- Their inputs can be measured in monetary terms.- Their output can be measured in physical terms.- The optimal dollar amount of input required to produce one unit of output can be established.

Engineered expense center usually are found in manufacturing operations. Warehousing, distribution, trucking and similar units in the marketing organization also may be engineered expense center and so many certain responsibility center within administrative and support department. Examples are accounts receivable account payable and payroll section in the controller department personnel record and cafeteria in the human resource department shareholder record in the corporate secretary department and the company motor pool. Such units perform repetitive task for which standard cost can be developed. In an engineered expense center the output multiplied by the standard cost of each unit produced represents what the finished product should have cost. When this cost is compared to actual costs, the difference between the two represents the efficiency of the organization unit being measured. We emphasize that engineered expense centers have other important tasks not measured by cast alone. The effectiveness of this aspect of performance should be controlled.

For example expenses center supervisor are responsible for the quality of good and for the volume of production in addition to their responsibility for cost efficiency. Therefore the type and amount of production is prescribed and specific quality standards are set so that manufacturing costs are not minimized at the expense of quality. Moreover manager of engineered expense center may be responsible for activities such a training that are not related to current production judgment about their performance should include an appraisal of how well they carry out these responsibilities. There are few if any responsibility center in which all cost items are engineered. Even in highly automated production department the amount of indirect labor and of various services used can vary with management discretion. Thus, the term engineered costs center refers to responsibility center in which engineered cost predominate but in does not imply that valid engineering estimates can be made for each and every cost item.

Discretionary expense center:

The output of discretionary expenses center cannot be measured in monitory terms. They include administration and support units research and development organization and most marketing activities. The term discretionary does not mean that management judgment is capricious or haphazard. Management has decided on certain policies that should govern the operation of the company. Whether to match exceed or spend less than the marketing effort of its competitor; the level of service that the company provides to the customer. The appropriate amount of spending for R & D, financial planning public relation and many other activities. One company may have a small headquarter staff another company of similar size and in the same industry may have a staff that is 10 times as large. the management of both companies may be concerned that they made the correct decision on staff size but there is no objective way judging which decision was actually better manager are hired and paid to make such decision. After such a drastic change the level of discretionary expenses generally has a similar pattern from one year to the next. The difference between budgeted and actual expense is not a measure of efficiency in a discretionary expense center it is simply the difference between the budgeted input and the actual input. It in no way measures the value of the output. if actual expense do not exceed the budget amount, the manager has ‘lived within the budget ‘ however ,because by definition the budget does not purport to measure the optimum amount of spending we cannot say that living within the budgeted is efficient performance.

Profit Center

A responsibility centre is called a profit centre when the manager is held responsible for both costs (inputs) and revenues (outputs) and thus for profit. Despite the name, a profit centre can exist in nonprofits organizations (though it might not be referred to as such) when a responsibility centre receives revenues for its services. A

41

Page 42: MCS theory

profit centre is a big segment of activity for which both revenues and costs are accumulated: A centre, whose performance is measured in terms of both - the expense it incurs and revenue it earns, is termed as a profit centre. The output of a responsibility centre may either be meant for internal consumption or for outside customers. In the latter case, the revenue is realized when the sales are made. That is, when the output is meant for outsiders, then the revenue will be measured from the price charged from customers. If the output is meant for other responsibility centre, then management takes a decision whether to treat the centre as profit centre or not. In fact, any responsibility centre can be turned into a profit centre by determining a selling price for its outputs. For instance, in case of a process industry, the output of one process may be transferred to another process at a profit by taking into account the market price. Such transfers will give some profit to that responsibility centre. Although such transfers do not increase the Company’s assets, they help in management control process.

Investment Centre

An investment centre goes a step further than a profit centre does. Its success is measured not only by its income but also by relating that income to its invested capital, as in a ratio of income to the value of the capital employed. In practice, the term investment centre is not widely used. Instead, the term profit centre is used indiscriminately to describe centers that are always assigned responsibility for revenues and expenses, but may or may not be assigned responsibility for the capital investment. It is defined as a responsibility centre in which inputs are measured in terms of cost / expenses and outputs are measured in terms of revenues and in which assets employed are also measured. A responsibility centre is called an investment centre, when its manager is responsible for costs and revenues as well as for the investment in assets used by his centre. He is responsible for maintaining a satisfactory return on investment i.e. asset employed in his responsibility centre. The investment centre manager has control over revenues, expenses and the amounts invested in the centre’s assets. The manager of an investment centre is required to earn a satisfactory return. Thus, return on investment (ROI) is used as the performance evaluation criterion in an investment centre. He also formulates the credit policy, which has a direct influence on debt collection, and the inventory policy, which determines the investment in inventory.

Budget Preparation.

The decision that management make about a discretionary expense budget are different from the decisions that it makes about the budget for an engineered expense center. For the latter management decides whether the proposed operating budget represent the cost of performing task efficiently for the coming period. management is not so much concerned with the magnitude of the task because this is largely determined by the actions of other responsibility centers, such as the marketing departments ability to generate sales. In formulating the budget for a discretionary expense center, however management principal task is to decide on the magnitude of the job that should be done. These tasks can be divided generally into two types continuing and special. Continuing task are those that continue from year to year for example financial statement preparation by the controller’s office. Special tasks are one shot project for example developing and installing a profit budgeting system in a newly acquired division. The technique management by objective is often used in preparing the budget for a discretionary expense center. Management by objective is a formal process in which a budget purposes to accomplish specific tasks and state a mean for measuring whether these tasks have been accomplished. There are two different approach to planning for the discretionary expense center increment budgeting and zero based review.

SET .9

Q1. Describe inherent difficulties creation of profit centres may cause and advantages possible?Under which situation creation of profit centre is not advisable.

Under which situation creation of profit centre is not advisableDecentralized decision making will force top management to rely more on management control reports than on personal knowledge of an operation, entailing some loss of control. If headquarters management is mere capable or better informed than the average profit center manager, the quality of decisions made at the unit level way be reduced. Friction may increase because of arguments over the appropriate transfer price, the assignment of common costs, and the credit for revenues that were formerly generated jointly by two or more business units working together.

Organization units that once cooperated as functional units may now be in competition with one another. An increase in profits for one manager may mean a decrease for another. In such situation a manager may fail to refer sales leads to another business unit better qualified to pursue them; may hoard personnel or equipment that, from the overall company’s, standpoint, would be better off used in another unit; or may make production decisions that have undesirable cost consequences for other units.

Divisionalization may impose additional costs because of the additional management, staff personnel, and record keeping required, and may lead to task redundancies at each profit center.Competent general managers may not exist in a functional organization because there may not have been sufficient opportunities for them to develop general management competence.

42

Page 43: MCS theory

There may be too much emphasis on short-run profitability at the expense of long-run profitability. In the desire to report high current profits, the profit center manager may skimp on R&D, training programs, or maintenance. This tendency is especially prevalent when the turnover of profit center managers is relatively high. In these circumstances, managers may have good reason to believe that their actions may not affect profitability until after they have moved to other jobs. There is no completely satisfactory system for ensuring that optimizing the profits of each individual profit center will optimize the profits of the company as a whole.

Q2.What are the challenges faced in pricing corporate services provided to Business Units operating as “profit centers?”

Business Units as Profit CentersMost business units are created as profit centers since managers in charge of such units typically control product development, manufacturing, and marketing resources. These managers are in a position to influence revenues and costs and as such can be held accountable for the "bottom line." However, as pointed out in the next section, a business unit manager's authority may be constrained in various ways, which ought to be reflected in a profit center's design and operation.

Constraints on Business Unit AuthorityTo realize fully the benefits of the profit center concept, the business unit manager would have to be as autonomous as the president of an independent company. As a practical matter, however, such autonomy is not feasible. If a company were divided into completely independent units, the organization would lose the advantages of size and synergy. Furthermore in delegating to business unit management all the authority that the board of directors has given to the CEO, senior management would be abdicating its own responsibility. Consequently, business unit structures represent trade-offs between business unit autonomy and corporate constraints. The effectiveness of a business unit organization is largely dependent on how well these trade-offs are made.

Constraints from Other Business Units.One of the main problems occurs when business units must deal with one another. It is useful to think of managing a profit center in terms of control over three types of decisions: (1) The product decision (what goods or services to make and sell), (2) The marketing decision (how, where, and for how much are these goods or services to be sold?), and (3) The procurement or sourcing decision (how to obtain or manufacture the goods or services). If a business unit manager controls all three activities, there is usually no difficulty in assigning profit responsibility and measuring performance. In general, the greater the degree of integration within a company,the more difficult it becomes to assign responsibility to a single profit center for all three activities in a given product line; that is, if the production, procurement, and marketing decisions for a single product line are split among two or more business units, separating the contribution of each business unit to the overall success of the product line may be difficult.

Constraints from Corporate ManagementThe constraints imposed by corporate management can be grouped into three types: (1) Those resulting from strategic considerations,(2) Those resulting because uniformity is required, and(3) Those resulting from the economies of centralization.

Most companies retain certain decisions, especially financial decisions, at the corporate level, at least for domestic activities. Consequently, one of the major constraints on business units results from corporate control over new investments. Business units must compete with one another for a share of the available funds. Thus, a business unit could find its expansion plans thwarted because another unit has convinced senior management that it has a more

Attractive program. Corporate management .also imposes other constraints. Each business unit has a "charter" that specifies the marketing and/or production activities that it is permitted to undertake, and it must refrain from operating beyond its charter, even though it sees profit opportunities in doing so. Also, the maintenance of the proper corporate image may require constraints on the quality of products or on public relations activities.

Companies impose some constraints on business units because of the necessity for Uniformity. One-constraint is that business Units must conform to corporate accounting and MCS This constraint is especially troublesome for units that have been acquired from another company and that have been accustomed to using different systems.

Q.3) Write Short Notes on

1. Zero Based Budgeting

2. Internal Control\

Zero Based Budgeting:: Zero-based budgeting is a technique of planning and decision-making which reverses the working process of traditional budgeting. In traditional incremental budgeting, departmental managers justify only increases over the previous year budget and what has been already spent is automatically sanctioned. No reference is made to the previous level

43

Page 44: MCS theory

of expenditure. By contrast, in zero-based budgeting, every department function is reviewed comprehensively and all expenditures must be approved, rather than only increases.[1] Zero-based budgeting requires the budget request be justified in complete detail by each division manager starting from the zero-base. The zero-base is indifferent to whether the total budget is increasing or decreasing.

The term "zero-based budgeting" is sometimes used in personal finance to describe the practice of budgeting every dollar of income received, and then adjusting some part of the budget downward for every other part that needs to be adjusted upward. It would be more technically correct to refer to this practice as "active-balanced budgeting".

Advantages of Zero-Based Budgeting:

1. Efficient allocation of resources, as it is based on needs and benefits. 2. Drives managers to find cost effective ways to improve operations. 3. Detects inflated budgets. 4. Municipal planning departments are exempt from this budgeting practice. 5. Useful for service departments where the output is difficult to identify. 6. Increases staff motivation by providing greater initiative and responsibility in decision-making. 7. Increases communication and coordination within the organization. 8. Identifies and eliminates wasteful and obsolete operations. 9. Identifies opportunities for outsourcing. 10. Forces cost centers to identify their mission and their relationship to overall goals.

Disadvantages of Zero-Based Budgeting:

1. Difficult to define decision units and decision packages, as it is time-consuming and exhaustive. 2. Forced to justify every detail related to expenditure. The R&D department is threatened whereas the production

department benefits. 3. Necessary to train managers. Zero-based budgeting must be clearly understood by managers at various levels to be

successfully implemented. Difficult to administer and communicate the budgeting because more managers are involved in the process.

4. In a large organization, the volume of forms may be so large that no one person could read it all. Compressing the information down to a usable size might remove critically important details.

5. Honesty of the managers must be reliable and uniform. Any manager that exaggerates skews the results

Internal Control:

Internal control is defined as a process affected by an organization's structure, work and authority flows, people and management information systems, designed to help the organization accomplish specific goals or objectives.[1] It is a means by which an organization's resources are directed, monitored, and measured. It plays an important role in preventing and detecting fraud and protecting the organization's resources, both physical (e.g., machinery and property) and intangible (e.g., reputation or intellectual property such as trademarks). At the organizational level, internal control objectives relate to the reliability of financial reporting, timely feedback on the achievement of operational or strategic goals, and compliance with laws and regulations. At the specific transaction level, internal control refers to the actions taken to achieve a specific objective (e.g., how to ensure the organization's payments to third parties are for valid services rendered.) Internal control procedures reduce process variation, leading to more predictable outcomes

Describing Internal Controls:

Internal controls may be described in terms of: a) the objective they pertain to; and b) the nature of the control activity itself.

Objective categorization

Internal control activities are designed to provide reasonable assurance that particular objectives are achieved, or related progress understood. The specific target used to determine whether a control is operating effectively is called the control objective. Control objectives fall under several detailed categories; in financial auditing, they relate to particular financial statement assertions,[5] but broader frameworks are helpful to also capture operational and compliance aspects:

1. Existence (Validity): Only valid or authorized transactions are processed (i.e., no invalid transactions) 2. Occurrence (Cutoff): Transactions occurred during the correct period or were processed timely. 3. Completeness: All transactions are processed that should be (i.e., no omissions) 4. Valuation: Transactions are calculated using an appropriate methodology or are computationally accurate. 5. Rights & Obligations: Assets represent the rights of the company, and liabilities its obligations, as of a given date. 6. Presentation & Disclosure (Classification): Components of financial statements (or other reporting) are properly

classified (by type or account) and described. 7. Reasonableness-transactions or results appear reasonable relative to other data or trends.

44

Page 45: MCS theory

Activity categorization

Control activities may also be described by the type or nature of activity. These include (but are not limited to):

Segregation of duties - separating authorization, custody, and record keeping roles to limit risk of fraud or error by one person.

Authorization of transactions - review of particular transactions by an appropriate person. Retention of records - maintaining documentation to substantiate transactions. Supervision or monitoring of operations - observation or review of ongoing operational activity. Physical safeguards - usage of cameras, locks, physical barriers, etc. to protect property. Analysis of results, periodic and regular operational reviews, metrics, and other key performance indicators (KPIs). IT Security - usage of passwords, access logs, etc. to ensure access restricted to authorized personnel. S

Q4 .Veena Pvt. Ltd., a small multiproduct company is taken over by a multinational company ( e.g. Hindustan Lever.) What changes in the control system would you expect and why?

Since Veena is a small multiproduct company it would require changes in control system which would be related to transfer pricing a, as this company would generally provide inputs to HUL. Thus the domestic operations generally involve transfer of goods and services only In view of this difference many other considerations, in addition to the criteria used in domestic operations for the determination of transfer price, are involved. These include:

(a) Fair Price: This is an important factor one needs to consider while determining the transfer price for foreign operations. Companies that enter into joint ventures must ensure that the transfer price charged is fair. If such companies charge a higher transfer price, it would reduce the profits of the joint venture and as a result reduce the foreign partner's share of profits.

(b) Government Regulations: All countries have a regulatory framework under which business units operate. Where government rules and regulations regarding transfer prices are lenient, the parent company should fix a higher transfer price for all transfers to countries with high income tax rates. This approach would enable the parent company to minimize taxes in such countries.

(c) Exchange Control Restrictions: Every country has foreign exchange control regulations. These regulations impose a limit on the amount of foreign exchange available for the import of certain goods. To accommodate the foreign subsidiary the parent company may have a lower transfer price so that the subsidiary is able to import a larger quantity of required goods.

(d) Income Tax Regulations: The rates of income tax vary from country to country. To overcome this difference the transfer price should be so fixed that countries with low tax rates show profits while others end up with a loss. This helps the parent company to reduce its taxes on a global basis.

(e) Desire to accumulate funds: A company that wishes to accumulate funds in a particular country may fix the transfer prices in such a manner that it facilitates shifting of funds into that country.

(I) Tariffs- and Duties: No country likes high imports. In order to restrict imports countries impose restrictions such as quantitative restrictions, high duties and tariffs and banning import of products. The general practice is to charge import duties as a percentage of the value of products imported, although a lower tariff may be levied if the import value is lower. It is seen that the impact of tariffs on the profitability of foreign operations is generally the reverse of the incidence of income taxes in transfer pricing. As such a low transfer price would lead to low import duties on transfer, the profit arising in that country would be high. This results in high income taxes in that country. It is therefore advisable that companies must compute the net effect of these factors while determining transfer prices.

In designing performance evaluation systems for acquired Veena company,HUL could use the following guidelines

Subsidiary managers should not be held responsible for translation effects. The simplest way to achieve this objective is to compare budgets and actual results using the same metric and isolate inflation-related effects through variance analysis. It is pointless for managers to worry about the appropriate metric. The MNE should choose whatever metric is more convenient.

Transaction effects are best handled through centralized coordination of the MNE's overall hedging needs. This is likely to be cheaper and simpler, and it prevents the subsidiary manager from becoming a foreign exchange rate forecaster and speculator.

The subsidiary manager should be held responsible for the dependence effects of exchange rates resulting from economic exposure.

Evaluation of the subsidiary as a basis for a decision to locate operations in a country or to relocate operations from a country should reflect the consequences of translation. Transaction and economic exposures.

45

Page 46: MCS theory

SET 10

Q.1) What are the Special Characteristics of Professional Service Organization?

Answer:

GoalsA dominant goal of a manufacturing company is to earn a satisfactory profit, specifically a satisfactory return on assets employed. A professional organization has relatively few tangible assets; its principal asset is the skill of its professional staff, which 'doesn't appear on its balance sheet. Return on assets employed, therefore, is essentially meaningless in such organizations. Their financial goal is to provide adequate compensation to the professionals.

In many organizations, a related goal is to increase their size. In part, this reflects the natural tendency to associate success with large size. In part, it reflects economies of scale in using the efforts of a central personnel staff and units responsible for keeping the organization up-to-date. Large public accounting firms need to have enough local offices to enable them to audit clients who have facilities located throughout the world.

Professionals Professional organizations are labor intensive, and the labor is of a special type. Many professionals prefer to work independently, rather than as part of a team. Professionals who are also managers tend to work only part time on management activities; senior partners in an accounting firm participate actively in audit engagements; senior partners in law firms have clients. Education for most professions does not include education in management, but quite naturally stresses the skills of the profession, rather than management; for this and other reasons, professionals tend to look down on managers.

Professionals tend to give inadequate weight to the financial implications of their decisions; they want to do the best job they can, regardless of its cost. This attitude affects the attitude of support staffs and nonprofessionals in the organization; it leads to inadequate cost control.

Output and Input MeasurementThe output of a professional organization cannot be measured in physical terms, such as units, tons, or gallons. We can measure the number of hours a lawyer spends on a case, out this is a measure of input, not output. Output is the effectiveness of the lawyer's work, and this is not measured by the number of pages in a brief or the number of hours in the courtroom. We can measure the number of patients a physician Teats in a day, and even classify these visits by type of complaint; but this is by no means equivalent to measuring the amount or quality of service the physician has provided. At most, what is measured is tl1e physician's efficiency in treating patients, which is of some use in identifying slackers and hard workers. Revenues earned is one measure of output in some professional organizations, but these monetary amounts, at most, relate to the quantity of services rendered, not to their quality (although poor quality is reflected in reduced revenues in the long run).

Furthermore, the work done by many professionals is non repetitive. No two consulting jobs or research and development projects are quite the same. This makes it difficult to plan the time required for a task, to set reasonable standards for task performance, and to judge how satisfactory the performance was. Some tasks are essentially repetitive: the drafting of simple wills, deeds, sales contracts, and similar documents; the taking of a physical inventory by an auditor; and certain medical and surgical procedures. The development of standards for such tasks may be worthwhile, although in using these standards, usual circumstances that affect a specific job must be taken into account. Some professionals, notably scientists, engineers, and professors, are reluctant to keep track of how they spend their time, and this complicates the task of measuring performance. This reluctance seems to have its roots in tradition; usually, it can be overcome if senior management is willing to

46

Page 47: MCS theory

put appropriate emphasis on the necessity of accurate time reporting. Nevertheless, difficult problems arise in deciding how time should be charged to clients.

If the normal work week is 40 hours, should a job be charged for 1140th of a week's compensation for each hour spent on it? If so, how should work done on evenings and weekends be counted? (Professionals are "exempt" employees-that is, they are not subject to government requirements of overtime payments.) How to account for time spent reading literature, going to meetings, and otherwise keeping up-to-date?

Small SizeWith a few exceptions, such as some law firms and accounting firms, professional organizations are relatively small and operate at a single location. Senior management in such organization can personally observe what is going on and personally motivate employees. Thus, there is less need for a sophisticated management control system, with profit centers and formal performance reports. Nevertheless, even a small organization needs a budget, a regular comparison of performance against budget, and a way of relating compensation to performance.

How is Marketing done in them?In a manufacturing company there is a clear dividing line between marketing activities and production activities; only senior management is concerned with both. Such a clean separation does not exist in most professional organizations. In some, such as law, medicine, and accounting, the profession's ethical code limits the amount and character of overt marketing efforts by professionals (although these restrictions have been relaxed in recent years). Marketing is an essential activity in almost all organizations, however. If it can't be conducted openly, it takes the form of personal contacts, speeches, articles, conversations on the golf course, and so on. These marketing activities are conducted by professionals, usually by professionals who spend much of their time in production work-that is, working for clients. In this situation, it is difficult to assign appropriate credit to the person responsible for "selling" a new customer. In a consulting firm, for example, a new engagement may result from a conversation between a member of the firm and an acquaintance in a company, or from the reputation of one of the professionals as an outgrowth of speeches or articles. Moreover, the professional who is responsible for obtaining the engagement may not be personally involved in carrying it out. Until fairly recently, these marketing contributions were rewarded subjectively-that is, they were taken into account in promotion and compensation decisions. Some organizations now give explicit credit, perhaps as a percentage of the project's revenue, if the person who "sold" the project can be identified.

How do we evaluate the Performance Appraisal? As noted earlier in regard to teachers, at the extremes the performance of professionals is easy to judge. Appraisal of the large percentage of professionals who are within the extremes is much more difficult. For some professions, objective measures of performance are sometimes unavailable: The recommendations of an investment analyst can be compared with actual market behavior of the securities; the accuracy of a surgeon's diagnosis can be verified by an examination of the tissue that was removed; and the doctors' skill can be measured by the success ratio of operations. These measures are, of course, subject to appropriate qualifications, and in most circumstances the assessment of performance is finally a matter of human judgment by superiors, peers, self, subordinates, and clients. Judgments made by superiors are the most common. For these, professional organizations increasingly use formal systems to collect performance appraisals as a basis for personnel decisions and for discussion with the professional. Some systems require numerical ratings of specified attributes of performance and provide for a weighted average of these ratings. Compensation may be tied, in part, to these numerical ratings. In a matrix organization, both the project leader and the head of the functional unit that is the professional's organizational "home" judge performance. "

Appraisals by a professional's peers, or by subordinates, are sometimes part of a formal control system. In some organizations, individuals may be asked to make a self-appraisal. Expressions of satisfaction or dissatisfaction from clients are also an important basis for judging performance, although such expressions may not always be readily forthcoming.

The budget can be used as the basis for measuring cost performance, and the actual time taken can be compared with the planned time. Budgeting and control of discretionary expenses are as important in a professional firm as in a manufacturing company.

Such financial measures are relatively unimportant in assessing a professional's contribution to the firm's, profitability, however. The professional's major contribution is related to quantity and above all quality of work, and its appraisal must be largely subjective. Furthermore, the appraisal must be made currently; it cannot wait until one learns whether a new building is well designed, a new control system actually works well, or a bond indenture has a flaw.

47

Page 48: MCS theory

In some professions, internal audit procedures are used to control quality. In many accounting firms, the report of an audit is reviewed by a partner other than the one who is responsible for it, and the work of the whole firm is "peer reviewed" by another firm. The proposed design of a building may be reviewed by architects who are not actively involved in the project.

Q.2) What is a Non - Profit Organization? How is the performance of this organization evaluated?

Answer:

IntroductionA nonprofit organization, as defined by law, is an organization that cannot distribute assets or income to, or for the benefit of, its members, officers, or directors. The organization can, of course, compensate its employees, including officers and members, for services rendered and for goods supplied. This definition does not prohibit an organization from earning a profit; it prohibits only the distribution of profits. A nonprofit organization needs to earn a modest profit, on average, to provide funds for working capital and for possible “rainy days.”

Performance evaluation of nonprofit organizationFor any organization, the most important reasons to measure performance are to improve effectiveness and to acquire information that will allow the organization to drive its agenda forward. If the motivation for doing evaluation remains outside an organization, the evaluation will have limited impact. To do performance assessment effectively, an organization must commit to adopting a culture of measurement, because acceptance must come from senior management, staff, funders, and board members alike.

Board self-evaluationMembers of the Board of Directors should regularly evaluate the quality of their activities on a regular basis. Activities might include staffing the Board with new members, developing the members into well-trained and resourced members, discussing and debating topics to make wise decisions, and supervising the CEO. Probably the biggest problem with Board self-evaluation is that it does not occur frequently enough. As a result, Board members have no clear impression of how they are performing as members of a governing Board. Poor Board operations, when undetected, can adversely affect the entire organization.

Staff and volunteer (individual) performance evaluationMost of us are familiar with employee performance appraisals, which evaluate the quality of an individual’s performance in their position in the organization. Ideally, those appraisals reference the individual’s written job description and performance goals to assess the quality of the individual’s progress toward achieving the desired results described in those documents. Continued problems in individual performance often are the results of poor strategic planning, program planning and staff development. If overall planning is not done effectively, individuals can experience continued frustration, stress and low morale, resulting in their poor overall performance. Experienced leaders have learned that continued problems in performance are not always the result of a poor work ethic – the recurring problems may be the result of larger, more systemic problems in the organizations.

Program evaluationProgram evaluations have become much more common, particularly because donors demand them to ensure that their investments are making a difference in their communities. Program evaluations are typically focused on the quality of the program’s process, goals or outcomes. An ineffective program evaluation process often is the result of poor program planning – programs should be designed so they can be evaluated. It can also be the result of improper training about evaluation. Sometimes, leaders do not realize that they have the responsibility to verify to the public that the nonprofit is indeed making a positive impact in the community. When program evaluations are not performed well, or at all, there is little feedback to the strategic and program planning activities. When strategic and program planning are done poorly, the entire organization is adversely effected.

Evaluation of cross-functional processes

48

Page 49: MCS theory

Cross-functional processes are those that span several systems, such as programs, functions and projects. Common examples of major processes include information technology systems and quality management of services. Because these cross-functional processes span so many areas of the organization, problems in these processes can be the result of any type of ineffective planning, development and operating activities.

Organizational evaluationOngoing evaluation of the entire organization is a major responsibility of all leaders in the organization. Leaders sometimes do not recognize the ongoing activities of management to actually include organizational evaluations – but they do. The activities of organizational evaluation occur every day. However, those evaluations usually are not done systematically. As a result, useful evaluation information is not provided to the strategic and program planning processes. Consequently, both processes can be ineffective because they do not focus on improving the quality of operations in the workplace.

Q.3) A Well Diversified company – Pritam International Ltd. sells one of its divisions to a group of its own company managers. Explain what significant changes in systems and control procedures can be expected? Why?

Answer:As, we Pritam International is a well diversified company. Sometimes, excessive diversification and that too in unrelated lines of business causes failure in the business operations. One of the major reason for failures of many Mergers and Diversification is excessive diversification. As, excessive diversification is ominous especially, in unrelated lines of business. As, there may be no advantage of operating synergy. Neither through:

I) Sharing common resources norII) Sharing common core competencies

Therefore, it may be a strategic decision by the promoters and directors of the company to sell one of its divisions. As, this may be impacting their core business. Sometimes, your core business tends to get neglected mainily due to excesive diversification. As, the division is being sold to its own company managers. There, might not be major changes in management control and systems. As, most of its managers will be the same. But, they will have more autonomy to take decisions independently after acquisition. Now there will be less red tapism and managers can take more risk. The managers will manage the firm in their own style. As, they are not answerable to their superiors.

SET 11

Q.1)Why Balance Score Card is considered superior to other methods of Performance Appraisal?

Prepare Balance Score Card for any organization you are familiar with.

What is the Balanced Scorecard?

The rationale for the development of the Balanced Scorecard was a growing dissatisfaction with traditional,

financial measures of performance. These measures suffer from a number of serious drawbacks in that they

take a short-term, lagged (i.e., historic) view of performance. The shift towards flexible, lean production/service

systems in many firms has strengthened the requirement for performance measurement systems to become

more broadly based, incorporating both non-financial and external measures of performance. According to

Kaplan and Norton, the Balanced Scorecard provides a better assessment of performance as it "enables

companies to track financial results while simultaneously monitoring progress in building the capabilities and

acquiring the intangible assets they need for future growth".

The original scorecard designed by Kaplan and Norton contained four key groupings of performance measures.

These four groupings, called ‘perspectives’ by Kaplan and Norton, were considered sufficient to track the key

drivers of both current and future financial performance of the firm. The perspectives focused on the

achievements of the firm in four areas: namely the financial, customer, internal business process and

innovation/learning perspectives. The four perspectives can be represented as an interlinked hierarchy. The

firm’s strategy underlies the whole scorecard, as the measures for each of the four perspectives are drawn from

this strategy.

49

Page 50: MCS theory

To obtain a satisfactory overview of performance, the scorecard will require a mix of lagging and leading

(forward looking) measures. Financial measures tend to be lagged and consequently, the measures chosen for

the other perspectives will need to include leading measures. In general, outcome measures tend to be lagged,

for example, current market share is the result of past decisions and consequently is a lagging measure. Thus

the challenge in designing a Balanced Scorecard is to choose driver measures which lead changes in the

outcome measures in the non-financial perspectives and which ultimately drive the financial measures.

Once the firm’s objectives have been agreed and the appropriate outcome and driver measures chosen for each of the perspectives, firm and managerial performance is assessed by comparing actual attainment on each measure with the target set for that measure.

Objective Measure Target Actual

Benefits from adopting the Balanced ScorecardThere are several benefits from implementing a Balanced Scorecard. Originally the Balanced Scorecard was seen as a useful tool for performance measurement. In this role, the Balanced Scorecard was seen as integrating financial/non-financial, internal/external and leading /lagging information on firm performance in a coherent fashion.

Later it was realised that the Balanced Scorecard could play a pivotal role in the strategic management process. Because the Balanced Scorecard requires management to clarify and obtain consensus on the strategic objectives of the firm, it can assist in the communication of the chosen strategy, consequently aligning the efforts both of individuals and of departments. In this role, there is a clear link between the Balanced Scorecard and management by objectives (MBO). Effective implementation of a Balanced Scorecard project will generally involve the development of a series of hierarchical (cascaded) scorecards. Given the overall corporate scorecard, supporting scorecards can be developed for each department within the firm. Within each department, a scorecard can be developed for each manager (or perhaps even for each individual member of staff) which links the objectives on each perspective for that manager back to the objectives for each perspective outlined in the scorecard for the department and finally, back to the objectives listed in the firm’s overall scorecard.

The Balanced Scorecard could be used to assist in corporate restructuring. In recent years, many firms have migrated away from a traditional hierarchical structure to a flatter, team-based organisational structure. The Balanced Scorecard can support such changes, as it can help clarify the objectives and the critical success factors for the newly formed teams.

50

Page 51: MCS theory

Apart from the communication and co-ordination roles of the Balanced Scorecard in strategic implementation, the Balanced Scorecard can be used to link strategy to specific critical success factors in the customer, internal business process and growth/learning perspectives. By setting both short and long-term targets for driver and outcome measures and by comparing actual attainment against target, feedback is obtained on how well the strategy is being implemented and on whether the strategy is working.

Building on the Balanced Scorecard’s use as a strategic management tool, it has been suggested that the Balanced Scorecard can play a role in the investment appraisal process(5). Traditional methods of investment appraisal such as discounted cash flow do not cope well with investments which generate indirect rather than direct financial returns. Examples of these include investments which enhance the future ‘flexibility’ of a firm or investments in the firm’s infrastructure, such as an enhanced management information system. The Balanced Scorecard can assist management’s investment appraisal decisions as it provides managers with a mechanism to incorporate the strategic aspects of the investment into the appraisal process. This could be achieved by using a weighting system developed from a firm’s Balanced Scorecard measures to evaluate new projects. An index score would be calculated for each investment opportunity and projects would then be ranked and selected based on this score.

Balance Score Card of Credit Card Company

Q .2)

51

Page 52: MCS theory

Soniya Company has two Divisions: A & B. Return on Investment for both divisions is 20%. Details are given below:-

Particulars Div A Div B

Divisional sales 4000000 9600000

Divisional Investment 2000000 3200000

Profit 400000 640000

Analyse and comment on divisional performance of each.

ANSWER

As Profit Margin = Profit *100 Sales

Profit Margin for Division ‘A’= 4,00,000 /40,00,000 *100 = 10%

Profit Margin for Division ‘B’ = 6,40,000/ 96,00,000 *100 = 6.6%

Turnover of Investment = Sales * 100

Investment

Turnover of Investment for Division ‘A’ = 40,00,000/20,00,000 = 2 times

Turnover of Investment for Division ‘B’ = 96,00,000/32,00,000 = 3 times

As Return on investment for both Divisions A and B is 20%.

COMMENTS:-

Division ‘A’ – Although ‘A’ has more profit margin than Division ‘B’ that is 10% as compared to 6.6% of ‘B’, so it has more profitability but inspite of it, division ‘A’ has lower turnover of investment that its assets management is bad than Division ‘B’, it can be improved by increased sales or reducing investment.

Division ‘B’ – Needs to improve profit margin by increasing sales and reduce variable cost and sales at same price or by reducing salesprice and increase the volume of sales so that its profit would improve. As it has good assets management shown by its turnoverof Division ‘B’ that is 3 times which is better than Division ‘A’. So it can become profitable organisation by improving Profit Margin.

Q.4)Discuss and illustrate differences and similarities between

- Strategy Formulation and Management Control

52

Page 53: MCS theory

- Management Control and Task Control

Some Distinction between Strategy Formulation and management Control

Characteristics Strategy Formulation Management Control

a) Focus of plan On one aspect at a time On entire organisation

b) Complexities Many variables hence complex Less complex

c) Nature of information Tailor-made for the issue, more external and predictive, less accurate.

Integrated, more internal and historical, more accurate.

d) Structure Unstructured and irregular, each problem being different

Rhythmic, definite pattern, set procedure

e) Communication of information Relatively simple Relatively difficult

f) Purpose of estimates Show expected results Lead to desired result

g) Persons involved Staff and top management Line and top management

h) No. of persons involved Small Large

i) Mental activity Creative, analytical Administrative, persuasive

j) Planning and control Planning dominant but some control

Emphasis on both planning and control

k) Time horizon Tends to be long Tends to be short

l) End result Policies and precedents Action within policies laid

m) Appraisal of job done Extremely difficult Less difficult

b) Some Distinction between Management Control and Task Control

Characteristics Task Control Management control

a) Focus of plan Single task or transaction On entire organisation

b) Nature of information Tailor-made to operation, specific, often non- financial, real time

Integrated, more internal and historical, more accurate

c) Persons involved Supervisors Line and top management

d) Mental activity Follow directives or none as in case of machines or set objectives

Administrative, persuasive

e) Time horizon Day to day Tends to be short

SET 12

53

Page 54: MCS theory

Q.2 Suresh Ltd. (Numerical) (MCS-2007)

(a) Define profit in this case and prepare a statement for both divisions and overall company.

Solution:

i) Profitability statement of Division A:-

Particulars Amount(Rs.)Selling price p.u. 35Variable Cost p.u. 11Contribution p.u. 24

Contribution p.u. Expected sales (no. of units)

Total contribution Total Fixed cost (Rs.)

Net profit (Rs.)

24 2000 48000 60000 (12000)24 3000 72000 60000 1200024 6000 144000 60000 84000

ii) Profitability statement of Division B:-

Selling p.u. Total variable cost p.u.

Contribution p.u.

Expected sales (no. of units)

Total contribution

Total Fixed cost (Rs.)

Net profit (Rs.)

90 42 48 2000 96000 90000 600080 42 38 3000 114000 90000 2400050 42 8 6000 48000 90000 (42000)

[Note: Total Variable cost p.u. = Variable cost p.u. (Rs.7) + Transfer price of intermediate product (Rs.35)]

iii) Profitability statement of Company as a whole:-

Expected sales Net profit of division A (Rs.)

Net profit of Division B (Rs.)

Total Net profit

2000 (12000) 6000 (6000)3000 12000 24000 360006000 84000 (42000) 42000

(b) State the selling price which maximizes profits for division B and company as a whole. Comment on why the latter price is unlikely to be selected by division B.

Solution:

As per the calculation in part (a), selling price p.u. of Rs.80 maximizes profit for division B whereas selling price p.u. of Rs.50 maximizes profit for the Company as a whole. However, if Division B opts for selling price p.u. of Rs.50 in order to maximize Company’s profit, it would suffer a loss of Rs.42000. Therefore, Division B would not select Selling price p.u. of Rs.50.

Q.3 Explain different organizational goals. Comment on goal of shareholder value maximization in particular.

Goals

54

Page 55: MCS theory

Although we often refer to the goals of a corporation, a corporation does not have goals; it is an artificial being with no mind or decision-making ability of its own. Corporate goals are determined by the chief executive officer (CEO) of the corporation, with the advice of other members of senior management, and they are usually ratified by the board of directors. In many corporations, the goals originally set by the founder persist for generations. Examples are Henry Ford, Ford Motor Company; Alfred P. Sloan, General Motors Corporation; Walt Disney, Walt Disney Company; George Eastman, Eastman Kodak; and Sam Walton, Wal-Mart.

Economic GoalsShareholder's value, Earning per share and Market value, all relate to maximizing shareholder's value, which is not a desirable goal, because what is 'maximum' is difficult to determine. Although optimizing shareholder value may be one goal, but there are other stakeholders in the business also such as customers, employees, creditors, community and so on. Again, shareholder value is usually equated with the market value of the company's stock. But market value is not an accurate measure of the worth of shareholders' investments. Besides, such value can be obtained only when the share is traded in the stock exchange.

It is interesting to note that Henry Ford's operating philosophy was 'satisfactory profit', not 'maximum profit'. He said, "A reasonable profit is right, but not too much. So, it has been my policy to force the price of the car down as fast as production would permit and give the benefit to the user and laborers, with resulting surprisingly enormous benefit to ourselves"

Other goals such as adding new products, or product-line or new business actually indicate normal organizational growth.Social GoalsHowever, every organization has its share of responsibility towards the local community where it is situated, and the public at large. It is very difficult to incorporate in Management Control System such goals as taking pride in an organization which cares for the society and renders service to the public. Of course, any concrete structural programme indicating its operational expenses, methods of providing service, personnel involved in rendering service and the nature of the service in details can, however, be mentioned through an appropriate system.

Profitability

In a business, profitability is usually the most important goal.

Return on investment can be found by simply dividing profit (i.e., revenues minus expenses) by investment, but this method does not draw attention to the two principal components: profit margin and investment turnover.

In the basic form of this equation, "investment" refers to the shareholders' investment, which consists of proceeds from the issuance of stock, plus retained earnings.

One of management's responsibilities is to arrive at the right balance between the two main sources of fi-nancing: debt and equity. The shareholders' investment (i.e., equity) is the amount of financing that was not obtained by debt, that is, by borrowing. For many purposes, the source of financing is not relevant; "investment" thus means the total of debt capital and equity capital.

"Profitability" refers to profits in the long run, rather than in the current quarter or year. Many current expenditure (e.g., amounts spent on advertising or research and development) reduce current profits but increase profits over time.

Some CEOs stress only part of the profitability equation. Jack Welch, former CEO of General Electric Company, explicitly focused on revenue; he stated that General Electric should not be in any business in which its sales revenues were not the largest or the second largest of any company in that business. This does not imply that Welch neglected the other componentsof the equation; rather, it suggests that in his mind there was a close correlation between market share and return on investment.

Other CEOs, however, emphasize revenues for a different reason: For them, company size is a goal. Such a priority can lead to problems. If expenses are too high, the profit margin will not give shareholders a good return on their investment. Even if the profit margin is satisfactory, the organization may still not earn a good return if the investment is too large.

Maximizing Shareholder ValueIn the 1980s and 1990s the term shareholder value appeared frequently in the business literature. This concept is that the appropriate goal of a for-profit corporation is to maximize shareholder value. Although the meaning of

55

Page 56: MCS theory

this term was not always clear, it probably refers to the market price of the corporation's stock. We believe, however, that achieving satisfactory profit is a better way of stating a corporation's goal, for two reasons.

First, "maximizing" implies that there is a way of finding the maximum amount that a company can earn. This is not the case. In deciding between two courses of action, management usually selects the one it believes will increase profitability the most. But management rarely, if ever, identifies all the possible alternatives and their respective effects on profitability. Furthermore, profit maximization requires that marginal costs and a demand curve be calculated, and managers usually do not know what these are. If maximization were the goal, managers would spend every working hour (and many sleepless nights) thinking about endless alternatives for increasing profitability; life is generally considered to be too short to warrant such an effort.

Second, although optimizing shareholder value may be a major goal, it is by no means the only goal for most organizations. Certainly a business that does not earn a profit at least equal to its cost of capital is not doing its job; unless it does so, it cannot discharge any other responsibilities. But economic performance is not the sole responsibility of a business, nor is shareholder value. Most managers want to behave ethically, and most feel an obligation to other stakeholders in the organization in addition to shareholders.

Example: Henry Ford's operating philosophy was satisfactory profit, not maximum profit. He wrote let me say right here that I do not believe that we should make such an awful profit on our cars. A reasonable profit is right, but not too much. So it has been my policy to force the price of the car down as fast as production would permit, and give the benefits to the users and laborers-with resulting surprisingly enormous benefits to ourselves.By rejecting the maximization concept, we do not mean to question the validity of certain obvious principles. A

course of action that decreases expenses without affecting another element, such as market share, is sound. So is a course of action that increases expenses with a greater than proportional increase in revenues, such as expanding the advertising budget. So, too, are actions that increase profit with a less than proportional increase in shareholder investment (or, of course, with no such increase at all), such as purchasing a cost-saving machine. These principles assume, in all cases, that the course of action is ethical and consistent with the cor-poration's other goals.

An organization's pursuit of profitability is affected by management's willingness to take risks. The degree of

risk-taking varies with the personalities of individual managers. Nevertheless there is always an upper limit;

some organizations explicitly state that management's primary responsibility is to preserve the company's

assets, with profitability considered a secondary goal. The Asian .financial crisis during 1996-1998 is traceable,

in large part, to the fact that banks in Asia's emerging markets made what appeared to be highly profitable

loans without paying adequate attention to the level of risk involved.

Multiple Stakeholder ApproachOrganizations participate in three markets: the capital market, the product market, and the factor market. A firm raises funds in the capital market, and the public stockholders are therefore an important constituency. The firm sells its goods and services in the product market, and customers form a key constituency. It competes for resources such as human capital and raw materials in the factor market and the prime constituencies are the company's employees and suppliers and the various communities in which the resources and the company's operations are located.

The firm has a responsibility to all these multiple stakeholders-shareholders, customers, employees, suppliers, and communities. Ideally, its management control system should identify the goals for each of these groups and develop scorecards to track performance.

Example: In 2005, the Acer Group, headquartered in Taiwan, was one of the largest computer companies The Company subscribed to the multiple stakeholder approach and managed its internal operations to satisfy the needs of several constituencies. To quote Stan 'Shih,-the founder, "The customer is number 1, the employee is number 2, the shareholder is number 3. I keep this message consistent with all my colleagues. I even consider the company's banks, suppliers, and others we do business with are our stakeholders; even society is stakeholder. I do my best to run the company that way." Lincoln Electric Company is well known for its philosophy that employee satisfaction was more important than

shareholder value. James Lincoln wrote: "The last group to be considered is the stockholders who own stock because they think it will be more profitable than investing more in any other way. The absentee stockholder is not' of any value to the customer or to the worker, since he has no knowledge of nor interest in the company other than greater dividends and advance in the price of his stock." Donald F. Hastings, chairman and chief executive officer, emphasized that this was still the company's philosophy in 1996.

56

Page 57: MCS theory

Q.4 Explain and illustrate with one example differences between 3 forms of internal audit-

Financial, Operational & Management.

Financial Audit-

Financial Audit is a historically oriented, independent evaluation performed by

internal auditor or external auditor for the purpose of attesting to the fairness, accuracy and reliability of the

financial data, providing protection for the entity's assets; evaluating the adequacy and accomplishment of the

system (internal control) designed,

provide for the aforementioned Fairness and Protection, Financial data, while not being the only source of

evidence, are the primary evidential source. The evaluation is

performed on a planned basis rather than a request".

Institute of Internal Auditor:-

Financial audit takes care of the protective aspect of the business and it does not

normally carry out constructive appraisal function of the business operations. It helps in detection and

prevention of fraud. It also verifies whether documentation and flow of activities arc in conformity with the

internal control system introduced and developed within the organization. It helps coordinating with statutory

auditor to help them in proper discharge of their function. Besides, financial audit also ensures compliance with

statutory laws especially in financial and accounting matters.

Objectives of Financial Audit:

-To see that established accounting systems and procedures have been complied with

-To see that proper records have been maintained for the fixed assets of the Concern to look into correctness

of the financial data and records along with correctness of the accounting procedure followed.

-To see whether scrap, salvage and surplus materials have been properly accounted for etc.

-To see that internal control system has been working properly.

-To see that any abrupt variation in sales, purchases etc.; with respect to immediate previous year are not due

to any irregularity

-To see that the credit control has been strictly followed.

-To see that all payments have been made with proper authorization and approval. .

-To see that preparation of salary and wage pay roll has been properly done.

budgetary control system, if any scope and performance of internal audit, if any, suggestions for improvements

in performance, if any, and improved inventory policies.

The opinion expressed by the auditors shall be based on verified data, reference to ich shall also be made here

and, if practicable, included after the company has been

forded on opportunity to comment on them.

Management Audit

It is a complex task closely related with the process of management. It is highly result oriented. It requires

inter/multi-disciplinary approach as it involves examination, review and appraisal of various policies and actions

of management on the basis of certain norms/standards.

It undertakes comprehensive and critical review of all organizational activities with wider perspective.

It goes beyond conventional audit and audits the efficacy of the management itself.Definition:

It's a comprehensive and constructive examination of an organization, the structure of a company,

institution or branch of government or of any components thereof, such as division or department and its plans,

objectives, its means of operations and its use of human and physical facilities.

William P. Leonard

It's an investigation of a business from the higher level downwards in order to ascertain whether sound

57

Page 58: MCS theory

management prevails throughout, thus facilitating the most effective relationship with the outside world and

the most efficient and smooth running internally.

Leslie Howard

It is an audit performed with the object of examining the efficacy of the institution/control systems,

management procedures towards the achievement of enterprise goals.

Churchill & Cyert

It is an objective and independent appraisal of the effectiveness of managers and the effectiveness of the

corporate structure in the achievement of company objectives and policies. Its aim is to identify existing and

potential management weaknesses within an organization and to recommend ways to rectify these

weaknesses.

Chartered Institute of Management Accountants London

Thus it can be seen that management audit is an examination, review and appraisal of the various policies

and actions of the management. It is a tool for the evaluation of methods and performance in all the areas of

the enterprise.

Objectives:

1. To ascertain the provision of proper control at different levels, their effectiveness I in accomplishing

management goals.

2. Ascertain objectives of the organization are properly communicated and understood at all levels.

3. To reveal defects or irregularities in any of the elements examined and to indicate what improvements

are possible to obtain the best results of the operations of the company.

4. To assist the management to achieve the most efficient administration of its operations.

5. To suggest to the management the ways and means to achieve the objectives if the management of the

organization itself lacks the knowledge of efficient management.

6. It aims to achieve the efficiency of management and assess the strength and weaknesses of the organization

structure, its management team and its corporate culture.

7. To ascertain the provision of proper control at different levels, their effectiveness in

accomplishing management goals.

8. Ascertain objectives of the organization are properly communicated and understood at all levels.

9. To reveal defects or irregularities in any of the elements examined and to indicate what improvements are

possible to obtain the best results of the operations of the company.

10. To assist the management to achieve the most efficient administration of its operations.

11. To suggest to the management the ways and means to achieve the objectives if the

management of the organization itself lacks the knowledge of efficient management.

12. It aims to achieve the efficiency of management and assess the strength and weaknesses of the

organization structure, its management team and its corporate culture.

13. To help the management at all levels in the effective and efficient discharge of their duties and

responsibilities.

The auditor must apprise managerial performance at all levels of the organization. The audit starts right at

the top level of the management. It studies the managerial performance at all the levels of management. The

audit has to study the decision-making system of the organization and also the level of autonomy granted to the

managers at different levels of the organization. The authority and responsibility given at the different levels of

the management. One of the most important things that the audit must study is that the mangers at various

levels use the authority.

Conducting Management Audit

Management audit requires an interdisciplinary approach since it involves a review of all aspects of

58

Page 59: MCS theory

management functions. It has to be conducted by a team of experts because this requires 3 varieties of skills,

which one individual may not possess.

The team may consist of management experts, accountants, and the operation research specialists, the

industry experts and even social scientists.

The auditors must have analytical mind and ability to look at a management function form the point of view

of the organization as a whole. They therefore have to be properly trained in this aspect. They need to have

through knowledge of the management science and they should be acquainted with the salient features of

various functional areas.

Under financial audit, the entire emphasis is on macro-aspect, the individual transactions being- scrutinized for

check of the aggregates. It is concerned with examination of transactions recorded in the books of account. It

reviews the procedure and internal checks, and scrutinizes individual transactions for the purpose of verification,

of Profit and Loss Account and Balance Sheet. Financial audit is not concerned with ~ avoidance of profiteering

motive. It indicates the financial position and over~ performance of the business, regardless of its performance

in various segments. Financial audit is applicable to all classes of companies and industries irrespective of size

and Dan of operations.

Instead of serving the interest of the management and the Government, it serves interest of shareholders.

Financial audit is organization - oriented. It is conducted under Sections 224 - 232 of the Companies Act 1956.

Financial Audit Management Audit

It is concerned with financial aspects of

business transactions of the year under

audit

It is concerned with the review of the past Performance to ascertain whether it is in tune with the objectives, policies and procedures of the enterprise.

The auditor examines the past financial

records to report his opinion on the truth

and fairness of the representations made in the

financial statements. Examination of the

performance of the management is

beyond his scope

The management auditor reports on performance of the management during a particular period and suggest ways to remedy the deficiencies, including modification of objectives, policies etc.

Past year '(Financial) transactions areCovered Enterprises such as companies, trust and societies etc.

No limit as to the period to be covered

Financial audit is compulsory in the case of certain enterprises such as companies, trust and societies etc.

There is legal compulsion as regards management audit.

The auditor reports to the owner, i.e.shareholders in the Case of a company

The auditor reports to the management

Q.5 Explain briefly various stages of management control process citing salient features of each.Management control process involves communication of information to the managers at various levels of hierarchy and their interactions arising out of them. These communications aim towards attaining the

59

Page 60: MCS theory

organization's goals. But individual managers have their personal goals also. For example, a young manager with good education, experience, personality and social background joins a company like Britannia Industries or Reliance. The company finds him fit for the position as per job specifications, appoints him and makes him aware of what the company expects of him. The young manager sets his goals of gaining rich experience for his career progress besides adequate compensation packages. Naturally, his actions will be directed towards achieving his own objectives and goals while serving the company. Thus, his self-interest and the best interest of the organization are apparently in conflict. But the best results can be achieved by perfectly matching the two interests and this is called 'goal congruence'.

It is quite apparent that perfect congruence between the goals of the individual and the organization individual's goals and the organization's goals can never happen. Yet, the main purpose of a management control system is to assure goal congruence between the interest of the individual and the organization as far as practicable.

Management control systemsFormal and Informal CommunicationAs mentioned earlier, all the communication of information may be either formal or informal. The formal communication system involves strategic plan, budgets, standards and reports whereas the informal communication is made through letters and memos, verbally or even by facial expression.Formal communications are all documented and addressed to the responsible managers for their information and actions, if necessary. However, the actions depend on the perception of the individual managers.Informal communication, on the other hand, relates to some external factors-work ethics, management style and culture. Added to these factors is the existence of an informal organization within the structured formal organization.

Informality refers to the relaxation of sharp differentiation and explicit description of behavior as indicated in the hierarchy and thereby, moving away from superior/subordinate relationship. However, such relations depend on the personal capabilities of the manager such as education, experience, expertise, trust and cooperation. For example, Accounts Manager of Nasik Plant (see the organization chart in the diagram 3.2) reports to the General Manager of the Plant. While visiting the Corporate Office for attending a Training Course, he meets other colleagues, parallel officers and even the Finance Director. The latter communicates some important matter to him verbally and wants action thereon. Accounts Manager carried out the instructions so given. As per the organization chart, he should inform his General Manager, but it depends on his own perception of the situation, and he mayor may not report to the General Manager.

Work Ethics, Management Style and CultureExternal factors like work ethics vary from place to place. Therefore, organization work culture depends on the general behavior of the people in the society where the organization situates. Work culture generally differs because of the life style and the attitude towards the work. For example, people of Mumbai lead very fast life. Time has more value at Mumbai as compared to Kolkata, where people take things easily and leisurely. Japanese and Korean people have reputation for their excellent work culture.However, the most important internal factor is the organization's culture and climate. The culture refers to the set of common beliefs, attitudes, norms, relationships and assumptions that are explicitly or implicitly accepted and evidenced throughout the organization. The writer joined Union Carbide as an Assistant just three days before Christmas Eve. On the very second day, when he attended Christmas lunch, his table was shared by none other than the General Sales Manager Dr. W.R. Correa. He kept us amused with various stories of his recent tour abroad and recited Urdu 'shairies', even sharing jokes. Such a situation was unthinkable in Jessop & Co., where sharp differences were maintained at every level of hierarchy.

Management control systems

Climate is used to designate the quality of the internal environment that conditions the quality of cooperation, the development of individuals, the extent of members' dedication or commitment to organizational purpose and the efficiency with which that purpose is translated into results. Climate is the atmosphere in which individuals work help, judge, and reward, constrain and find out about each other. It influences moral-the attitude of the individual towards his/her work and environment.

Culture differs between the organizations, but cultural norms are extremely important. They are not written like formal communication. But the existence of a good culture can be felt from the behavior of the members of the organization. Once the writer landed up with his family at Hyderabad in the early morning to discover that nobody had come to receive them at the station. His visit was arranged through non other than the Director of the company himself. His unit being new, telephone directory did not include any number of his unit, but the

60

Page 61: MCS theory

parent organization's telephone number was located. When an executive of the parent company was contacted, he immediately sent an officer of the company with a car to pick us up to their Guest House, entertain with coffee and then put up in a Hotel. What subsequently happened is a different matter, but the attitude and treatment of that member of organization speak volumes about their excellent culture.

In any organization, the culture remains unchanged as long as the Chief Executive remains in position. When a new executive replaces him, there is likelihood of some change in the culture, unless the new Chief follows the footsteps of his predecessor and maintains it. Generally, if higher positions are filled in through promotion of internal executives, the culture remains unchanged and the traditions are maintained.

The other important internal factor which influences management control system is management style-that is the attitude of the superior to his subordinates and the latter's reaction through their perception of the attitude of their superiors. Again, the attitude ultimately stems from the temperament of the Chief Executive, who controls the entire organization. That is why R. W. Emerson said "an institute is the lengthened shadow of a man".

Importance of Informal CommunicationAn organization indulges in informal control process when encountering non-routine decision-making or when seeking new information to increase understanding of some problem areas. During a very critical period in an organization, the writer found that the Chief Executive used to call managers informally at his residence or club to extract information in a relaxed manner rather than in a tense situation prevailing in the factory.

Formal Control Process

Formal communication system is structured as per the 'hierarchy outlined in the organization chart. The system has the following four components:

(a) Strategic plan and programme(b) Budgeting

(c) Operations and measurement in responsibility centers (d) Reporting

(a) Strategic Plan and Programme

The foundation of management control process lies in the organization's goals and its strategies for attaining these goals. A strategic plan is prepared in order to implement the strategies, after carefully considering opportunities and threats in the external environment as well as the strengths and weaknesses in the internal environment. Thus, a strategic plan and programme is prepared as a guideline to budgeting.

(b) Budgeting

The strategic plan is converted to an annual budget incorporating planned expenditure and revenues for individual responsibility centers. Expenses and revenues are marked for each responsibility centre period wise, say monthly, quarterly, half yearly, and annually.

(c) Operations and MeasurementResponsibility centers operate within the framework of the budget, established standards, standing instructions, practices and operating procedures embodied in 'rules', and 'manuals'. Thus, besides budget, the responsibility centers are also guided by a large number of rules. They record the resources actually used and revenue earned. They also classify the data by programmes as well as by responsibility centers for performance measurement.

(d) ReportingActual performance is analyzed, measured and reported against plan, indicating variances and highlighting areas of weaknesses. If the performance is satisfactory, feedback information is sent to the responsibility centre concerned for praise or reward. If the same is unsatisfactory feedback communication is sent to the responsibility centre concerned for corrective action. If such action requires to be included in the budget, then the latter is revised to give effect to the changed position. If required, then the plan itself can be revised and a new basis of control may be established.

The aforesaid formal control process has been presented in the following diagram:

61

Page 62: MCS theory

STUDY CASE: WESTPORT ELECTRIC CORPORATION

HIGHLIGHT OF CASE STUDY

Organization of Westport

Westport Electric is U.S based Giant Corporation manufactures and sells electric and electronic product. The operating activities are divided in four groups they are: the electrical Generating and Transmission

Group, the Home Appliance Group, the military and Space group, and the Electronics group. Each divisions has profit centers At the corporate level there is research and development staff and six administrative staff such as Finance,

industrial relation, legal, marketing, manufacturing and public relation. Responsibility are divided in to three categories

o Top management adviceo Advice to operating division and other staff office.o Coordinating among the division.

The Budgeting Organization

Kelly is in charge of budgeting department. Reporting to Kelly is king, how is in charge of administrative staff budget section.

Approval procedure1. Information submitted: budgeting department issue instructions & time tables for preparation,

submission and approval of the budget for the coming year. 2. Budget by expense classification: This shows proposed budget, last year budget and current year

budget classification. 3. Budget by activity: The same information as Budget by expense classification except this

information is classified by organization component. The main purpose is to show increase and decrease activity and information about new activity.

4. Explanation of changes: This schedule explains the reasons for propose budget. They are divided into following category economic changes increase and decrease in existing activity, new addition and old activities dropped.

5. Presentation of budget proposal: The final presentation is approved by the president and executive vice president. Ones the budget is approved than budgeted activities are under taken for the coming year.

62

Page 63: MCS theory

Functions of the budgeting department- to prescribe the schedules to be submitted and timetable for their submission and to “keep the presentation honest. The budgeting department has to analyze the proposed budgets and make sure that the facts are correctly stated.

PROBLEM STATEMENT      The budgeting system of Westport should be assessed as to its efficiency and effectiveness and identify changes that will develop the current system.

Analysis of the problems stated in the case:

Legal Department: The legal staff has asked for 30 % increase in their budget to take care of the added cost resulting from the expansion of their work load. But as Mr.King has justified that the increase in budget is quite high aster compared the quality of the work done by the staff of legal department. The simple contract handled by outside lawyer in 2 to 3 days takes almost 3 months for legal staff to get approved, so already time period for work done is more and fees they are charging is quite high compare to work.

Industrial relation: Industrial relation is proposing to increase training budget by nearly 50 percent as they are increasing trainings for management executives. The training program taken by department is good for the company as they can evaluate the executives which help in performance appraisal and even increase knowledge of the management executive. But the problem is the quality of training program as said by Mr. King the course taken in quality control was so easy that the secretary without attending the course got 100% in test and even she doesn’t know what quality control is.Again the training program in division was worst, they had to teach about economics in three lessons and they showed film of “Doc Dollar” explaining to “Jim Foremen” about money markets, capitalism and so forth instead of covering basic in economics which would have been more useful.

The process of approving budget is quite not reasonable as the vice president of staff do not have the power to approve the budget. He has to make presentations to executive vice president and president and after long discussion and questions budget is approved. Again the procedure is quite time consuming as the vice-president of staff and his executives prepare budget and they have to get approval from president and if changes has to be made in budget it needed to be modified. The person approving the budget has little or no knowledge and time to evaluate the staff as said by Mr. king, as in the case of legal staff , president get super deluxe treatment so that he get convinced and approve the budget. Added by Mr. King that “none of us are lawyer we have a hard time proving inefficiency, but we know it is there”

What should Westport Electric do about the evaluation problems raised in the case?

Among other things, the group recommends that top management actually pay considerable attention to the efficiency and effectiveness of each business division.

Currently, it appears that focus is given simply on bottom line numbers; that is, each unit’s financial success is assessed solely on the basis of how handsome the profits brought in for the company, without being given much performance evaluation as is needed in any organization.

Being distinct profit centers, both revenues and costs must be calculated for each business segment.

It is important to note that while the individual divisions may report the most exorbitant of profit figures, the numbers do not carry

with them as much meaning as when these are put into context.

The problem stated are with expense center but the revenue center is good as it is generating profit and sales are good

There is lack of co-ordination among the each manager’s levels which needs to be maintained.

Efficiency and effectiveness are the two main criteria by which the performance of a responsibility centre is judged, this

terms are almost always used in a comparative, rather than an absolute.

Efficiency is the ratio of outputs to inputs, or the amount of output per unit of input, so every staff should be efficient to

achieve the overall goal of the company and in this case both legal and industrial relation staff are less efficient than

other staff so before increase in budget they should be efficient in their work

63

Page 64: MCS theory

Effectiveness is determined by the relationship between a responsibility center output and its objectives, the more this objective contribute to the output the more effective the unit. So Westport Electric Corporation should specify clear objectives to every staff and division so that they can achieve it and be more effective.

The budget of legal staff should be on the basis of work or number of contracts, means it should not be incremental budget as it is assumed, it should be flexible or zero-based budgeting. The same for the industrial relation staff and all other depending of the work as the expenses are engineered or discretionary.

It is assumed that the decision for approving budget is centralized as only president has the power to approve or reject the budget, instead it should be decentralized and vice-president of staff should have the power to approve the budget as he has knowledge about the work and he should discuss with president in final approval of budget which will save time.

As the case says, that the person who approves the budget must have the knowledge about the functioning so that it may lead the less expanse for the company or if don’t have knowledge may lead to increase the expanse.

All divisions are profit centers and are interested in having their operation best and lack similarity with the earning for the corporation

CASE STUDY General Electric B Company 64

Introduction

The General Electric Company is a large multilocation corporation engaged in the manufacture and marketing of a wide variety of electrical and applied products. In 1964 there were almost 400 separate product lines and over three million catalog items. Sales volume in that year totaled $ 4941 million and net income was $ 237 million. Total employment was about 262000.

Early in 1950s General Electric initiated an extensive decentralization of authority and responsibility for operations of company. The basic unit of organization became product department. The company recognized that if this decentralization was to be fully effective it would need an improved system of management control. It also recognized that any improved system of control would require better measures of performance. To meet this need, company established a measurements project and created a special organizational unit to carry out this project.

The Measurement Project

The measurement project was established in 1952. Responsibility for project was assigned to accounting services,

one of corporate functional services divisions. A permanent organizational unit initially called measurement service

was set up to carry out this project.

An early step in measurement project was development of set of principles by which project was to be governed.

Five such principles were formulated as follows-

1. Measurements were to be designed to measure performance of organizational components, rather

than managers.

2. Measurements were to involve common indexes of performance, but not common standards of

performance. For example Rate of Return on investment might be index of performance common to all

product departments but standard in terms of this index might be 12% for one department and 25% for

another department.

3. Measurements were to be designed as aids to judgment in appraisal of performance and not supplant

judgment.

4. Measurements were to give proper weight to future performance as well as current performance in

order to facilitate maintenance of balance between long run and short run.

5. Measurements were to be selected so as to facilitate constructive action, not to restrict such action.

64

Page 65: MCS theory

The overall measurement project was divided into three major subprojects-

1. Operational measurements of results of product department.2. Functional measurements of work of engineering, manufacturing, marketing and finance, employee and plant

community relations, and legal components of organization.3. Measurements of work of managing as such planning, organizing, integrating, and measuring itself.

The first step in subproject on operational measurements was to develop an answer to following question

What are specific areas for which measurements should be designed, bearing in mind that sound measurements of overall performance require a proper balance among various functions and among aspects of managing?

In seeking an answer to this question organization made careful analysis of nature and purposes of basic kinds of work performed by each functional unit with purpose of singling out those functional objectives that were of sufficient importance to welfare of business to be termed as “Key result areas”.

Q.1 For the purpose described, how should profitability be defined? The definition should be specific enough so that a quantitative measure can be constructed from it?

Answer- ProfitabilityThe amount of profit, or the excess of revenue over expense during any given fiscal period. Profit will be reflected on the balance sheet as an increase in a nonprofit's net assets, offset by an increase in some asset or mix of assets. performance dimension based on the financial performance of channel members in terms of Return On Investment, liquidity, leverage, growth in sales and profits, etc. The purpose is to prioritize segments of the market to improve marketing profitability and to provide a means to choose the most appropriate communication media and messages for each unique market segment.

segmentation variable includes Age , Income , Social Class, price, Product Usage,- Lifestyle - The company could look at the activities and interests of the consumers.- Benefit - Segment users by benefit. That is benefit of speed, or graphics or ease of use.

Strategic Profit Model a tool used to assess a firm's profitability; return on equity is calculated by multiplying the net profit margin by the asset turnover to obtain the return on assets which, in turn, is multiplied by the financial leverage.

Return on Equity - a measure of a firm's profitability; profit achieved in a given period is expressed as a percentage of the total amount invested in the firm by its owners.

Asset Turnover - a ratio used to evaluate the profitability of a firm; net sales in a given period are divided by total assets.

Financial Leverage a measure of the extent to which a firm uses debt in its total capital structure; the higher the component of debt the more leveraged is the firm. Leverage is calculated by dividing total assets by equity.

Profitability Control - marketing effort intended to assess the level of profitability of each product in the portfolio, of each market segment, of each marketing channel, etc.

Breakeven analysis: shows the relationship between total costs and total revenue in order to assess the profitability of different levels of sales volume.

Financial institutions, such as brokerage firms and other financial consulting firms, are also in the business of producing original research. Financial firm assist investors by offering research report presenting financial firm’s take on an industry.

Key result areas are as follows-

1. Profitability2. Market position3. Productivity4. Product leadership5. Personnel development6. Employee attitudes7. Public responsibility8. Balance between short-range and long-range goals.

65

Page 66: MCS theory

Profitability

The key index used by General Electric to measure profitability was “dollars of residual income”. Residual income was defined as net profit after taxes less a capital charge. The capital charge was certain percentage of net assets assigned to department; it corresponded to an imputed interest charge. The criteria formulated to guide development of satisfactory measure of profitability were expressed as follow-

1. An index that recognized contribution of capital investment to profits.2. An index that recognized what human work and effort contribute to profits.3. An index that recognized the “Corporate facts of life” (for example one consistent with General Electric’s needs

and organizational objectives.)4. An index that served to make operating decisions of individual managers in company’s best interests.

In process of selecting and developing a measure of profitability, measurements organization considered several more conventional indexes, including rate of return on investment, ratio of profit to sales, and ratio of profit to value added.

Weakness of these ratios or indices was stated in this way-

Acid test of an index should be its effectiveness in guiding decentralized management to make decisions in best interests of company overall, operating managers efforts will be to improve performance of their businesses in terms of index used for evaluation. This test points up weaknesses of rate of return and other ratio indexes such as percent profit to sales. This weakness is tendency to encourage concentration on improvement of ratios rather than on improvement in dollar profits. Business with better results in terms of ratios will tend to make decisions based on effect decisions will have on particular business’s current ratio without consideration of dollar profits involved. This tends to incentive to growth and expansion because it dampens incentive of more profitable businesses to grow.

Q.2 What if anything do factors other than profitability add to proposed measurement system? Isn’t impact of other factors reflected in profitability measure if it is properly constructed?

Answer-

Other factors of key result areas which add to proposed measurement systems are as follows-

Market Position

Performance in this key result area was measured in terms of share of market obtained during given measurement period. The measurement was expressed as percentage of available business in market.

The first major consideration in designing market position measurement is determination of what constitutes a product line and what constitutes market for each product line of business. A product line may be defined as grouping of products in accordance with purposes they serve or essential wants they satisfy. This definition is some what misleading in that product line may be a broad classification, such as clocks, or it may be narrow classification such as alarm clocks, kitchen clocks or mantel clocks. Product lines may overlap so that particular product could be included in several product lines. Hence actual grouping of products by product lines must be accurately identified.

There may be wide variations in interpretation of what constitutes market for given product line. Therefore it is important that for each of their lines product departments must identified such things as follows-

1. Whether market includes not only directly competing products but also indirectly competing products (electric ranges versus electric ranges, electric ranges versus all types of ranges – electric, gas, oil, etc.)

2. Whether market includes sales by all domestic competitors or only those reporting to trade associations.3. Whether market includes imports, if foreign sellers are competing in domestic market.4. Whether market includes export sales, captive sales etc.

In establishing measurements of market position there should be a clear understanding of precisely what comprises product line and what comprises market. Purpose of having these items is to avoid being misled into thinking we are doing better than we actually are simply because of failure to identified nature and extent of competition.

66

Page 67: MCS theory

Productivity

Concept of productivity is relatively a relationship of output of goods and services to resources consumed in their production. For national economy as a whole it has been practice to look at productivity in terms of amount of output per unit of labour input. In any firm labour is only one factor contributing to output. Therefore company sought to develop an index that would accomplish two things-

1. Broaden input base so as to recognize that capital as well as labour contributed to improvements in productivity.

2. Eliminate from measure those improvements contributed by suppliers of materials.

On output side of productivity Ratio Company considered refinements of sales billed. One such refinement was use of value added.

On Input Side Company considered payroll dollars plus depreciation dollars. Payroll dollars were included in variable rather than labour hours so as to give effect to differences in labour skills employed. All factors were readjusted for changes in price level so that changes in resulting ratio would more nearly reflect real changes in productivity.

Product Leadership

It is defined as “ability of business to lead its industry in originating or applying most advanced scientific and technical knowledge in engineering, manufacturing, marketing, fields to development of new products and to improvements in quality or value of existing products. Procedures were established for appraising periodically products of each department.

1. How did each product compare with competition and with company standards?2. Where within company was research conducted upon which product was based?

Appraisal procedures were based on qualitative rather that quantitative considerations. It was made by experts from areas of engineering, marketing accounting etc.

Personnel Development

It defined as “systematic training of managers and specialists to fill present and future needs of company, to provide for further individual growth and retirements and to facilitate corporate growth and expansion”. Management of General Electric define personnel development as including “programs in each field of functional endeavor such as engineering, manufacturing, marketing, and finance and broad programs aim at developing an understanding of principle of managing such programs must be designed to provide continuous flow of potentially promotable employees in sufficient numbers to permit proper selection and development of individual for each position.

Three steps were involved in measurement of performance in this key result area-

1) The basic soundness of various programs or techniques being sponsored by product department for development of its employees was appraised.

2) An inventory was taken of available supply of trained men, as well as their qualifications, for key positions that must eventually be filled within department.

3) Effectiveness with which department executed its personnel development programs was evaluated.

First step consisted of judgments regarding adequacy of following elements in development process-

Recruitment - How good a job was being done in selection of candidates for development process?

On the Job training – What programs were available for training candidates, for providing information and knowledge about both general company matters and job particulars, and for advance training for those who had been on job for while?

Review and Counsel – Was there any provision for periodically reviewing performance of men, for discussing with an individual caliber of his work, for providing help and consultation and for identifying especially promising talent?

Placement- What was being done to see that recruits were placed in jobs commensurate with their interest and abilities that more promising was rotated and that promotion came to those who merited them?

67

Page 68: MCS theory

Second step was accomplished with aid of manning table and related inventorying procedures. These procedures were directed primarily at determining training background of each employee in inventory. Investigating group uses two statistical measures in carrying out third step. The first of these was ratio of number of employees promoted in given period to total number of employees regarded as “promotable”. The second measure was tied in with personnel rating procedure employed throughout company. At conclusion of each performance review cycle rating forms for particular department were analyzed to determine proportions of employees whose performance was considered to be a a)Improving b) Unchanged c) deteriorating.

Employee Attitudes

For purposes of developing measurements of performance in this key result area group defined an attitude as “a generalized point of view toward objects, events, or persons which might serve to influence future behavior”. It used two basic approaches to measurement of attitudes. First involve use of statistical indicators, such as turnover rate, absenteeism, number of grievances, lateness, and accident experience. Second approach involved a periodic survey of employees through questionnaires.

Several shortcoming were recognized in first approach

1) The statistical indicators provided little or no information about underlying causes.2) In general indicators told of trouble only after harm had been done.3) Because these indicators were traditionally associated with personnel functions managers tended to minimize

their importance or else place responsibility for improvement on personnel function.4) Unfavorable trends in certain of these indicators might be due to external factors for example short labour

supply rather than to some shortcoming of management.

Public Responsibility: This key result area evolve from General Electric’s recognition of its obligation to conduct itself as good citizen within society, complying faithfully with laws and ethics governing business conduct. Responsibility to certain public such as shareowners, educational institutions, and federal government could best be measure from overall company viewpoint rather than departmentally, actions taken by product department could have an important impact on whole company’s reputation as good corporate citizen.

The company attempted to assure wholehearted observance of legal and ethical standards of business by insisting that all managerial and professional employees conducted surveys of activities of those reported with respect to antitrust compliance, conflict of interest, and other areas of business practices.

Other measurements related to effectiveness of department action in strengthening company’s reputation and business relationships. With respect to fulfilling obligations to customers it was determined that product leadership and market position areas were best indicators. For remaining publics following measures were recommended.

Shareowners: The total shares of General Electric Company stock were to be “ allocated” to the various operating components that were assigned responsibility for preserving and enhancing “ their portion” of shareowners” investment in company.

Vendors, Dealers, and Distributors

Suppliers of raw materials and parts were to be surveyed periodically to determine their appraisal of department’s practices in conducting its business as compared with practices of others who bought from them. Dealers and distributors were likewise to be interviewed from time to time measure whether these important relationships were being responsibly maintained.

Plant Community

Comprehensive reaction surveys were to be used, aimed at identifying impact of actions of product development on individual who made up community. These reactions disclosed by opinion surveys were to be supplemented by use of trends developed from various types of data such as community wage rates, number of employment applications received, volume of purchases made locally, contributions to local charities, and participation in church, business organization.

Balance between Short- Range and Long-Range Goals:This factor was set out separately as a key result area in order to emphasize importance of long-term survival and growth of company. Short range goals and performance had to be balance against need for satisfactory performance 5, 10, 15 years in future, since undue pressure for current profits could borrow from future. When eight key result areas were established, each of first seven had both short-range and long-range dimensions. The eight area balance between short-range and long-range dimensions goals had been specifically identified to make sure that long-range health of company would not sacrificed for short-term gains. Plans, goals, and actions in each area were to be appraised in terms of both short-term and long-term implication.

68

Page 69: MCS theory

Initial Implementation

The company’s business planning, budgeting, and forecasting program incorporated use of selected key result areas in –

1) Reviewing recent history and current status2) Setting standards for each department3) Planning to achieve standard4) Periodic reporting and measurement of accomplishment.

Since first four key result areas that are profitability, market position, productivity, product leadership are lent themselves readily to numerical evaluations, they were a part of planning, budgeting, forecasting, reporting, measuring system. Building on this experience in using key result areas to plan and measure performance, management, at General Electric Company made search for effective business measurements a continuing evolutionary process

Birch Paper Company

"If I were to price these boxes any lower than $480 a thousand," said James Brunner, manager of Birch Paper Company's Thompson Division, "I'd becountermanding my order of last month for our salesmen to stop shaving their bids and to bid full-cost quotations. I've been trying for weeks to improve the quality of our business, and if I turn around now and accept this job at $430 or $450 or something less than $480, I'll be tearing down this program I've been working so hard to build up. The division can't very well show a profit by putting in bids that don't even cover a fair share of overhead costs, let alone give us a profit."

Birch Paper Company was a medium-sized, partly integrated paper company, producing white and kraft papers and

paperboard. A portion of its paperboard output was converted into corrugated boxes by the Thompson Division,

which also printed and colored the outside surface of the boxes. Including Thompson, the company had four

producing divisions and a timberland division, which supplied part of the company's pulp requirements.

For several years, each division had been judged independently on the basis of its profit and return on investment.

Top management had been working to gain effective results from a policy of decentralizing responsibility and

authority for all decisions except those relating to overall company policy. The company's top officials believed that

in the past few years the concept of decentralization had been applied successfully and that the company's profits

and competitive position definitely had improved.

The Northern Division had designed a special display box for one of its papers in conjunction with the Thompson

Division, which was equipped to make the box. Thompson's staff for package design and development spent

several months perfecting the design, production methods, and materials to be used. Because of the unusual color

and shape, these were far from standard. According to an agreement between the two divisions, the Thompson

Division was reimbursed by the Northern Division for the cost of its design and development work.

When all the specifications were prepared, the Northern Division asked for bids on the box from the Thompson

Division and from two outside companies. Each division manager was normally free to buy from whatever supplier

he wished, and even on sales within the company, divisions were expected to meet the going market price if they

wanted the business.

During this period, the profit margins of such converters as the Thompson Division were being squeezed

Thompson, as did many other similar converters, bought its paperboard, and its function was to print, cut, and

shape it into boxes. Though it bought most of its materials from other Birch divisions, most of Thompson's sales

were made to outside customers. If Thompson got the order from Northern, it probably would buy its linerboard and

corrugating medium from the Southern Division of Birch. The walls of a corrugated box consist of outside and inside

sheets of linerboard sandwiching the fluted corrugating medium.

69

Page 70: MCS theory

About 70 percent of Thompson's out-of-pocket cost of$400 for the order represented the cost of linerboard and

corrugating medium. Though Southern had been running below capacity and had excess inventory, it quoted the

market price, which had not noticeably weakened as a result of the oversupply. Its out-of-pocket costs on both liner

and corrugating medium were about 60 percent of the selling price.

The Northern Division received bids on the boxes of $480 a thousand from the Thompson Division, $430 a

thousand from West Paper Company, and $432 a thousand from Eire Papers, Ltd. Eire Papers offered to buy from

Birch the outside linerboard with the special printing already on it, but would supply its own inside liner and

corrugating medium. The outside liner would be supplied by the Southern Division at a price equivalent of $90 a

thousand boxes, and it would be printed for $30 a thousand by the Thompson Division. Of the $30, about $25

would be out-of-pocket costs.

Since this situation appeared to be a little unusual, William Kenton, manager of the Northern Division,

discussed the wide discrepancy of bids with Birch's commercial vice president. He told the vice president: "We sell

in a very competitive market, where higher costs cannot be passed on. How can we be expected to show a decent

profit and return on investment if we have to buy our supplies at more than 10 percent over the going market?"

Knowing that Mr. Brunner on occasion in the past few months had been unable to operate the Thompson

Division at capacity, it seemed odd to the vice president that Mr. Brunner would add the full 20 percent overhead

and profit charge to his out-of-pocket costs. When he was asked about this, Mr. Brunner's answer was the

statement that appears at the beginning of the case. He went on to say that having done the developmental work

on the box, and having received no profit on that, he felt entitled to a good markup on the production of the box

itself.

The vice president explored further the cost structures of the various divisions. He remembered a comment

that the controller had made at a meeting the week before to the effect that costs which were variable for one

division could be largely fixed for the company as a whole. He knew that in the absence of specific orders from top

management Mr. Kenton would accept the lowest bid, which was that of the West Paper Company for $430.

However, it would be possible for top management to order the acceptance of another bid if the situation warranted

such action. And though the volume represented by the transactions in question was less than 5 percent of the

volume of any of the divisions involved, other transactions would conceivably raise similar problems later.

70

Page 71: MCS theory

1. Which bid should Northern Division accept that is in the best interests of Birch Paper Company?

Sol: Calculating all three options based on costs

BIRCH PAPER COMPANY

Per 1,000 boxes

WEST PAPER CO: Out of pocket Cost to Birch $430

EIRE PAPER, LTD: $432

Less: Southern profit ($90*40%) $36

Thompson profit ($30-$25) 5 $41

Out of pocket cost to Birch $391

THOMPSON $480

Less: Southern profit ($280*40%) $112

Thompson profit ($480-$400) 80 $192

Out of pocket cost to Birch $288

As shown in the calculations above, Northern should accept the bid from Thompson division as it has the

lowest cost if all transfer prices within the company were calculated at costs. Incurring the lowest costs would also

enable Birch Paper Company to earn the highest profits possible.

71

Page 72: MCS theory

2. Should Mr. Kenton accept this bid? Why or why not?

Sol: As alternatives for sourcing exists, Mr. Kenton should be permitted to choose the alternative that is in Northern

division's own interests. The transfer price policy gives him the right to deal with either insiders or outsiders at his

discretion. If he is unable to get a satisfactory price from the inside source which is Thompson division, he is free to

buy from outside.

Mr. Kenton, manager of the Northern division should not accept the bid from Thompson division. The three

bids from Thompson division, West Paper Company and Eire Paper Company are $480, $430 and $432

respectively. Accepting the bid from Thompson division would be accepting the highest bid amongst all three offers

(highest costs). This would result in the lowest profits. As the Northern division is evaluated as an investment

center, it is judged independently on the basis of its profit and return on investment. Mr. Kenton should not accept

the bid from Thompson division.

It is in the best short-term economic interests of the Northern Division to accept the bid of West Paper

Company that is this will give Northern Division its maximum profit. Assume that the transfer price policy of Birch

Paper is to transfer at market price (adjusted for any internal savings

3. Should the vice president of Birch Paper Company take any action?

Sol: The method of using transfer price to decide whether to in source is optimum if the selling profit center can sell

all of its products to either insiders or outsiders and if buying center can obtain all of its requirements from either

outside or insiders. The market price then represents the opportunity costs to the seller of selling the product inside.

In this case, Thompson division had been running below capacity and Southern division also had excess inventory.

The transfer price of $480 offered by Thompson division does not represent the opportunity costs of selling inside

as there is no demand market for the product outside. Also, the transfer price of $480 is higher than the market

price which is around $430. Deciding based on transfer price will not induce goal congruence as the situation is not

ideal.

Without any intervention from the vice president of Birch Paper Company, the Northern division would most

probably accept the lowest bid from West Paper Company. This might result in the highest profits for Northern

division but it is not in the best interests of Birch Paper Company. Accepting the bid from Thompson division would

boost demand for the two other divisions. The losses cut would most probably be more than the costs saved by

Northern division which is $50 ($480-$430).

The vice president should give specific orders to Northern division to accept the bid from Thompson

division. However, as the transaction in this case represents less than 5% of the volume of any of the divisions

72

Page 73: MCS theory

involved, it might not be possible for the vice president to intervene other transactions when similar problems arise.

4. In the controversy described, how, if at all, is the transfer price system dysfunctional? Doest his

problem call for some change, or changes, in the transfer pricing policy of the overall firm? If so,

what specific changes do you suggest?

Sol: Ideally, when there is an availability of market price, the division should use it. However, Thompson used a

cost-based transfer price instead. Cost-based transfer price should only be used when the market price is not

available. The problem with Birch's transfer pricing system is that they allow each division to set their own price

freely which is inline with the company's policy to decentralize responsibility and authority. When each division can

set their own price, conflicts and disagreements can occur on a frequent basis and each division could make

decisions that only benefit their own division rather than the company as a whole.

Firstly, we look at the transfer price that Thompson quoted. It is about $50 more than the market price. This

shows that their price is not competitive enough. Thompson is operating below capacity and yet it quoted a price

which is higher than the market price. The reason given was that anything less than $480, they will not be able to

earn a profit and also, given that they did not get any profit from developing the product for Northern, Brunner feels

that they are entitled to a good markup. This is inconsistent with the expectation that the division must meet the

market price if they wanted the business. Market price should be used as it reflects how well is the division doing as

compared to competitors.

The amount of upstream fixed costs and profits that are included in the final price that was sold to the

outside customer could be substantial if Thompson's bid was accepted. And Northern might not be willing to reduce

its own profit to optimize company profit. Hence, Thompson, if unwilling to follow the market price blindly, could use

the two-step pricing to calculate their transfer price. That is, transferring the goods to Northern on standard variable

cost on a per unit basis and fixed cost and profit on a lump sum basis.

In this way, Thompson will not be transferring majority of their fixed cost to Northern because they are

operating on excess capacity. But of course, this method must be discussed with Northern.

It was mentioned that Southern quoted the market price to Thompson even though they are operating on

excess capacity. This will not pose a problem as the market price reflects the demand and supply situation of the

market and is adjusted automatically by the demand and supply. Also, account must be taken into of the fact that

Thompson will not be able to get a better price from other outside sources as most will follow the market price too.

The underlying problem of the transfer price system could be that each division is judged based on profits

and return on investment. This causes the division to over-emphasize on profits and encourages goal

incongruence. Each division aims at achieving short-term profits so as to look better in the company's eyes. In their

bid to achieve a high profit figure, they fail to optimize the company's profit as a whole. This will affect the company

long-term profits.

Hence, the company should not just assess each division based solely on financial figures like profit and return on investment. The company should assess them based on other non-financial things like quality so as to divert the division's emphasis on profits. In addition, the company should allow the divisions concerned to negotiate between

73

Page 74: MCS theory

themselves as they are the ones closest to the situation, rather than just asking the divisions to meet the market price.

74